Download as pdf or txt
Download as pdf or txt
You are on page 1of 198

1600+ English All Previous Year Questions

SCORE BOOSTER PDF


ODISHA JOB INFO (SUNIL SIR)
Exam Name Page No
1. OSSSC PEO & JA Exam 2023 (35 MCQ) ---------------------------------------------------------------------------- 2 - 6
2. OSSSC ARI, AMIN, SFS, FG, EC Exam 2021 (35 MCQ) ---------------------------------------------------------- 6 - 11
3. OSSSC Livestock Inspector Exam 2021 (10 MCQ) -------------------------------------------------------------- 11 - 12
4. OSSSC REVENUE INSPECTOR Exam 2021 (50 MCQ) ----------------------------------------------------------- 12 - 19
5. OSSSC Junior Assistant Exam 2018 (25 MCQ) ------------------------------------------------------------------ 19 - 22
6. OSSSC JAIL WARDER Exam 2017 (15 MCQ) --------------------------------------------------------------------- 22 - 24
7. OSSSC ICDS SUPERVISOR Exam 2016 (10 MCQ) --------------------------------------------------------------- 24 - 25
8. OSSSC Statistical Field Surveyor Exam 2016 (15 MCQ) ------------------------------------------------------ 25 - 27
9. OSSSC Junior Clerk Exam 2015 (40 MCQ) ----------------------------------------------------------------------- 27 - 32
10. Jail Warder Exam 2022 (180 MCQ) ----------------------------------------------------------------------------- 32 - 49
11. Sub Inspector Excise Exam 2022 (225 MCQ) ----------------------------------------------------------------- 49 - 75
12. Traffic Police SI Exam 2022 (170 MCQ) ------------------------------------------------------------------------ 75 - 99
13. OPRB SI Exam 2018 (160 MCQ) --------------------------------------------------------------------------------- 99 - 116
14. OSSC Junior Executive Assistant Exam 2022 (225 MCQ) -------------------------------------------------- 116 - 145
15. OSSC CGL Exam 2021 (240 MCQ) ------------------------------------------------------------------------------- 145 - 182
16. OSSC SCEW Exam 2022 (38 MCQ) ------------------------------------------------------------------------------ 182 - 188
17. OPRB SI Exam 2019 (40 MCQ) ------------------------------------------------------------------------------------ 188 - 192
18. OPRB SI Exam 2021 (40 MCQ) ------------------------------------------------------------------------------------ 192 - 196
19. Fireman Exam 2023 (15 MCQ) ------------------------------------------------------------------------------------ 196 - 198
OSSSC PEO & JA Exam 2023 (c) Aplication

Directions (Q1 to Q4): Find out the wrongly spelt (d) Applicasion
word.
Ans - A
1. (a) National
6. (a) Addmision
(b) Anthem
(b) Admission
(c) Business
(c) Admision
(d) Trycolour
(d) Admmision
Ans - D
Ans - B
2. (a) Usual
7. (a) Wellcome
(b) Actual
(b) Welcom
(c) Arival
(c) Welcome
(d) Approval
(d) Wellcom
Ans - C
Ans - C
3. (a) Patrol
8. (a) Urggent
(b) Incom
(b) Urrggent
(c) Village
(c) Urrgent
(d) Enough
(d) Urgent
Ans - B
Ans - D
4. (a) Independenc
Directions (Q9 to Q12): Choose the most
(b) Foreigner correct/appropriate option.

(c) Partition 9. I _______ have won the match; but sadly I lost.

(d) Trader (a) will

Ans - A (b) dare to

Directions (Q5 to Q8): Find out the correctly spelt (c) need to
word.
(d) could
5. (a) Application
Ans - D
(b) Applecation
10. There _______ be a famous sweet shop here 20
years back. (b) completely

(a) was to (c) Completing

(b) would (d) completion

(c) used to Ans - A

(d) should 14. (a) regularity

Ans - C (b) regulately

11. We _____ always respect our elders. (c) regulatingly

(a) could (d) regularly

(b) should Ans - D

(c) can 15. (a) sufferingly

(d) may (b) suffer

Ans - B (c) suffering

12. All students _______ obey the school (d) suffered


regulations to maintain discipline.
Ans - B
(a) must
16. (a) taken
(b) are able to
(b) taking
(c) could
(c) take
(d) will
(d) took
Ans - A
Ans - C
Directions (Q13 to Q20): Fill in the blanks of the
paragraph written below with most 17. (a) affordance
suitable/appropriate word from the options given
against each question as indicated in the blanks. (b) affording

"Milk is a (Q13) food, full of vitamins and proteins. (c) afford


Children should drink it (Q14) otherwise they might
(Q15) from under-nourishment. Elders must also (d) afforded
(Q16) it; but those who cannot (Q17) it should eat
fruits and vegetables as an (Q18). We should (Q19) Ans - C
milk of good quality from milk booths (Q20) by
government." 18. (a) alternation

13. (a) complete (b) alteration


(d) Perhaps wheel is man's greatest the invention.
(c) alter
Ans - A
(d) alternative
23. very difficult/it was / to travel / earlier / long
Ans - D distances

19. (a) purchase (a) It was very difficult earlier travel to long
distances.
(b) purchasingly
(b) It was difficult to very travel long distances
(c) purchased earlier.

(d) purchasing (c) Earlier long distances was very difficult to travel
it.
Ans - A
(d) Earlier it was very difficult to travel long
20. (a) openingly distances.

(b) opened Ans - D

(c) open 24. backs / their belongings / travellers carried/ on


their
(d) opening
(a) Belongings carried their travellers on their
Ans - B backs.

Directions (Q21-Q24): Arrange the words (parts of (b) Travellers carried their belongings on their
a sentence) properly to form a correct sentence. backs.

21. few places/take rest/on the/ there (c) Their travellers carried on belongings on their
were/to/way backs.

(a) Places there to take rest were few on the way. (d) Their backs carried their belongings on
travellers.
(b) There were few places the way to take rest on.
Ans - B
(c) There were few places to take rest on the way.
Directions (Q25 to Q27): Fill in the blanks of the
(d) The way to take rest on few places were there. paragraph written below with the most
suitable/appropriate passive voice from the
Ans - C options given against each question as indicated in
the blanks.
22.
wheel/is/man's/the/invention/greatest/perhaps "How to make a cup of tea In order to make a cup
of tea, boiled water (Q.25). Then a spoonful of
(a) The wheel is perhaps man's greatest invention. sugar (Q.26) to it. Some milk (Q.27). A cup of tea is
ready."
(b) The wheel is man's greatest perhaps invention.
25. (a) pour into a cup
(c) Perhaps the greatest invention is man's wheel.
(b) is poured into a cup
(c) has been poured into a cup (d) streams flowing round the year

(d) had been pour into a cup Ans - D

Ans - B 30. Amongst “seasonal” fruits, I like mango the


best for its sweetness.
26. (a) will be added
(a) not available in all seasons
(b) add
(b) available in the local area
(c) is added
(c) available in all seasons
(d) added
(d) tastes sweet
Ans - C
Ans - A
27. (a) is put in it and stirred
Directions (Q31 to Q35): Fill in the blanks with the
(b) is putted in it and stir most appropriate option given.

(c) had been put in it and stirred 31. They ______ the train if they do not hurry.

(d) put in it and stir Directions (a) will miss

Ans - A (b) will have missed

(Q28 to Q30): Choose the best/appropriate (c) are missing


meaning of the underlined words.
(d) miss
28. As directed by my boss, I have prepared the
meeting “agenda”. Ans - A

(a) list of food items be served in the meeting 32. I try to visit my relatives in my home town
______ other month.
(b) issues to be discussed in the meeting
(a) any
(c) persons to be present in the meeting
(b) every
(d) agencies to be present in the meeting
(c) each
Ans - B
(d) some
29. Koraput district of Odisha, which is a very hilly
district, boasts of many “perennial” streams that Ans - B
help in local agriculture.
33. The dentist cleaned my _____ with some liquid.
(a) streams supplying water for irrigation
(a) teeth
(b) streams flowing only during rainy season
(b) teeths
(c) streams flowing from hills and forest
(c) tooth Ans - C

(d) tooths 82. The patient is admitted in hospital for acute


_____
Ans - A
(a) diarhoca
34. The crowd ____ restless as the day got hotter.
(b) diarrhoea
(a) were getting

(c) was getting (c) diarrhia

(b) will get (d) dyrrhoca

(d) is getting Ans - B

Ans - C 83. The boy was punished for _____ of Similar


Mistakes.
35. Your writing is bad but mine is _____
(a) repitition
(a) worst
(b) repeatation
(b) better

(c) badder (c) Repitation

(d) worse (d) repetition

Ans - D Ans - D

84. Any _______ or ______ in the affidavit will


attract penalty.

(a) ommission, commission


OSSSC ARI, AMIN, SFS, FG, EC Exam 2021
(b) omission, commission
Fill in the blanks with appropriate and correctly
spelt words. (c) omission, comission

81. Gandhiji's death was an _____ loss to the (d) ommision, commision
country.
Ans - B
(a) irrepairable
85. The terrible news of Indira Gandhi's __ spread
(b) irepairable like a flash of lightening across the world.

(c) irreparable (a) asassination

(d) iriparable (b) assasination


(c) assassination (a) is

(d) asasination (b) are

Ans - C (c) were

Directions (Q86 to Q90): Fill in the blanks with the (d) shall
correct form of the verb.
Ans - A
86. A pair of shoes _______ lying in the corner.
90. The crew ______ rescued from the sunken ship.
(a) are
(a) was
(b) were
(b) is
(c) is
(c) were
(d) am
(d) am
Ans - C
Ans - A
87. It is you who, _____ cheated me.
91. The word 'insolvent' means
(a) is
(a) rich
(b) has
(b) dangerous
(c) was
(c) notorious
(d) have
(d) bankrupt
Ans - D
Ans - D
88. Neither of the boys _____ won a prize.
92. Morning shows the day.
(a) have
(Choose the appropriate Odia meaning of the
(b) has above English proverb)

(c) was (a) ଅର୍ଥ ଅନର୍ଥ ର ମୂଳ

(d) are (b) ତୁ ଳସୀ ଦୁ ଇପତ୍ରରୁ ବାସସ

Ans - B (c) ସ ାର ଯାହାର, ମୂଳକ ତାହାର

89. Sixty years _______ a long time for a country to


(d) ଧୂର ପାଣି ପାରି କାସେ
grow and prosper.
Ans - B
93. He was born with a silver spoon in his mouth. Fill in the blanks with appropriate prepositions.

(Find out the appropriate meaning of the above 96. Beware _____ the pickpockets.
sentence)
(a) from
(a) He was born in a rich family.
(b) of
(b) He was born in a poor family.
(c) into
(c) He was born in a middle-class family.
(d) at
(d) He was born with the congenital disorder in his
mouth. Ans - B

Ans - A 97.I prefer death ______ dishonor.

94. ପାପଧନ ପ୍ରାୟଶ୍ଚି ତସର ଯାଏ। (a) than

(Translate the above proverb in Odia into English.) (b) from

(a) A penny saved is a penny gained. (c) to

(b) As you sow, so you reap. (d) in

(c) Take care of the pence and the pounds will take Ans - C
care of themselves.
98. He could not comply ______ my demand.
(d) Ill got, ill spent.
(a) to
Ans - D
(b) with
95. Raja Ram Mohan Roy was an atheist.
(c) upon
(Choose the correct meaning of the underlined
word.) (d) against

(a) reformer Ans - B

(b) rebel 99. I could not “ lay by “ anything for future due to
my family compulsions.
(c) believer in God
(Choose the correct meaning of the underlined
(d) non-believer in God phrase.)

Ans - D (a) save

Directions (Q96 to Q98): (b) utilise


(b) Lion is a beast of prey.
(c) gain
(c) The lion is a beast of prey.
(d) plan
(d) The lion is the beast of prey.
Ans - A
Ans - C
100. The Commission decided to “ abolish “ the
prevailing system of recruitment examination. 104. Autobiography' means:

(Choose the appropriate phrasal verb of the (a) The history of the life of a person
underlined word)
(b) A record of one's life written by himself
(a) deal with
(c) A piece of writing or speech in praise of a
(b) deal in person

(c) do away with (d) A person's own hand-writing

(d) put forth Ans - B

Ans - C 105. Happiness or sadness is “ transitory “ in


human life.
Directions (Q.101 to Q.103): Find out the sentence
which is grammatically correct. (Find out the correct meaning of the underlined
word.)
101. (a) He is senior than me by two years.
(a) permanent
(b) He is more senior than me by two years
(b) pre fixed
(c) He is senior to me by two years.
(c) short lived
(d) He is senior than me for two years.
(d) inevitable
Ans - C
Ans - C
102. (a) I as well as John is to blame.
106. A person who lives at the same time with
(b) as well as John are to blame. another :

(c) as well as John were to blame. (Substitute one word for the above)

(d) I as well as john am to blame. (a) contemporary

Ans - D (b) accomplice

103. (a) Lion is beast of prey. (c) activist


(d) adjutant Direction (110 to 112): Transform the sentences
into Indirect speech.
Ans - A
110. He said to me, "May you be happy."
107. Prevention is better than “ cure “ (Choose the
correct part of speech of the underlined word) (a) He said to me that I may be happy.

(a) verb (b) He wished me that I might be happy.

(b) noun (c) He wished me that you might be happy.

(c) adjective (d) He said to me that I might be happy

(d) adverb Ans - B

Ans - B 111. Rama said to me, "Are you going there?"

108. The man was beaten “ black and blue “ by the (a) Rama said to me that you were going there.
villagers.
(b) Rama asked me whether I was going there.
(Choose the correct meaning of the underlined
phrase) (c) Rama asked me whether you were going there.

(a) slowly (d) Rama said to me whether you were going there.

(b) moderately Ans - B

(c) severely 112. The teacher said to me, "You are wrong."

(d) occasionally (a) The teacher said to me that you were wrong.

Ans - C (b) The teacher told me that you were wrong.

109. The statement of the Principal was (c) The teacher told me that he was wrong.
communicated to all in “ black and white “.
(d) The teacher told me that I was wrong.
(Choose the correct meaning of the underlined
phrase) Ans - D
(a) writing
113. ______ sun rises in ___ east.
(b) speech
(Fill in the blanks with appropriate Articles.)
(c) vain
(a) A, an
(d) protest
(b) The, the
Ans - A
(c) The, an
(a) with
(d) A, the
(b) on
Ans - B
(c) in
114. What is the feminine gender of fox'?
(d) about
(a) bitch
Ans - C

(b) ewe 33. They have “ postponed “ the meeting because


of bad weather.
(c) heifer
(Choose the correct phrasal verb of the underlined
(d) vixen word

Ans - D (a) put up

115. The room needs repairing. (b) put on

(c) put down


(a) The room needs to be repaired.
(d) put off
(b) Repairing is needed for the room.
Ans - D
(c) "The room is being repaired.
34. Which of the following sentences is
(d) The room has been repaired. grammatically correct?

Ans - A (a) Ganges is considered holy by the Hindus

Livestock Inspector 2021 (b) The Ganges is considered holy by the Hindus.

31. None “ but “ the brave deserves the fair. (c) The Ganges are considered holy by the Hindus.
(Choose the correct part of speech of the
underlined word) (d) The Ganges is considered holy by Hindus.

(a) noun Ans - B

(b) pronouns 35. John Hawkins is ____ European.

(c) preposition (Fill in the blank with appropriate Article.)

(d) conjunction (a) an

Ans - C (b) a

32. Hari deals _____ rice and vegetables in the (c) the
market. (Fill in the blank with appropriate
preposition.) (d) none of these

Ans - B
(c) are
36. His writing is “ such that cannot be read” .
(d) have
(Substitute one word for the underlined words)
Ans - D
(a) legible

(b) ineligible

(c) illegible

(d) eligibl

Ans - C

37. The word 'pessimist' means-

(a) A person who believes in the existence of God REVENUE INSPECTOR 2021

(b) A person who does not believe in the existence Instructions for questions from No. 1 to 5: Read the
of God sentences and transform them from Direct speech
to Indirect speech.
(c) A person who looks at the dark side of life or
things 1. Rama said to me, "You will be rewarded".

(d) A person who looks at the bright side of life or (a) Rama told me that you will be rewarded.
things
(b) Rama told me that I would be rewarded.
Ans - C
(c) Rama told me that I will be rewarded.
39. The black cobra is a _____ snake.
(d) Rama told me that he will be rewarded.
(Fill in the blank with correct spelling of the word.)
Ans - B
(a) venomus
2. He said to me, "Are you well?"
(b) venomous
(a) He asked me if you were well
(c) vanomous
(b) He asked me if he was well.
(d) vanomus
(c) He asked me if I was not well.
Ans - B
(d) He asked me if I was well.
40. The poultry _____ not been fed.
Ans - D
(Fill in the blank with correct form of verb.)
3. My friend said to me, "Let us go there
(a) has tomorrow."

(b) is (a) My friend proposed that we should go there the


next day.
7. (a) No sooner had I left home than it began to
(b) My friend told me to go there tomorrow. rain.

(c) My friend ordered me to go there the next day. (b) No sooner I left home than it began to rain.

(d) My friend asked me to go there the next day. (c) No sooner had I left home it began to rain.

Ans - A (d) No sooner i had left home than it began to rain.

4. He said, "How beautiful the rose is!" Ans - A

(a) He said that how beautiful the rose was! 8. (a) My father helped me to clean the room.

(b) He exclaimed that the rose was very beautiful. (b) My father helped me cleaning the room.

(c) He exclaimed that how beautiful the rose was. (c) My father helped me clean the room.

(d) He told that how beautiful the rose was! (d) My father helped me cleaned the room.

Ans - B Ans - A

5. The teacher said, "Honesty is the best policy." 9. (a) He asked me why have you come here?

(a) The teacher said that honesty was the best (b) He asked me why you have come here.
policy.
(c) He asked me why you had come here?
(b) The teacher said that honesty is the best policy.
(d) He asked me why I had come here.
(c) The teacher said that honesty should be the
best policy. Ans - D

(d) The teacher said that honesty would be the 10. (a) I enjoy to play the piano.
best policy.
(b) I enjoy playing the piano.
Ans - B
(c) I enjoy play the piano.
Instructions for questions from No. 6 to 10: Go
through the sentences and find out which one is (d) I enjoy played the piano.
grammatically
Ans - B
correct.
Instructions for questions from No. 11 to 13: Read
6. (a) One should keep his promises. the sentences and choose the correct parts of
speech of the underlined words.
(b) One should keep our promises.
11. The function came to a “ close “ at half “ past “
(c) One should keep their promises. ten.

(d) One should keep one's promises. (a) verb, adjective

Ans - D (b) noun, verb


(c) noun, preposition “ rejected “ by the principal.

(d) verb, preposition (a) put on, turned up

Ans - C (b) put up, turned on

12. He is the “ first “ man to sit on a “ fast “. (c) put up with, turned down

(a) adjective, noun (d) put off, put down

(b) pronoun, noun Ans - C

(c) adjective, verb 16. Thieves “ forcibly entered “ our house last night
and escaped
(d) adjective, adverb
(a) broke down, got over
Ans - A
(b) broke up, got into
13. Jack and Jill went “ up “ the hill and looked “ up
“. (c) broke into, got away with

(a) adjective, adverb (d) broke off, got down

(b) preposition, adverb Ans - C

(c) adverb, preposition 17. Not only the boy but also his parents _______
responsible for lack of supervision, a result of
(d) verb, preposition which two-

Ans - B thirds of the ______ work done in a day.

Instructions for questions from No. 14 to 16: Read (a) were, was
the sentences and substitute the underlined words
with appropriate phrasal verbs (b) was, were

14. As soon as the teacher “ entered “ the room, all (c) were, were
the lights were “ extinguished “.
(d) was, was
(a) got into, put out
Ans - A
(b) reached at, blown away
18 The old man as well as his sons _____ at fault
(c) arrived at, blown over for this tragic incident for which a crowd of people

(d) walked up, turned on ______ gathered on the spot.

Ans - A (a) are, have

15. I couldn't “ tolerate “ such humiliation as the (b) is, has


president of the students Union for our proposal
was (c) is, have
(d) are, has (c) No other girl in the class is as good as mira.

Ans - B (d) Mira is better than the girls in the class.

19. I who ________ , his friend, must look to his Ans - B


interest.
23. He is too weak to walk. (Rewrite the sentence
(a) is removing 'too")

(b) was (a) He is so weak that he cannot walk

(c) are (b) He is very weak to walk.

(d) am (c) He is weak enough to walk.

Ans - D (d) He is so weak to walk.

20. All that glitters ______ not gold. Ans - A

(a) is 24. A barking dog seldom bites.

(b) are (Change it into complex sentence)

(c) were (a) A dog that bites seldom barks o

(d) was (b) A dog that barks seldom bites.

Ans - A (c) A dog that is barking seldom bites.

21. Slow and steady _____ the race, because haste (d) A biting dog seldom barks.
______ waste.
Ans - B
(a) win, makes
25. He does not like fish. He does not like meat.
(b) wins, makes Join the above two sentences using 'neither ------
nor’
(c) won, made
(a) He neither likes fish nor meat.
(d) winning, makes
(b) He does not like neither fish nor meat.
Ans - B
(c) He likes neither fish nor meat.
Instructions for questions from No. 22 to 30: Read
the sentences and do as directed. (d) He does neither like fish nor meat.

22. Mira is the best girl in the class. (Change the Ans - C
degree of comparison to comparative form)
26. The boy is very industrious. (Change it into
(a) Mira is better than most of the girls in the class. exclamatory sentence)

(b) Mira is better than any other girl in the class. (a) What an industrious boy is he!
(b) What an industrious boy he is!
30. The word 'carnivorous' means
(c) What a very industrious boy is he!
(a) An animal eating meat
(d) What a very industrious boy he is!
(b) A man-hunting animal
Ans - B
(c) A person eating human flesh
27. The police resorted to firing 10 “ disperse “ the
violent “ mob “. (d) A wild animal

(Choose the correct meaning of the underlined Ans - A


words)
Instructions for questions from No. 31 to 35: Fill in
(a) strike, pilgrims the blanks with appropriate Articles.

(b) punish people 31. ______ earth revolves round, ______ sun.

(c) dispel, crowd (a) The, a

(d) pacify, gathering (b) An, the

Ans - C (c) A, a

28. ______ the road and ______ the child. (d) The, the

(Fill in the blanks with an appropriate pair of verbs Ans - D


in the proverbial expression)
32. _______ bird in hand is worth two in ______
(a) Strike, scold bush.

(b) Spare, spoil (a) The, the

(c) Show, shake (b) A, the

(d) Spring, spurn (c) The, a

Ans - B (d) A, a

29. His father is a teacher. His mother is a teacher. Ans - B


(Join the above two sentences using 'both --------
and")
33. Dasaratha ________ , king of Ayodhya, was
(a) His both father and mother are teachers. ________ humble king.

(b) Both his father and mother are teachers. (a) the, an

(c) Both his father and mother is a teacher. (b) the, a

(d) His both father and mother is a teacher. (c) a, a

Ans - B (d) a, an
Ans - A (b) with, to

34. _________ honest man is ________ asset to (c) into, with


the society. king.
(d) upon, to
(a) The, a
Ans - A
(b) An, the
38. All of the teachers congratulated me _____ my
(c) An, an success, but never boast ______ it.

(d) The, the (a) for, of

Ans - C (b) on, for

35. Kalidasa, ________ great Sanskrit poet, is (c) on, of


________ Shakespeare of our country.
(d) for, into
(a) a, a
Ans - C
(b) the, the
39. The old man was very fond ______ her
(c) the, a daughter, who was deprived _______ her property
after father's death.
(d) a, the
(a) of, of
Ans - B
(b) at, of
Instructions for questions from No. 36 to 40: Fill in
the blanks with appropriate prepositions. (c) of, from

36. Although the man was accused _______ (d) with, from
murder by the police, he was acquitted _______
the charges in Ans - A

the court. 40. The woman was suffering _______ cancer, but
the doctor cured her _______ it.
(a) in, of
(a) from, of
(b) of, of
(b) with, from
(c) from, of
(c) from, with
(d) of, from
(d) of, from
Ans - B
Ans - A
37. They agreed ________ his proposal in the
meeting, but I did not agree _______ them. Instructions for questions from No. 41 to 45: Read
the sentences and fill in the blanks with
(a) to, with appropriate and
correctly spelt words. (d) miscellaneous, restaurant

41. An owl is a ______ bird, whose species are Ans - C


almost ____
45. Frequent power failure in the examination
(a) Nocturnal, extinct centre caused much ________ to the _______
invigilators and examinees.
(b) noctoral, extint
(a) inconvenience, superintendant
(c) nockurnal, extinct
(b) inconvenience, superintendent
(d) noctoral, extinguish
(c) inconvenice, superintendent
Ans - A
(d) inconvenience, supraintendent
42. Death of Sushant Singh Rajput has caused not
only a ________ in Indian film industry but also Ans - B
much _______ for the investigating police team.
Instructions for questions from No. 46 to 50: Read
(a) vaccum, embarasment the sentences and change their voice from Active
to Passive or vice versa.
(b) vacuum, embarrassment
46. The teacher made him stand up in the class.
(c) vaccuum, embarrasment
(a) The teacher was made him stand up in the
(d) vacume, embarrassment class.

Ans - B (b) He was made stand up by the teacher in the


class.
43. During winter, a large number of ________
birds, both colourful and ______ come from Siberia (c) He was made to stand up by the teacher in the
to Chilika Lake. class.

(a) migratory, exotic (d) He had been made to stand up by the teacher
in the class.
(b) migratory, excotic
Ans - C
(c) migretery, esoteric
47. I saw the monkeys climbing the tree.
(d) migratory, extinct
(a) The monkeys were seen climbing the tree.
Ans - A
(b) The tree was seen the monkeys climbing.
44. The clerk dealing with _______ works of the
office is occasionally seen in this _______ . (c) The monkeys were seen to climb the tree.

(a) misllaneous, resturant (d) The monkeys was seen to climb the tree.

(b) micelaneous, restaurant Ans - A

(c) miscellaneous, restaurant 48. The poor should be helped.


2. Before we .............our meal, he ............ us back
(a) You should help the poor. to work.

(b) He should help the poor. (Fill in the blanks with alternatives given below)

(c) They should help the poor. (a) finished, had ordered

(d) One should help the poor. (b) have finished, ordered

Ans - D (c) had finished, had ordered

49. It was proposed to hold a meeting. (d) had finished, ordered

(a) They proposed to hold a meeting. Ans - A

(b) They proposed it to hold a meeting. 3. ______ Camel Can go ________ long distance
without water.
(c) They proposed to be held a meeting.
(Select correct articles and fill in the blanks if
(d) They had proposed to hold a meeting. required.)

Ans - A (a) the, a

50. The prizes were given away by the chief guest. (b) A, the

(a) The chief guest was giving away the prizes. (c) A, a

(b) The chief guest gave away the prizes. (d) None of the above

(c) The chief guest was given away the prizes. Ans - C

(d) The prizes were given to the the chief guest. 4. I found ______ book last night, _______ book is
very useful.
Ans - B
(Select correct articles and fill in the blanks if
Junior Assistant 2018 required.)
1. If you examination ______ harder, you _______
(a) the, a
passed your examination.
(b) the, the
(Fill in the blanks with alternatives given below)
(c) a, the
(a) worked, have
(d) a, a
(b) had worked, had
Ans - C
(c) had worked, will have
5. I _____ when my friend ______ (Choose the
(d) had worked, would have
correct pair of verbs to complete the sentence)
Ans - D
(a) left, had arrived
(b) had left, arrived
(a) in, in (b) on, in
(c) left, arrive
(c) in, on (d) on, on
(d) had left, was arriving
Ans - B
Ans - B
10. The promise that We .......................Shall be
6. ______ I win, _______ You lose. (Choose the faithfully ______ .
appropriate nouns)
(Fill in the blanks with alternatives given below)
(a) Heads, tail
(a) made, to be kept
(b) Head, tails
(b) had made, keep
(c) Heads, tails
(c) have made, kept
(d) Head, tail
(d) are making, kept
Ans - C
Ans - C
7. He is indebted _____ his friend ........... a large
sum. (Choose appropriate prepositions). 11. Anyone can do it easily. (Choose the correct
one in passive voice)
(a) to, for
(a) It can be done by easily anyone.
(b) for, to
(b) It can be easily done by anyone.
(c) by, for
(c) It can be done easily by anyone.
(d) to, to
(d) Can it be done easily by anyone?
Ans - A
Ans - C
8. I could not _____ what he hinted at, but............
my mind to help him. (Choose the correct phrasal 12. He is a 'novice' in the trade. (Choose the correct
verb pairs) meaning of the underlined word)

(a) make in, made for (a) Expert

(b) make out, made up (b) New

(c) make for, made of (c) Experienced

(d) made out, made in (d) Old

Ans - B Ans - B

9. Jyoti is working ______ a farm, and her mother 13. The box is too heavy to be lifted.
is working ............. a bank.
(Remove too from the sentence and find the
(Choose the correct pair of prepositions) correct one from the choices given)
Ans - B
(a) The box is very heavy to be lifted.
17. What does a teetotaller' mean?
(b) The box is very very heavy to lift.
(a) A person who never smokes.
(c) The box is so heavy that it cannot be lifted.
(b) A person who never takes tea
(d) The box is very heavy that it cannot be lifted.
(c) A person who never drinks
Ans - C
(d) A person who never laughs
14. Very few cities in India are as rich as Mumbai.
Ans - C
(Change into comparative and find the new
sentence from the choices given) 18. He cannot be trusted with important secret
informations otherwise I would have made him my
(a) Mumbai is richer than other city in India. assistant.

(b) Mumbai is richer than most other cities in India. (Spot the error in a part of the sentence)

(c) Mumbai is as rich as other cities in India. (a) He cannot be trusted

(d) Mumbai is the richest city in India. (b) with important secret informations

Ans - B (c) otherwise I would have

15. What does Penultimate mean? (Choose the (d) made him my assistant
correct one)
Ans - B
(a) The last one
19. Being a rainy day, I decided to stop work, stay
(b) Last but one at home and enjoy a movie.

(c) Middle one (Spot the error in a part of the sentence)

(d) First one (a) Being a rainy day (c) Stay at home

Ans - B (b) I decided to stop work (d) And enjoy a movie

16. What is the meaning of 'Run down' in the Ans - A


following sentence?
20. He said, "I saw her the day before yesterday".
He runs me down everywhere. (Choose the correct one in indirect speech.)

(a) Praise (a) He said that he saw her the day before
yesterday
(b) Criticizes
(b) He said that he had seen her two days before.
(c) Discuss
(c) He said that he had seen her one day before
(d) Run away yesterday.
(d) He said that he has seen her two days before (a) Misogynist
yesterday.
(b) Optimist
Ans - B
(c) Philanthropist
21. Mrs. Sudha Murty is a versatile genius and also
known for her acts of 'Philanthropy'. What does (d) Pessimist

Philanthropy mean? Ans - B

(a) Philosophy 25. When winter comes, can spring be _______


(Complete the popular pro-verb)
(b) Charity
(a) much behind
(c) Honesty
(b) more behind
(d) Spendthrift
(c) far behind
Ans - B
(d) waiting behind
22. What is the singular form of 'data'?
Ans - C
(a) Data

(b) Datum

(c) Dat JAIL WARDER 2017

(d) None of these 26. Answer the first question before you ____
further.
Ans - B
(a) proceeded
23. Many acts of 'homicide' are reported in the (b) proceed
city. (Pick up the word/words that is/are a
synonym of (c) have proceeded
(d) are proceeding
homicide.)
Ans - B
(a) Death 27. Neil Armstrong was first man to walk on moon.

(b) Suicide (Pick out the correct Articles to rectify the sentence
if required)
(c) An attempt to murder (a) the & a

(d) Killing of a person by another (b) an & the


(c) a&an
Ans - D
(d) none of these
24. A person who always believes that good things
Ans - D
will happen. (Give one word substitute for this
sentence) 28. Youth is the time _____ the seeds of character
are sown.
(a) where (b) fat, tall, dwarf, short
(b) in which (c) green, red, yellow, violet
(c) when (d) grain, rain, pain, chain
(d) that Ans - D
Ans - C 34. His health has improved since he ______ India.
29. The rise and fall of the tide _____ due to lunar (a) left
eclipse.
(b) has left
(a) were
(c) had left
(b) was
(d) was leaving
(c) are
Ans - A
(d) is
35. He _____ fights and runs away, lives to fight
Ans - D another day. (Pick out the correct Connector to fill
up the gap)
30. Nature teaches the beasts to know their
friends. (Change the sentence into Passive Voice) (a) that
(a) The beasts are taught by nature to know their (b) when
friends.
(c) who
(b) The beasts are taught to know their friends.
(d) whom
(c) The beasts were taught by nature to know their
Ans - C
friends.
37. The thief jumped _____ the compound wall.
(d) The beasts taught to know their friends by
nature. (a) on
Ans - A (b) over
31. No other story-book is so popular as The (c) above
Arabian Nights. (Change the degree of comparison
without changing its meaning) (d) against

(a) The Arabian Nights is the most popular of all Ans - B


story books. 38. What would I not give to see you happy!
(b) The Arabian Nights are more popular than (Change the give sentence into a statement)
some- other story books. (a) He assured that he would not withhold but
(c) The Arabian Nights is one of the most popular everything to see me happy.
story books. (b) He exclaimed that he would give everything to s
(d) The Arabian Nights are more popular story him happy.
books. (c) He wondered that he would give everything to
Ans -A see me happy.

33. Find out the one of the following series of (d) He declares that he gives everything happy.
Vocabularies" which does not match with the Ans - C
others.
39. He said to me, 'Wait until I come'. (Change the
(a) sweet, sour, salty, bitter sentence to see them into indirect speech)
(a) He asks me to wait until he came. Ans - C
(b) He told me to wait until I came.
83. Ireland was part of the UK, ______?
(c) He ordered me to wait until he came.
(a) Isn't it
(d) He asked me to wait until I had come.
Ans - C (b) wasn't it

40. Find out the correct one of the sentences given (c) hasn't it
below.
(a) He is suffering from a fever since three days. (d) weren't it

(b) He has been suffering from fever for the last Ans - B
three days.
(c) He has suffered from a fever from last three 84. Will he ______ completed his work by
days. tomorrow?

(d) He was suffering from fever for the last three (a) had
days..
(b) have
Ans - B
(c) has
ICDS SUPERVISOR 2016
(d) been
Instructions: Fill up the gaps with appropriate
Verb/Auxiliary Verb/Preposition from the Ans - B
alternatives given below the Question No.81 to 85.
85. The ski resorts are usually crowded. There are
81. I like listening to the radio but I am not always many people _____ skiing.
impressed ______ the quality of the programs.
(a) enjoy
(a) with
(b) are enjoying
(b) at
(c) who enjoy
(c) about
(d) who enjoying
(d) by
Ans - C
Ans - D
86. He enquired, 'When do you intend to pay me? -
82. Take possession of the records immediately so Change the sentence into indirect speech.
that they are not _____ with
(a) He enquired when I intend to pay him.
(a) tempering
(b) He enquired when I intended to pay him.
(b) tamper
(c) He queried if I intend to pay him.
(c) tampered
(d) He queries when I intend to pay him.
(d) tempered
Ans - B
87. Identify the sentence-Kailash Satyarthi was (d) Negative
awarded Nobel Prize for Peace in 2014.
Ans - C
(a) Declaratory

(b) Exclamatory Statistical Field Surveyor 2016

(c) Interrogatory 36. I feel ______ than I did yesterday. (Supply the
correct Comparative to fill the blank)
(d) Satisfactory
(a) bad
Ans - A
(b) worse
88. I always love my country. - Change the
sentence into negative without changing its (c) worst
meaning.
(d) more bad
(a) There is no occasion when I don't love my
country. Ans - C

(b) There are occasions when I don't love my 37. No other girl in the class is as intelligent as
country. Uma. (Identify the Degree of Comparison)

(c) There is no occasion when I love my country. (a) Imperative (c) Positive

(d) No occasion is there when I love my country. (b) Superlative (d) Comparative

Ans - C Ans - C

89. Pick out the correct sentence from the 38. Select the correct question tag for the
following. statement Barking dog seldom bites, _____?

(a) None of the two books is useful. (a) does it

(b) Neither of these two books is useful. (b) doesn't it

(c) None of this two books is useful. (c) is not it

(d) None of the above. (d) won't it

Ans - B Ans - A

90. Identify the degree of comparison in the 39. After driving Professor Kumar to the museum,
sentence Some beans are at least as nutritious as she dropped him at his hotel.
meat.
(Select the correct Passive form from the
(a) Comparative following).

(b) Superlative (a) After being driven to the museum, Professor


Kumar was dropped at his hotel.
(c) Positive
(b) Professor Kumar was being driven and dropped 43. We agreed _____ the proposal _____ an
at his hotel. excursion. (Supply the correct Preposition/s to fill

(c) After she had driven Professor Kumar to the the blank)
museum she had dropped him at his hotel.
(a) at & for (b) to & for
(d) After she was driven Professor Kumar to the
museum she had dropped him at his hotel. (c) with & of (d) on & of
Ans - A
Ans - B
40 . They cried out, "Bravo! it is a capital hit !" (
Select the correct Indirect Speech) 44. The aeroplane flew _______ Kolkata City and
crashed Bay of Bengal.
(a) They cried out that it was a capital hit.
(Supply the The correct Preposition/s to fill the
(b) They exclaimed that it is a capital hit.
(a) on & at
(c) They exclaimed in applause that it was a capital
hit. (b) above & in

(d) They shouted that it Ks a capital hit. (c) over & into

Ans - C (d) about & on

41. Tall man who is walking in lawn is working in Ans - C


University.
45. Neither Rosalin nor her sister _____ present in
(Select the correct Article/s in sequence to rectify the class room yesterday. (Supply the correct
the sentence if required) Auxiliary /ies to the blank)

(a) the, the & a (a) is

(b) a, a & the (b) were

(c) the, the & an (c) have been

(d) not required (d) was

Ans - A Ans - D

42. My sister who is doctor in foreign country has 46. A herd of cattle _______ grazing in our field
writte useful article on dentistry. (Select the since morning (Supply the correct Auxiliary /ies to
correct Article/s sequence to rectify the sentence if fill the blank)
required)
(a) has been
(a) a & a (c) the the & an
(b) are
(b) a, an & the (d) the, an & a
(c) was
Ans - A
(d) have been
Ans - A (a) go & counsel

47. Usually Nirmala _________white shirts, but (b) going & guide
today she ___a green one.
(c) am going & recommend
(Supply the correct Verbs in sequence to fill up the
gaps) (d) shall be going & advocate

(a) wear & is wearing Ans - C

(b) wears & is wearing Junior Clerk 2015

(c) wearing & wom 1. Why should he be suspected for the theft?
(Choose its appropriate active voice)
(d) wearing & has worn A. Why should they suspected him for the theft?

Ans - B B. Why did we suspect him for the theft?


C. Why should we suspect him for the theft?
48. I ________ everybody, but nobody admitted
______ my books. (Supply the correct Verbs to fill D. Why shall we suspect him for the theft?
the gaps)
Ans - C
(a) asked & saw 2. He was arrested on change of theft but released
for want of evidence. (Choose its appropriate
(b) ask & seeing active voice)
A. The police arrested him on charge of theft but
(c) asked & having seen
released for want of evidence.
(d) ask & had seen B. The police arrested him on charged of theft
released for want of evidence.
Ans - C
C. The police arrested him on charged of theft
49. She stood there with tears ______ down her released for want of evidence.
checks. D. The police was arrested him on charged of theft
but released for want of evidence.
(Supply the correct Verb to fill the blank)
Ans - A
(a) rolling 3. Mahabaleshwar is cooler than Mysore. (Select its
appropriated positive degree keeping the meaning
(b) drop unchanged)

(c) forming A. Mysore is as cool as Mahabaleswar.


B. Mysore is not so as Mahabaleswar
(d) shed
C. Mahabaleswar is more cool than Mysore.
Ans - A
D. Mahabaleswar is more coller than Mysore.
50. I ______ to Cuttack tomorrow to see the doctor Ans - D
you
4. Abdul is as strong as his brother. (Select its
appropriate positive degree keeping the meaning
(Supply the correct Verbs to fill the gaps)
unchanged)
A. His brother is not so stronger than Abdul. D. Might
B. His brother is not so strong as Abdul. Ans - C
C. Abdul is stronger than his brother. 9. They erected signposts in order that the road
_____be know. (Select the right modal fill the
D. Abdul is not so strong as his brother.
blank)
Ans - A

A. Will
5. Helen of Troy was more beautiful than any other
B. Would
woman. ( Select its appropriate superlative degree
keeping the meaning unchanged) C. Can
A. Helen of Troy was the beauty fullest woman of D. May
all.
Ans - B
B. Helen of Troy was the most beautiful of all
10. He finished first though he ______late. (Select
women.
the right verb to fill the blank)
C. Some women were more beautiful than Helen of
A. Began
Troy.
B. Has began
D. Some women are as beautiful as Helen of Troy.
C. Had began
Ans - B
D. Was began
6. They sold the house because of it ______ very
old. (Select the right auxiliary to fill the black) Ans - A
A. Had been 11. Whenever we _____, we talk of old times. (
Select the right verb to fill the blank)
B. Was
A. Met
C. Is
B. Have met
D. Were
C. Meet
Ans - C
D. Were meeting
7. He speaks as though he ______very angry.
(Select the right auxiliary to fill the blank) Ans - C
A. Had been 12. The committee ______ divided on one major
point. (Select the correct auxiliary in agreement
B. Was
with the subject)
C. Has been
A. Have
D. Is
B. Are
Ans - D
C. Had
8. The notice was published in order that all
D. Was
______know the facts.(Select the right modal to fill
the blank) Ans - D
A. May 13. He had a cow that_______ enormous
quantities of milk. (Select the correct verb in
B. Might have know
agreement with the subject)
C. Should
A. Was giving 18. The way was long, the wind was cold, the
minstrel was infirm and old. (Select the underlined
B. Gave
section which is not a noun pronoun)
C. Had given
A)
D. Is giving
B)
Ans - A
C)
14. The poor are deprived off two square meals a
D)
day. (Select the right preposition/s to rectify the
sentence if required) Ans - A
A. From 19. Find out one of the four pairs given below of
noun in its singular and plural from which is
B. Of
incorrect.
C. For
Sl.No. Noun(Singular) Noun(Plural)
D. Not required
A wife wives
Ans - B
B leaf leaves
15. It is six o’clock with my watch. (Select the right
C wolf wolves
preposition/s to replace the wrong one if any)
D handkerchief handkerchieves
A. By
Ans - D
B. On
20. Pick out the incorrect one of following pairs of
C. In
Nouns in their masculine and feminine forms.
D. At
Sl. No. Noun (masculine) Noun(feminine)
Ans - A
A prince princess
16. She is good in mathematics. (Select the right
B widow widower
preposition/s replace the wrong one if any)
C conductor conductress
A. On
D bachelor bachelors
B. In
Ans - B
C. With
21. Find out which of the following phrases is
D. At
incorrect?
Ans - D
A. A child’s voice
17. Have you seen M. F Hussain, the artist’s
B. Women’s clothes
sprinting? (Select the underlined part which is not
a noun or pronoun) C. The girl’s hostel
A) D. My brother-in-law’s car
Ans - B
B) 22. Pick out which of the following series does not
match with the other?
C)
A. Length, wisdom, bravery, cruelty and darkness
D)
Ans - B
B. Laughter, belief, choice, concealment and Ans - C
success
27. We reached the railway station. The train had
C. Humble, bitter humane, vacant and prudent left. (Select the appropriate connector to join the
two )
D. Infant, agent, rogue, pirate and priest
A. As soon as
Ans - A
B. Where
23. Time we live ought not to be computed by
number of years we live but by use that has been C. After
made of it. (Select the correct article/s to rectify
D. When
the sentence if required)
Ans - C
A. The, a and an
29. Is that story true? Is that story false? (Select the
B. The and a
co-ordinating connector to join the two questions)
C. The, the and the
A. And
D. Not required
B. Either, or
Ans - D
C. Neither . . . nor
24. Tiger, animal equal to lion in size, is native of
D. Or
Asia. (Select the correct article/s to rectify the
sentence if required) Ans - D
A. The & an 30. These fishing nets are all the wealth I own.
(Select its correct negative from keeping the
B. The, an and a
meaning unchanged)
C. The and a
A. I own no wealth other than these fishing nets.
D. None of these
B. These fishing nets are not all the wealth I own.
Ans - A
C. All the wealth I own are not these fishing nets.
25. Select the correct one of the following
D. I own not these fishing nets but all the wealth.
sentence without any mistake in the use of
article/s. Ans - A
A. I met an European in my school. 31. Who can touch the pitch without being defiled?
(Select its correct negative form keeping the
B. I met a European in my school.
meaning unchanged)
C. I met with European in my school.
A. He asked if anyone could touch the pitch
D. I met European in my school. without being defiled.
Ans - B B. Who can touch the pitch and not be defiled?
26. ______ I am dead, my dearest sing no sad C. Someone questioned if anyone touching the
songs for me. (Select the appropriate connector to pitch wouldn’t be defiled.
join the connector fill the blank)
D. Can touch the pitch and not be defiled?
A. As soon as
Ans - B
B. Where
32. Is that the way a gentleman should behave?
C. If (Change it into a statement keeping the meaning
unchanged)
D. When
A. He didn’t approve that a gentleman like him 36. We are not allowed to park our vehicles in front
should behave in that way. of the temple. (Select its correct Tag question
form)
B. He queried if you gentleman should behave in
that way. A) We are not allowed to park our vehicles in front
of the temple, aren’t we?
C. Someone asked if a gentleman shall behave in
that way. B) We are not allowed to park our vehicles in front
of the temple, are we?
D. Someone questioned if a gentleman shall
behave in that way. C) They do not allow us to park our vehicles in front
of the temple, did they?
Ans - C
D) We are not allowed to park our vehicles in front
33. Ah, what a wonderful sight was their! (Change
of the temple, don’t we?
it into a statement keeping the meaning
unchanged) Ans - B
A. What a wonderful sight was their! He exclaims. 37. You all have done very badly!’ remarks the
teacher.(Select its correct indirect speech)
B. He could not but wonder at the beautiful sight
there. A) The teacher remark that we all had done very
badly.
C. They exclaimed it is a wonderful sight.
B) The teacher remarks that we all have done very
D. He exclaims this is a wonderful sight.
badly.
Ans - C
C) The teacher remarks that they had all done very
34. The Government of India has conferred the badly.
International tag on the Biju Pattnaik Airport.
D) The teacher remarked that they all have been
(Select its correct interrogative form)
doing very badly.
Ans - C
A) Who has conferred the International tag on the
38. He urged them to be quiet and listen to his
Biju Pattnaik Airport?
words. (Select its correct direct speech)
B) What has conferred to Biju Pattnaik Airport?
A) He says, “Be quiet and listen to my words.”
C) What have been confer the International tag by
B) ‘Be quiet and listen to my words’, says he.
the Government of India?
C) He exhorted, “Be quiet and listen to my words.”
D) What had been done by the Government of
India to Biju Pattnaik Airport? D) “Be quiet and listened to my words,’ he said.
Ans - D Ans - C
35. All the guests have finished their lunch. (Pick 39. Alas! We will hear his voice no more. (Choose
out which of its following Yes/No type its appropriate passive voice)
interrogatives is correct)
A) Alas! We will not hear his voice anymore.
A) All the guests had finish their lunch?
B) It is grieved that his voice will be heard no more.
B) Had all the guests finished their launch?
C) Alas! His voice would be heard no more.
C) Have all the guests finished their lunch?
D) We grieved that we would not hear his voice
D) Had not all the guest finish their lunch? more.
Ans - C Ans - B
40. Shall I ever forget those happy days? (Choose Ans - B
its appropriate passive voice)
Q (5) The teacher said to the boy, "Leave the room
A) Those happy days would never be ever at once" (Change into Indirect Speech)
forgotten by me.
A. The teacher forced the boy to leave the room at
B) Those happy days can never be ever forgotten once.
by me.
B. The teacher requested the boy to leave the
C) Shall those happy days be ever forgotten by me? room at once.
D) Should those happy days be ever forgotten by C. The teacher ordered the boy to leave the room
me? at once.
Ans - C D. The teacher advised the boy to leave the room
at once.
Ans - C
Jail Warder 2022
Q (6) If the bus to the airport hadn't been so late,
Q (1) I saw him _________ the road. we _________ the plane.
A. to cross A. would have caught
B. cross B. caught
C. crossed C. would catch
D. has crossed D. had caught
Ans - B Ans - A
Q (2) They expected ________ by air. Q (7) She is always giving herself airs.
A. travel The underlined idiom means __________
B. to travel A. trying to impress people with her superiority.
C. travelling B. acting in a proud way.
D. travelled C. showing an affected manner intended to make
Ans - B her appear educated, elegant, etc.

Q (3) I would like you ________ my uncle. D. all the above

A. meet Ans - D

B. to meet Q (8) He fall from the tree, but fortunately he was


_______.
C. met
A. mishurt
D. meeting
B. inhurt
Ans - B
C. unhurt
Q (4) What was the notice ________?
D. dishurt
A. at that you were looking
Ans - C
B. you were looking at
Q (9) The old man has no _________ to lift the box.
C. you were looking at it
A. strong
D. which you were looking
B. strongly D. will buy
C. strengthen Ans - A
D. strength Q (5) Paul said, "The man did not go earlier."
(Change into Indirect Speech)
Ans - D
A. Paul said that the man had not gone before.
Q (10) Select the antonym of
B. Paul said that the man had not gone earlier.
malignant
C. Paul said that the man had to go before.
A. mortal
D. Paul said that the man has to go before.
B. internecine
Ans - B
C. benign
Q (6) I haven't got a ticket. If _______ one, I could
D. lethal
get in.
Ans - C
A. I would have
Q (1) They _______ Indian food.
B. I had
A. have been loving
C. I have
B. are loving
D. I've got
C. love
Ans - B
D. were loving
Q (7) I suggested a game to break for ice. The
Ans - C underlined phrase means __________

Q (2) The lion ________ in the Gir forest of Gujrat. A. make people happy

A. finds B. overcome initial shyness

B. is found C. begin something new

C. has found D. separate into pieces.

D. has been finding Ans - B

Ans - B Q (8) Choose the correct antonym of

Q (3) State the grammatically not acceptable wisdom : __________


sentence.
A. ignorance
A. I live in Sambalpur.
B. knowledge
B. His English is getting better all the time.
C. labour
C. Look ! Two swans carry a tortoise.
D. apprehension
D. I usually go to school on foot.
Ans - A
Ans - C
Q (9) His illness has made him completely
Q (4) She ________ any new clothes for years. ________.
A. has not bought A. unmobile

B. not bought B. immobile

C. isn't have bought C. inmobile


D. dismobile C. The officer commanded his men to halt.
Ans - B D. The officer shouted to his men that they will
halt.
Q (10) A part of the building was ruined in the
_________ Ans - C
A. explode Q (5) I was very cold, ________.
B. explosion A. as I was wearing my coat.
C. exploding B. though I was wearing my coat.
D. explosive C. and I was wearing my coat.
Ans - B D. but I was wearing my coat.
Q (1) I choose the corner table, seated myself and Ans - B
started skimming through the newspaper.
Q (6) A friend of mine phoned __________ me to a
The above sentence is a _________ party.
A. Simple Sentence A. for invite
B. Compound Sentence B. for inviting
C. Complex Sentence C. to invite
D. Wrong Sentence D. for to invite
Ans - B Ans - C
Q (2) She would not let me _________ the letter. Q (7) He stuck too his resolution.
A. read Here the underlined word means _______
B. reading A. solution of a problem
C. to read B. firm intention
D. have read C. solution
Ans - A D. determination.
Q (3) I am young and strong, ________ I ? Ans - B
A. amn't Q (8) Choose the correct antonym of
B. haven't justice : _______
C. aren't A. faint
D. isn't B. depression
Ans - C C. unfairness
Q (4) "Halt !" shouted the officer to his men. D. captivity
(Change into Indirect Speech)
Ans - C
A. The officer shouted to his men that they should
Q (9) The government intend to bring in a new
halt.
________ regarding the sale of alcohol.
B. The officer shouted to his men that they shall
A. legislate
halt.
B. legislative
C. legislature Ans - A
D. legislation Q (5) My teacher said to me, "Never keep bad
company" (Change into Indirect Speech)
Ans - D
A. My teacher asked me not to keep bad company
Q (10) If I ______ you, I would buy this mobile
phone. B. My teacher told me not to keep a bad company
A. was C. My teacher ordered me not to keep a bad
company
B. had been
D. My teacher advised me never to keep bad
C. were
company.
D. am
Ans - D
Ans - C
Q (6) The librarian asked us __________ so much
Q (1) Choose the grammatically correct sentence. noise.

A. Did you not finish yet ? A. don't make

B. Have not you yet finished ? B. not make

C. Haven't you finished yet ? C. not making

D. Haven't yet you finished ? D. not to make

Ans - C Ans - D

Q (2) What ________ at 7 o' clock yesterday Q (7) He called me names. The underlined idiom
evening ? means __________

A. were you doing A. He used insulting words about me.

B. have you been doing B. He gave my name involved in something wrong.

C. have been you doing C. He said my name loudly.

D. have you done D. He called me by my name.

Ans - A Ans - A

Q (3) Gopal and his sister ________ come. Q (8) Someone kacking honesty is _________

A. has A. inhonest

B. have B. dishonest

C. are C. mishonest

D. is D. ilhonest

Ans - B Ans - B

Q (4) The jury _______ selected its chairman. Q (9) He started the work after a lot of ________

A. has A. hesitate
B. have B. hesitation

C. is C. hesitant

D. are D. hesitantly
Ans - B Ans - A
Q (10) Select the antonym of Q (5) He says, "The girl is reading" (Change into
Indirect Speech)
Shallow
A. He said that the girl was reading
A. Narrow
B. He says that the girl is reading
B. wide
C. He said that the girl is reading
C. deep
D. He says that the girl was reading
D. low
Ans - B
Ans - C
Q (6) The headmaster gave a certificate to the girl
Q (1) Do ________ die.
_________ had saved the life of a child.
A. and
A. who
B. but
B. which
C. or
C. whom
D. for
D. when
Ans - C
Ans - A
Q (2) We _________ (withdrew from) the match
Q (7) I wish I ________ rich and had a lot of money.
after a quarrel.
A. am
A. dropped out of
B. are
B. kept out of
C. were
C. packed up
D. shall be
D. went back
Ans - C
Ans - A
Q (8) When I was a child, I _______ the flute.
Q (3) The teacher made him stand up.
A. I was playing
The Passive form of the above sentence will be
_________ B. I'm playing
A. The teacher was made stand up. C. I play
B. The teacher has made him stand up. D. I played
C. He has made the teacher to stand up. Ans - D
D. He was made to stand up. Q (9) Choose the correct antonym of
Ans - D captivity : ________
Q (4) English ________ all over the world. A. confinement
A. is spoken B. freedom
B. spoken C. restriction
C. has spoken D. temporary
D. has been speaking Ans - B
Q (10) In the following question, out of the four C. He asked whether I was going home with him.
alternatives, select the alternative which is
D. He asked whether I am going with him.
the best substitute of the phrase.
Ans - C
wide in range or effect
Q (5) Helen ___________ and left.
A. sweeping
A. said goodbye to me
B. exaggerate
B. said me goodbye
C. exclusive
C. told me goodbye
D. superficial
D. bid me goodbye
Ans - A
Ans - A
Q (1) Now you can live in that room. We ________
Q (6) I wish I ___________ have to work tomorrow,
the roof.
but unfortunately I do.
A. repaired
A. don't
B. have been repairing
B. didn't
C. have repaired
C. wouldn't
D. were repairing
D. won't
Ans - C
Ans - B
Q (2) Can you ever forget _______ that important
Q (7) After not speaking to each other for years,
letter without any postage on it?
the two brothers decided to bury the hatchet.
A. post
The underlined idiom means __________
B. posted
A. put an end to a quarrel.
C. to post
B. become reconciled
D. posting
C. end a pair of being enemies or of mutual dislike
Ans - D and become friendly.
Q (3) There ________ a whole regiment of people D. all the above.
here to see you.
Ans - D
A. is
Q (8) Choose the correct antonym of
B. are
criticism : ________
C. were
A. protection
D. have been
B. commendation
Ans - A
C. praise
Q (4) "Are you coming home with me?" he said to
D. support
me. (Change into Indirect Speech)
Ans - C
A. He told me whether you are going home with
him. Q (9) Someone lacking a sense of responsibility is
_______
B. He told me whether you are going home with
me. A. unresponsible
B. irresponsible B. He was deeply grieved and said that what can he
do for her.
C. inresponsible
C. He was deeply grieved and asked his wife what
D. disresponsible
he could do for her.
Ans - B
D. He grieved deeply and told his wife what could
Q (10) He was given a ________ driving licence. he do to her.

A. provide Ans - C

B. provision Q (5) It is not my favourite job but I like


__________ kitchen as often as possible.
C. provisional
A. cleaning
D. provisionally
B. clean
Ans - C
C. to clean
Q (1) Mina : Would you like to come to watch a
movie this weekend ? D. that I clean

Nina : I'd like to but I'm afraid ______. Ans - C

A. I don't have time Q (6) A plague of locusts would have left more than
this.
B. I shall not have time
The underlined word means __________
C. I won't have time
A. a very grave infections disease
D. I haven't time
B. a serious nuisance
Ans - C
C. a large number of insects that come to an area
Q (2) I hope everything _________ well for you. at one time
A. go D. an infections disease spread up rats.
B. shall go Ans - B
C. will go Q (7) Choose the correct antonym of
D. going vital : _________
Ans - C A. previous
Q (3) It's I who ________ your help. B. pride
A. need C. affluence
B. needs D. unimportant
C. is needing Ans - D
D. needed Q (8) Not dependent on another is ________
Ans - A A. undependent
Q (4) He was deeply grieved and said to his wife, B. imdependent
"What can I do for you ?" (Change into
C. independent
Indirect Speech)
D. misindependent
A. He was deeply grieved and asked his wife what
he can do for her. Ans - C
Q (9) It is ________ to send cards at the New Year B. She told me who went to Madras.
in our country.
C. She asked me who had gone to Madras.
A. custom
D. She told me who has gone to Madras.
B. customary
Ans - C
C. customer
Q (5) I'm looking forward _________ on a picnic.
D. customarily
A. going
Ans - B
B. to go
Q (10) I believe that I ______ got no right.
C. to going
A. have
D. that I go.
B. has
Ans - C
C. did
Q (6) She said that if she ________ the time, she
D. am would have come.
Ans - A A. was
Q (1) He _______ early everyday. B. had
A. gets up C. have
B. got up D. has
C. is getting up Ans - B
D. has got up Q (7) The rich man looked down on his poor
neighbours. The underlined phrase means
Ans - A
__________
Q (2) I ______ badminton with her. She always
A. regarded with contempt
won.
B. had a strong worm feeling for
A. often play
C. was very helpful to
B. will often play
D. took care of
C. am often playing
Ans - A
D. often played
Q (8) Choose the correct antonym of
Ans - D
populous : __________
Q (3) My hands are dirty, because ________
A. praise
A. I am gardening.
B. ignoble
B. I have been gardening.
C. isolated
C. I have gardened.
D. inessential
D. I was gardening.
Ans - C
Ans - B
Q (9) He doesn't know reading or writing. He is
Q (4) She said to me, "Who has gone to Madras ?"
_______.
(Change into Indirect Speech)
A. misliterate
A. She asked me who went to Madras.
B. illiterate B. arrived
C. inliterate C. have arrived
D. disliterate D. was
Ans - B Ans - B
Q (10) The ________ of wheat has increased this Q (7) She kept thinking, “How lucky Serena
year. ______!”
A. production A. can
B. productivity B. could be
C. producing C. is
D. produced D. supposedly
Ans - A Ans - C
Q (1) Do you usually go this way ? Q (12) Her grandmother ________ ninety next
week.
The above sentence is a _________
A. is
A. Simple Sentence
B. will
B. Compound Sentence
C. will be
C. Complex Sentence
D. has been
D. Wrong Sentence
Ans - C
Ans - A
Q (13) How long _______ tennis ?
Q (2) Anita ______ her work by the time he came.
A. did you play
A. had done
B. was you playing
B. has done
C. are you playing
C. is done
D. had you been playing
D. would done
Ans - D
Ans - A
Q (14) Choose the correct antonym of
Q (4) May that hope never be betrayed.
loyalty : _______
The underlined word here means __________
A. betrayal
A. to fail to justify
B. protection
B. to give evidence of
C. important
C. to reveal accidentally
D. arrogance
D. to reveal treacherously
Ans - A
Ans - A
Q (15) He is always _________ to elderly people.
Q (5) When I _______ at school, I remembered it
was Sunday. A. consider
A. arrive B. considerable
C. considerate D. has been
D. consideration Ans - C
Ans - C Q (5) The teacher said to me, "Have you done your
homework?"
Q (18) He said, "How cruel of him !" (Change into
Indirect Speech) The indirect speech of the above statement is
______
A. He remarked that it was very cruel of him.
A. The teacher asked me if I have done my
B. He told that it has been very cruel of him.
homework.
C. He lamented that it is very cruel of him.
B. The teacher asked me if you had done your
D. He said that it was cruel of him. homework.

Ans - A C. The teacher asked me whether I had done my


homework.
Q (1) State the grammatically correct sentence.
D. The teacher asked me whether I did my
A. What fine the weather is ! homework.
B. What an intelligent girl she is ! Ans - C
C. What fluently he speaks ! Q (6) Radha was made _________ at the party.
D. How a sharp memory she has got ! A. dance
Ans - B B. to dance
Q (2) Most children ________ (resemble) their C. dancing
parents.
D. danced
A. keep on
Ans - B
B. take after
Q (7) Choose the correct antonym of
C. go through
deadly : _______
D. make off
A. harmless
Ans - B
B. trivial
Q (3) He promised to help me, but later he
_______ (failed to keep) his promise. C. clean

A. dropped out of D. order

B. fell through Ans - A

C. did away with Q (8) I _________ you because you have cheated
me again.
D. went back
A. unbelieve
Ans - D
B. misbelieve
Q (4) The audience _________ requested to leave
by the nearest exit. C. disbelieve
A. is D. imbelieve

B. was Ans - C

C. are
Q (9) His foot prints were clearly ________ in the A. do
heavy dust.
B. to do
A. evidence
C. doing
B. evident
D. that I do
C. evidently
Ans - C
D. evidencing
Q (10) I could not sleep _________ very tired.
Ans - B
A. although I was
Q (10) These oils may cause ________ to your
B. despite I was
skins.
C. despite of being
A. irritate
D. in spite of being
B. irriation
Ans - A
C. irritatingly
Q (12) His failure has completely discouraged him
D. irritant
________ trying again.
Ans - B
A. to
Q (3) A crowd of people ________ shouting
B. from
slogans.
C. on
A. has
D. at
B. have
Ans - B
C. was
Q (13) There was ________ change in his life after
D. were
his father's death.
Ans - C
A. drama
Q (4) His clothing _________ very attractive.
B. dramatize
A. is
C. dramatical
B. are
D. dramatic
C. has
Ans - D
D. have
Q (14) What a difficult situation he is __________ !
Ans - A
A. in
Q (6) Choose the correct antonym of
B. on
crooked : _______
C. with
A. composed
D. at
B. surprised
Ans - A
C. honest
Q (15) Choose the synonym of
D. hostile
Infallible
Ans - C
A. Out stoked
Q (9) You can't stop me ________ what I want.
B. weak Ans - A
C. authentic Q (4) Do you want _________ with you or do you
want to go alone ?
D. unerring
A. me coming
Ans - D
B. me to come
Q (19) The woman said to her husband, “I met the
boy on the road today." C. that I come
The indirect speech of the above statement is D. that I will come
______
Ans - B
A. The woman told her husband that she met the
Q (5) The weather is horrible. I wish it __________
boy on the road today.
raining.
B. The woman told to her husband that she had
A. would stop
met the boy on the road that day.
B. stopped
C. The woman informed her husband that she had
met the boy on the road that day. C. stops
D. The woman informed her husband thet she met D. will stop
the boy on the road that day.
Ans - A
Ans - C
Q (6) I can remember _______ voices in the middle
Q (1) Which one of the following sentences is a of the night.
"Imperative Sentences" ?
A. hear
A. Beware of pickpockets.
B. heard
B. Cuttack is on the Mahanadi.
C. hearing
C. Anil teaches English grammar.
D. to hear
D. He is going to sell his car after Holi.
Ans - C
Ans - A
Q (7) The news of his death was a bolt from the
Q (2) The government ________ (rejected) the blue. The underlined phrase means _________
employees' request for pay revision.
A. a state of great shock
A. called off
B. fast unexpected event
B. cut down
C. lightning flash with noise of thunder
C. turned down
D. sad news
D. gone through
Ans - B
Ans - C
Q (8) Reading other people's letters is not playing
Q (3) We _________ (withdrew from) the match the game.
after a quarrel.
The underlined idiom means __________
A. dropped out of
A. performing for pleasure or amusement.
B. kept out of
B. playing according to the rules.
C. packed up
C. being honest and fair.
D. went back
D. doing something with little interest. D. come out with
Ans - C Ans - C
Q (9) Choose the correct antonym of Q (4) The child walked fast to ________ his
parents.
gather : _________
A. come out with
A. porous
B. catch up with
B. raise
C. look forward to
C. disperse
D. get on with
D. order
Ans - B
Ans - C
Q (5) She said, "My God ! I am ruined." (Change
Q (10) He lost his _______ in the accident.
into Indirect Speech)
A. memorable
A. She exclaimed sadly that she was ruined
B. memorial
B. She lamented that she is ruined
C. memoralize
C. She said that she is ruined.
D. memory
D. She told that she was ruined.
Ans - D
Ans - A
Q (1) Through he was born of poor parents, he rose
Q (6) The teacher was very pleased ________ their
to greatness.
work.
Pick out the "Main Clause" from the above
A. on
sentence.
B. with
A. Though he was born of poor parents
C. in
B. he rose to greatness
D. to
C. he was born of poor parents
Ans - B
D. was born of poor parents, he rose
Q (7) If _________ my ticket, I'll be in trouble.
Ans - B
A. I'll lose
Q (2) He is very funny. He makes me _________.
B. I lost
A. laughing
C. I would lose
B. laugh
D. I lose
C. to laugh
Ans - D
D. laughed
Q (8) Choose the correct antonym of
Ans - B
rival : _________
Q (3) Who can _______ such mischief ?
A. associate
A. catch up
B. helpful
B. run out of
C. fair
C. put up with
D. courteous
Ans - A A. is sleeping
Q (9) Find a job and end your ________ on your B. was slept
parents.
C. sleeps
A. depend
D. was being slept
B. dependence
Ans - C
C. dependent
Q (6) Mother said, "My son is returning home this
D. dependently evening."
Ans - B The indirect speech of the above statement is
______
Q (10) The big tree was ______ the lake.
A. Mother said that her son is returning home this
A. in
evening.
B. within
B. Mother said that her son will return home this
C. across evening.

D. off C. Mother said that her son was returning home


that evening.
Ans - C
D. Mother said that her son has been returning
Q (1) Your eyes are red. You ________. home that evening.
A. cried Ans - C
B. have cried Q (7) Please tell me what _________
C. were crying A. is your name ?
D. have been crying B. is your name.
Ans - D C. your name is.
Q (2) I did not notice you _______ your hand. D. your name is ?
A. raise Ans - C
B. raised Q (8) Choose the correct antonym of
C. raising triumph : _________
D. has raised A. success
Ans - A B. disaster
Q (3) This is the books I have been looking C. incredible
________.
D. fresh
A. about
Ans - B
B. off
Q (9) That which is contrary to law is ________
C. for
A. illegal
D. on
B. unlegal
Ans - C
C. inleged
Q (5) This baby _________ ten hours a day.
D. dislegal
Ans - A Ans - B
Q (10) Mohan's uncle has presented him with a Q (5) Ashok _________ ill for three weeks. He is
________ suit. still in hospital.
A. swim A. had been
B. swimming B. has been
C. swiming C. is
D. swam D. was.
Ans - A Ans - B
Q (1) Look at the clouds. It ________. Q (6) Are you in the black ? The underlined phrase
means _________
A. will rain
A. have money in your bank account
B. is to rain
B. wearing black clothes
C. is going to rain
C. owing more money than you have
D. will be rain
D. in debt
Ans - C
Ans - A
Q (2) A number of children _________ suffering
from cold. Q (7) He told us an incredible story about his
grandmother catching a thief.
A. is
The synonym of the underlined word is
B. are
___________
C. has
A. probable
D. have
B. amazing
Ans - B
C. acceptable
Q (3) 'We may not win the case in the court', the
D. plausible
lawyer said to his client, '_______, we can
Ans - B
appeal to the supreme court.'
Q (8) It rained ________ for an hour.
A. therefore
A. continue
B. however
B. continuous
C. for
C. continuation
D. none of the above.
D. continuously.
Ans - B
Ans - D
Q (4) "Be quiet, "I said to them. (Change into
Indirect Speech) Q (19) Arrange the words for meaningful
sentences.
A. I asked them that they should keep quiet
phones/not/used/must/mobile/be/driving /while
B. I told them to be quiet.
A. While not mobile phone we must drive
C. I warned them to be silent.
B. Mobile phones must not be used while driving
D. I advised them to maintain silence.
C. While driving, mobile phones must not be used. C. liking
D. While using mobile phones, we must not drive D. likely
Ans - C Ans - A
Q (20) Choose the meaning of the following Q (5) The youth ______ more serious than his
uncle.
A stitch in Time saves nine
A. was
A. One time more can be saved
B. were
B. Stitch can save live
C. has
C. Do your work within time period
D. have
D. Check and rectify small errors
Ans - A
Ans - D
Q (6) "Who lives next door ?" He said.
Q (1) What an excellent singer she is!
The indirect speech of the above statement is
The above sentence is ________
______
A. a Declarative Sentence.
A. He said who lives next door.
B. an Imperative Sentence.
B. He asked who lives next door.
C. an Interrogative Sentence.
C. He asked who lived next door.
D. an Exclamatory Sentence.
D. He said who lived next door.
Ans - D
Ans - C
Q (2) I can hardly _________ (understand) what
Q (7) Something too large or great to be measured
you say.
is ________.
A. bring up
A. unmeasurable
B. call off
B. inmeasurable
C. take after
C. ilmeasurable
D. make out
D. immeasurable
Ans - D
Ans - D
Q (3) India has _______ (stopped having) the caste
Q (8) Choose the correct antonym of
system.
harm : _______
A. gone back to
A. detriment
B. done away with
B. welfare
C. fallen through
C. unfit
D. dropped out of
D. continue
Ans - B
Ans - B
Q (4) Ram as well as his friends ________ to play
football. Q (9) My teeth are ________ to hot or cold food.
A. likes A. sensible
B. like B. sensitive
C. sensual C. has
D. sensuous D. have
Ans - B Ans - B
Q (10) This book is full of _______ interesting Q (5) " I must go soon", said Sarita.
characters.
The indirect speech of the above statement is
A. belief ______
B. believe A. Sarita said she has to go soon.
C. believable B. Sarita said that she must have gone soon.
D. believing C. Sarita said she would have gone soon.
Ans - C D. Sarita said that she would have to go soon.
Q (1) He must not be late; _______ he will be Ans - D
punished.
Q (6) Sarita ________ to phone me last night, but
A. and she didn't.
B. but A. supposed
C. yet B. is supposed
D. otherwise C. was supposed
Ans - D D. did suppose
Q (2) Every month I lay _______ a few rupees for Ans - C
my child's education.
Q (7) The ladder gave way and he fell to the
A. out ground.
B. in The underlined phrase means __________
C. by A. obstructed
D. down B. broke
Ans - D C. travelled in the same direction.
Q (3) We are all eagerly _______ our sister's D. did what was wanted.
wedding.
Ans - B
A. putting up with
Q (8) Choose the correct antonym of
B. catching up
priceless : ________
C. running out for
A. cheap
D. looking forward to
B. unfair
Ans - D
C. ignoble
Q (4) Neither the old man nor his sons _________
D. generous
prepared to sell their old house.
Ans - A
A. is
Q (9) A car that only uses a small amount of petrol
B. are
is _________.
A. Economics Choose the best option from the given alternatives
B. Economical which can be substituted for the

C. Economist given word/sentence.

D. Economy A person who does not want to see realities of life


and tries to escape
Ans - B
Q (10) You will ________ if you work hard. (A) Escapist

A. success (B) Decanter

B. successful (C) Curator


C. succeed (D) Arsenal
D. successfully Ans - A
Ans - C
Question No.4

Choose the best option from the given alternatives


Sub Inspector Excise 2022 which can be substituted for the

Question No.1 given word/sentence.

Fill in the blanks with suitable Article from the Code of diplomatic etiquette and precedence
given alternatives.
(A) Wardrobe
Trent Boult has taken _____________ cricketing
(B) Protocol
world by storm.
(C) Sheath
(A) a
(D) Secular
(B) an
Ans - B
(C) the
Question No.5
(D) No Article
Replace the underlined phrase grammatically and
Ans - C
conceptually with the help of the given
Question No.2
options. If the given sentence is correct then select
Fill in the blanks with suitable Article from the the option 'The given sentence is
given alternatives.
correct'.
The car is made of ____________ steel.
“According to the latest data on a” Reserve Bank of
(A) a India, overall value of transactions

(B) an contracted by 46 per cent in April over March.

(C) the (A) According to the latest data from the

(D) No Article (B) According to a latest data for the

Ans - D (C) According to an latest data from the

Question No.3 (D) The given sentence is correct


Ans - A I don’t know “when I will be able to visit India” or
she can come here.
Question No.11
(A) The given sentence is correct
Fill in the blanks with suitable Preposition from the
given alternatives. (B) when I will able for visit India

Our college is ______________ the hospital and (C) when I will able to visit India
the temple.
(D) when I will be able for visit India
(A) underneath
Ans - A
(B) across
Question No.15
(C) among
Choose the word which expresses nearly the
(D) between opposite meaning of the given word "

Ans - D TRANSCEND ".

Question No.12 (A) Excel

Find the word which is correctly spelt from the (B) Overstep
given options.
(C) Surrender
(A) Underwritte
(D) Best
(B) Santuary
Ans - C
(C) Temporal
Question No.16
(D) Trauema
Find the word which is correctly spelt from the
Ans - C given options.

Question No.13 (A) Documant

Find the word which is correctly spelt from the (B) Emanicipate
given options.
(C) Imaterial
(A) Reprimend
(D) Fragile
(B) Infalible
Ans - D
(C) Qualified
Question No.17
(D) Inadible
Fill in the blanks with suitable Preposition from the
Ans - C given alternatives.

Question No.14 Jack and I arrived ________ the same time.

Replace the underlined phrase grammatically and (A) on


conceptually with the help of the given
(B) at
options. If the given sentence is correct then select
(C) before
the option 'The given sentence is
(D) in
correct'.
Ans - B (D) The given sentence is correct

Question No.18 Ans - D

Choose the word which expresses nearly the Question No.21


opposite meaning of the given word "
Choose the best option from the given alternatives
TURMOIL ". which can be substituted for the

(A) Calm given word/sentence.

(B) Bustle Fit to be chosen

(C) Disorder (A) Invincible

(D) Disturbance (B) Legible

Ans - A (C) Eligible

Question No.19 (D) Trackable

Fill in the blanks with suitable Preposition from the Ans - C


given alternatives.
Question No.22
The essay she wrote was full _______ mistakes.
Replace the underlined phrase grammatically and
(A) beyond conceptually with the help of the given

(B) of options. If the given sentence is correct then select


the option 'The given sentence is
(C) for
correct'.
(D) by
Payment transactions “fell massively about the
Ans - B
country in” the month of April.
Question No.20
(A) fell massively on the country in
Replace the underlined phrase grammatically and
(B) fell massively across the country in
conceptually with the help of the given
(C) The given sentence is correct
options. If the given sentence is correct then select
the option 'The given sentence is (D) fell massively across the country on

correct'. Ans - B

Although remote coaching “might be the mantra Question No.23


right now, athletes realise” that it cannot
Choose the word which best expresses the similar
be a long-term solution. meaning of the given word " TRIVIAL

(A) might be an mantra right now, athletes realise ".

(B) might being an mantra right now, athletes (A) Important


realise
(B) Large
(C) might being the mantra right now, athletes
(C) Small
realise
(D) Avoid Find the word which is correctly spelt from the

Ans - C given options.

Question No.24 (A) Larcany


(B) Mandatery
Choose the word which best expresses the similar
meaning of the given word " ERUDITE (C) Pacify
(D) Insurgant
".
Ans - C
(A) Common
Question No.8
(B) Support
Choose the best option from the given alternatives
(C) Studious
which can be substituted for the
(D) Ignorant given word/sentence.
Ans - C One, who supervises in the examination hall
Question No.25 (A) Invigilator

Fill in the blanks with suitable Article from the (B) Shoal
given alternatives. (C) Epic

__________ girl sitting beside me raised her hand. (D) Aspirant

(A) a Ans - A
Question No.9
(B) The
Fill in the blanks with suitable Preposition from the
(C) an
given alternatives.
(D) No article
A tall perfectly decorated Christmas tree stood
Ans - B _____________ the entry gate.
(A) throughout
Question No.1 (B) since
Choose the best option from the given alternatives (C) besides
which can be substituted for the (D) beside
given word/sentence. Ans - D
Strong dislike between two persons Question No.10
(A) Sympathy Choose the word which expresses nearly the
(B) Telepathy opposite meaning of the given word "
(C) Antipathy INFIRM ".
(D) Apathy (A) Anemic
Ans - C (B) Strong
Question No.2 (C) Fragile
(D) Faint
Ans - B (A) a
Question No.11 (B) an
Fill in the blanks with suitable Article from the (C) the
given alternatives. (D) No Article
Dr. Philips visits schools and colleges to educate Ans - C
students on _________Tuberculosis. Question No.15
(A) a Choose the best option from the given alternatives
(B) an which can be substituted for the
(C) the given word/sentence.
(D) No article Animal that feeds on plants
Ans - D (A) Predators
Question No.12 (B) Herbivorous
Fill in the blanks with suitable Article from the (C) Omnivorous
given alternatives. (D) Carnivorous
The video has gone viral and has collected nearly Ans - B
______________ 4.2 million views. Question No.16
(A) a Replace the underlined phrase grammatically and
(B) an conceptually with the help of the given
(C) the options. If the given sentence is correct then select
(D) No article the option 'The given sentence is
Ans - D correct'.
Question No.13 The engaging show were set in a village where
Choose the word which expresses nearly the Jeetendra’s character has started his
opposite meaning of the given word " first government job.
DESOLATE ". (A) was set in a village whom
(A) Unused (B) was sets in a village where
(B) Lonely (C) was set in a village where
(C) Empty (D) The given sentence is correct
(D) Full Ans - C
Ans - D Question No.17
Question No.14 Find the word which is correctly spelt from the
Fill in the blanks with suitable Article from the given options.
given alternatives. (A) Perplax
Lionel Messi wins best FIFA player of (B) Sibbling
____________ year. (C) Regretable
(D) Inclination Choose the word which best expresses the similar
Ans - D meaning of the given word "
Question No.18 IMPREGNABLE ".
Replace the underlined phrase grammatically and (A) Breakable
conceptually with the help of the given (B) Calm
options. If the given sentence is correct then select (C) Crazy
the option 'The given sentence is (D) Firm
correct'. Ans - D
The Academy announced that the best picture Question No.22
category will now be fixed at 10 films, Fill in the blanks with suitable Preposition from the
rather than the fluctuating number of nominations. given alternatives.
(A) announce that the best picture category will I wear a ring __________ my finger.
(B) announced that the best picture category need (A) above
to (B) at
(C) The given sentence is correct (C) on
(D) announce which the best picture category will (D) below
Ans - C Ans - C
Question No.19 Question No.23
Find the word which is correctly spelt from the Fill in the blanks with suitable Preposition from the
given options. given alternatives.
(A) Prospactive There is a vegetable shop just ______________ the
(B) Sugestive road.
(C) Innovative (A) since
(D) Mercenery (B) besides
Ans - C (C) across
Question No.20 (D) about
Choose the word which best expresses the similar Ans - C
meaning of the given word " Question No.24
PROFOUND ". Replace the underlined phrase grammatically and
(A) Ignorant conceptually with the help of the given
(B) Moderate options. If the given sentence is correct then select
(C) Superficial the option 'The given sentence is
(D) Intellectual correct'.
Ans - D Residing primarily on Western Australia’s Rottnest
Question No.21 Island, Quokkas loves for sleep and
are the happiest of all animals. Zero-budget farming is ________ form of natural
farming which is neither chemicalloaded nor
(A) loves to sleep and is the happiest of
organic.
(B) The given sentence is correct
(A) a
(C) love to sleep and is the happiest of
(D) love to sleep and are the happiest of (B) an

Ans - D (C) the


Question No.25 (D) No article
Replace the underlined phrase grammatically and
Ans - A
conceptually with the help of the given
Question No.3
options. If the given sentence is correct then select
the option 'The given sentence is correct'. Fill in the blanks with suitable Preposition from the
given alternatives.
The face value of fake currency in Indian rupees
The rumors spread _____________ the country.
was around Rs 87 crore, as per the
preliminary estimate. (A) across
(A) The given sentence is correct (B) below
(B) of fake currency in Indian rupees were
(C) between
(C) of fake currency above Indian rupees were
(D) along
(D) of fake currency above Indian rupees was
Ans - A Ans - A

Question No.4

Question No.1 Fill in the blanks with suitable Article from the
given alternatives.
Choose the word which expresses nearly the
opposite meaning of the given word " The January of ______ 2019 was hot and dry.

IMBECILE ". (A) a

(A) Inane (B) an

(B) Dull (C) the

(C) Smart (D) No Article

(D) Thick Ans - D

Ans - C Question No.5

Question No.2 Choose the word which best expresses the similar
meaning of the given word "
Fill in the blanks with suitable Article from the
given alternatives. PERTURB ".

(A) Aid
(B) Organize Saudi Arabia is ______________ world's top oil
exporter and has been the supplier of last
(C) Confuse
resort for decades.
(D) Soothe
(A) a
Ans - C
(B) an
Question No.6
(C) the
Choose the word which best expresses the similar
meaning of the given word " (D) No article

ENCUMBRANCE ". Ans - C

(A) Advantage Question No.9

(B) Burden Find the word which is correctly spelt from the
given options.
(C) Support
(A) Paraphrese
(D) Laziness
(B) Resollution
Ans - B
(C) Uniformmity
Question No.7
(D) Receptive
Replace the underlined phrase grammatically and
conceptually with the help of the given Ans - D

options. If the given sentence is correct then select Question No.10


the option 'The given sentence is
Replace the underlined phrase grammatically and
correct'. conceptually with the help of the given

“Consuming an healthy diet and avoiding certain” options. If the given sentence is correct then select
food items go a long way in managing the option 'The given sentence is

cramps and other symptoms during the period. correct'.

(A) Consuming an healthy diet and avoide certain She “have been playing” volleyball when Rocky
came.
(B) Consuming a healthy diet and avoide certain
(A) has been playing
(C) The given sentence is correct
(B) The given sentence is correct
(D) Consuming a healthy diet and avoiding certain
(C) had been playing
Ans - D
(D) is playing
Question No.8
Ans - C
Fill in the blanks with suitable Article from the
given alternatives. Question No.16
Fill in the blanks with suitable Preposition from the given word/sentence.
given alternatives.
A person who regards the whole world as his
He was born ______________ 1990. country

(A) on (A) Cosmopolitan

(B) in (B) Obsolete

(C) at (C) Veteran

(D) for (D) Invocation

Ans - B Ans - A

Question No.17 Question No.20

Choose the best option from the given alternatives Find the word which is correctly spelt from the
which can be substituted for the given options.

given word/sentence. (A) Ordinence

Constant efforts to achieve something (B) Etternal

(A) Perseverance (C) Mitigate

(B) Dormitory (D) Grievuous

(C) Wardrobe Ans - C

(D) Reservoir Question No.21

Ans - A Replace the underlined phrase grammatically and


conceptually with the help of the given
Question No.18
options. If the given sentence is correct then select
Choose the word which expresses nearly the the option 'The given sentence is
opposite meaning of the given word "
correct'.
EFFEMINATE ".
One of the most important and culturally-rich
(A) Epicene festivals of the country, Onam is widely

(B) Manly celebrated by the Malayali community across the


nation.
(C) Womanlike
(A) One of the more important and culturally-rich
(D) Sissyish festival
Ans - B (B) One of the more important and culturally-rich
festivals
Question No.19
(C) The given sentence is correct
Choose the best option from the given alternatives
which can be substituted for the (D) One of the most important and culturally-rich
festival
Ans - C Ans - D

Question No.22 Question No.25

Choose the best option from the given alternatives Fill in the blanks with suitable Preposition from the
which can be substituted for the given alternatives.

given word/sentence. Susan and I agree ____________ the book.

A person who has changed his faith (A) about

(A) Chauvinist (B) under

(B) Bohemian (C) beneath

(C) Apostate (D) near

(D) Connoisseur Ans - A

Ans - C
Question No.1
Question No.23
Fill in the blanks with suitable Article from the
Find the word which is correctly spelt from the given alternatives.
given options. The Hindu is _________ ninth-most-circulated
newspaper in India.
(A) Negligible
(A) a
(B) Miscariage
(B) an
(C) Jugernaut
(C) No Article
(D) Immuenity (D) the
Ans - A Ans - D

Question No.24 Question No.2


Choose the word which best expresses the similar
Replace the underlined phrase grammatically and
meaning of the given word "
conceptually with the help of the given
RECEPTIVE ".
options. If the given sentence is correct then select
the option 'The given sentence is (A) Sensitive
(B) Unwilling
correct'.
(C) unsusceptible
Severe gum diseases “may also increases the risk
for” pre-term delivery and premature birth. (D) Against
Ans - A
(A) may also increases the risk of
Question No.3
(B) The given sentence is correct
Fill in the blanks with suitable Preposition from the
(C) can also increases the risk of given alternatives.
A gauge of oil-market volatility ____________
(D) may also increase the risk of
Thursday rose to the highest level since
March of last year. Question No.12
(A) on Fill in the blanks with suitable Preposition from the
given alternatives.
(B) at
The squad found a bomb ___________ the truck.
(C) in
(A) to
(D) through
(B) underneath
Ans - A
(C) for
Question No.4
(D) against
Fill in the blanks with suitable Article from the
given alternatives. Ans - B
_____________ essay is generally a short piece of Question No.13
writing outlining the writer’s perspective
Choose the best option from the given alternatives
or story. which can be substituted for the
(A) The given word/sentence.
(B) A House or shelter of a gipsy
(C) An (A) Sanatorium
(D) No Article (B) Octagon
Ans - C (C) Escort
Question No.5 (D) Caravan
Fill in the blanks with suitable Preposition from the Ans - D
given alternatives.
Question No.14
The chief guest asked the teachers to tie a ribbon
Replace the underlined phrase grammatically and
____________ the gift box.
conceptually with the help of the given
(A) beyond
options. If the given sentence is correct then select
(B) between the option 'The given sentence is
(C) around correct'.
(D) since Our team will played final match by tomorrow.
Ans - C (A) The given sentence is correct
Question No.6 (B) have played
Find the word which is correctly spelt from the (C) will have being played
given options.
(D) will have played
(A) Preciptate
Ans - D
(B) Pesimist
Question No.15
(C) Symetry
Replace the underlined phrase grammatically and
(D) Tributary conceptually with the help of the given
Ans - D options. If the given sentence is correct then select
the option 'The given sentence is
correct'. attention.
The holistic health and yoga guru believe people (A) for her looks that caughts
must do yoga to find an purpose in life
(B) The given sentence is correct
and not because they want to lose weight and get
(C) for her look that caughts
in shape.
(D) for her look that caught
(A) believes people must do yoga to find a
Ans - B
(B) believes people must does yoga to find a
Question No.19
(C) The given sentence is correct
Choose the word which expresses nearly the
(D) believes people must do yoga to find an
opposite meaning of the given word "POISED ".
Ans - A
(A) Float
Question No.16
(B) Refuse
Choose the best option from the given alternatives
(C) Balance
which can be substituted for the
(D) Stand
given word/sentence.
Ans - B
Loss of memory
Question No.20
(A) Amnesia
Choose the best option from the given alternatives
(B) Creche
which can be substituted for the
(C) Asylum
given word/sentence.
(D) Reflex
One who does not express himself freely
Ans - A
(A) Jury
Question No.17
(B) Pacifist
Find the word which is correctly spelt from the
(C) Numismatist
given options.
(D) Introvert
(A) Qualifiad
Ans - D
(B) Transgresion
Question No.21
(C) Robust
Choose the word which expresses nearly the
(D) Personaige
opposite meaning of the given word "BREVITY ".
Ans - C
(A) Crispness
Question No.18
(B) Transience
Replace the underlined phrase grammatically and
(C) Lengthiness
conceptually with the help of the given
(D) Shortness
options. If the given sentence is correct then select
the option 'The given sentence is Ans - C
correct'. Question No.22
It is Melania Trump's interesting choice of colours Find the word which is correctly spelt from the
for her looks that caught people's given options.
(A) Benevolant Ans - B
(B) Volataile
(C) Biodegradeble Question No.1
(D) Celestial Choose the word which best expresses the similar
meaning of the given word "
Ans - D
DISGUISE ".
Question No.23
(A) Divulge
Choose the word which best expresses the similar
meaning of the given word " INTERIM". (B) Disclose
(A) Acting (C) Represent
(B) Advantage (D) Conceal
(C) Continual Ans - D
(D) Permanent Question No.2
Ans - A Replace the underlined phrase grammatically and
conceptually with the help of the given
Question No.24
options. If the given sentence is correct then select
Replace the underlined phrase grammatically and
the option 'The given sentence is
conceptually with the help of the given
correct'.
options. If the given sentence is correct then select
the option 'The given sentence is We will playing cricket next Monday with our team.
correct'. (A) The given sentence is correct
Hiromu Inada win the Ironman world (B) will have playing
championship at age of 85.
(C) will be playing
(A) wons the Ironman world championship at the
(D) have played
(B) won the Ironman world championship at the
Ans - C
(C) wins the Ironman world championship at a
Question No.3
(D) The given sentence is correct
Choose the word which expresses nearly the
Ans - B opposite meaning of the given word "
Question No.25 MUNDANE ".
Fill in the blanks with suitable Article from the (A) Routine
given alternatives.
(B) Abnormal
In April last year, ______ Reserve Bank of India
(C) Normal
issued a directive on Storage of Payment
(D) Banal
System Data.
Ans - B
(A) an
Question No.4
(B) the
Fill in the blanks with suitable Article from the
(C) a
given alternatives.
(D) No Article
Our professor gave ________ important (D) Funambulist
information about agriculture.
Ans - A
(A) a
Question No.8
(B) the
Choose the word which best expresses the similar
(C) No Article meaning of the given word " GAZE ".
(D) an (A) Important
Ans - D (B) Look
Question No.5 (C) Ignorance
Fill in the blanks with suitable Preposition from the (D) Moderate
given alternatives.
Ans - B
Most of the Multinational Companies are having
Question No.9
their branches ___________ the globe.
Find the word which is correctly spelt from the
(A) about
given options.
(B) across
(A) Recipiant
(C) at
(B) Sanction
(D) above
(C) Quickan
Ans - B
(D) Chronecle
Question No.6
Ans - B
Choose the best option from the given alternatives
Question No.15
which can be substituted for the
Replace the underlined phrase grammatically and
given word/sentence.
conceptually with the help of the given
Person who eats too much
options. If the given sentence is correct then select
(A) Pessimist the option 'The given sentence is
(B) Predator correct'.
(C) Octogenarian Leading holistic health guru and corporate life
coach Dr Mickey Mehta cannot stress
(D) Glutton
enough in the importance on yoga.
Ans - D
(A) an enough on the importance
Question No.7
(B) a enough on the importance of
Choose the best option from the given alternatives
which can be substituted for the (C) enough on the importance of
given word/sentence. (D) The given sentence is correct
One who often talks of his achievements Ans - C
(A) Egotist Question No.16
(B) Reciprocate Fill in the blanks with suitable Preposition from the
given alternatives.
(C) Psephologist
We have been living in this palace ___________ Hiromu Inada from Japan already hold the title of a
1956. world’s oldest Ironman.
(A) against (A) The given sentence is correct
(B) since (B) from Japan already hold the title of an
(C) for (C) for Japan already holds the title of the
(D) through (D) from Japan already holds the title of the
Ans - B Ans - D
Question No.17 Question No.20
Replace the underlined phrase grammatically and Find the word which is correctly spelt from the
conceptually with the help of the given given options.
options. If the given sentence is correct then select (A) Transpire
the option 'The given sentence is
(B) Quantem
correct'.
(C) Comensurate
She wore minimal makeup and kept her hair open
(D) Colaborate
with a side partition.
Ans - A
(A) wores minimal makeup and kept her
Question No.21
(B) The given sentence is correct
Choose the best option from the given alternatives
(C) wore minimal makeup and keep her
which can be substituted for the
(D) wores minimal makeup and keep her
given word/sentence.
Ans - B
One who is not easily pleased by anything
Question No.18
(A) Fastidious
Fill in the blanks with suitable Article from the
(B) Renovation
given alternatives.
(C) Insolation
Today is _____________ hottest day of this year.
(D) Anonymous
(A) a
Ans - A
(B) the
Question No.22
(C) an
Choose the word which expresses nearly the
(D) No Article
opposite meaning of the given word "
Ans - B
ACCLAMATION ".
Question No.19
(A) Cheers
Replace the underlined phrase grammatically and
(B) Salutation
conceptually with the help of the given
(C) Silence
options. If the given sentence is correct then select
the option 'The given sentence is (D) Applause
correct'. Ans - C
Question No.23
Fill in the blanks with suitable Article from the (D) about
given alternatives.
Ans - A
__________ Johannes Ager is an Austrian
Question No.2
professional tennis player.
Choose the best option from the given alternatives
(A) A
which can be substituted for the
(B) An
given word/sentence.
(C) The
A list of explanations of rare, technical or obsolete
(D) No Article words
Ans - D (A) Antonym
Question No.24 (B) Synonym
Find the word which is correctly spelt from the (C) Glossary
given options.
(D) Acronym
(A) Circummlocution
Ans - C
(B) Gracious
Question No.3
(C) Falacy
Fill in the blanks with suitable Article from the
(D) Radecal given alternatives.
Ans - B I experienced _________ euphoric feeling after
winning the game.
Question No.25
(A) a
Fill in the blanks with suitable Preposition from the
given alternatives. (B) an
India won’t back down _________ its plan for (C) the
mandatory data localisation.
(D) No article
(A) under
Ans - A
(B) near
Question No.4
(C) on
Choose the best option from the given alternatives
(D) since which can be substituted for the
Ans - C given word/sentence.
Accidental good fortune

Question No.1 (A) Fluke

Fill in the blanks with suitable Preposition from the (B) Brood
given alternatives. (C) Shell
Centre approves the construction of buds _______ (D) Accomplice
Kaleswaram project.
Ans - A
(A) under
Question No.5
(B) since
Replace the underlined phrase grammatically and
(C) by conceptually with the help of the given
options. If the given sentence is correct then select (A) Unaffected
the option 'The given sentence is
(B) Jubilent
correct'.
(C) Ephimeral
The US Federal Reserve recently signalled a major
(D) Garulous
shift in its approach to managing
Ans - A
inflation in a bid to aid the country’s economy’s
recovery. Question No.9
(A) an major shift in its approach to managing Find the word which is correctly spelt from the
inflation in a given options.
(B) an major shift in its approach to managing (A) Gradiant
inflation in an
(B) Asimilate
(C) a major shift on its approach to managing
inflation in an (C) Observant

(D) The given sentence is correct (D) Mithical

Ans - D Ans - C

Question No.6 Question No.10

Choose the word which best expresses the similar Fill in the blanks with suitable Article from the
meaning of the given word " given alternatives.

SUBSTANTIAL ". My friend was _________ FBI agent in his early


age.
(A) Miniature
(A) a
(B) Selfish
(B) an
(C) Insignificant
(C) the
(D) Important
(D) No article
Ans - D
Ans - B
Question No.7
Question No.11
Choose the best option from the given alternatives
which can be substituted for the Choose the word which best expresses the similar
meaning of the given word " PLUNGE
given word/sentence.
".
A book written by an unknown
(A) Fall
(A) Extempore
(B) Ascent
(B) Beneficiary
(C) Rise
(C) Anonymous
(D) Rough
(D) Orator
Ans - A
Ans - C
Question No.12
Question No.8
Replace the underlined phrase grammatically and
Find the word which is correctly spelt from the conceptually with the help of the given
given options.
options. If the given sentence is correct then select Ans - D
the option 'The given sentence is
Question No.15
correct'.
Find the word which is correctly spelt from the
Very few clubs around a world may have afford the given options.
Argentinian superstar.
(A) Notorious
(A) clubs around the world must
(B) Eloquant
(B) The given sentence is correct
(C) Incorigible
(C) club on the world could
(D) Denomnation
(D) clubs around the world could
Ans - A
Ans - D
Question No.16
Question No.13
Fill in the blanks with suitable Article from the
Replace the underlined phrase grammatically and given alternatives.
conceptually with the help of the given
______ bread is the predominant food prepared
options. If the given sentence is correct then select from a dough of flour and water.
the option 'The given sentence is
(A) a
correct'.
(B) an
The best indicators of a university’s performance is
(C) the
the learning outcomes and how its
(D) No article
education has impacted the students and society.
Ans - D
(A) The given sentence is correct
Question No.17
(B) was the learning outcomes and how
Choose the word which expresses nearly the
(C) is the learning outcomes and what
opposite meaning of the given word "
(D) are the learning outcomes and how
BLISS ".
Ans - D
(A) Happiness
Question No.14
(B) Rapture
Replace the underlined phrase grammatically and
(C) Paradise
conceptually with the help of the given
(D) Sorrow
options. If the given sentence is correct then select
the option 'The given sentence is Ans - D
correct'. Question No.18
Actor Chadwick Boseman passed away in Friday Fill in the blanks with suitable Preposition from the
after a long battle with colon cancer. given alternatives.
(A) passed away on Friday after a long battle on India’s new data localisation provisions make it
mandatory ___________ foreign
(B) The given sentence is correct
companies to store personal data of their users and
(C) passed away in Friday after a long battle on
customers in India within the Indian
(D) passed away on Friday after a long battle with
territory.
(A) for Ans - D
(B) behind Question No.2
(C) about Choose the word which expresses nearly the
opposite meaning of the given word " PLAUSIBLE ".
(D) of
(A) Probable
Ans - A
(B) Logical
Question No.19
(C) Creditable
Fill in the blanks with suitable Preposition from the
given alternatives. (D) Impossible
A tablespoonful __________ peanut butter Ans - D
contains almost 100 calories.
Question No.8
(A) to
Choose the word which best expresses the similar
(B) of meaning of the given word " DILATE ".
(C) between (A) Curtail
(D) since (B) Condense
Ans - B (C) Diminish
Question No.25 (D) Enlarge
Choose the word which expresses nearly the Ans - D
opposite meaning of the given word "
Question No.9
SLENDER ".
Fill in the blanks with suitable Article from the
(A) Heavy given alternatives.
(B) Narrow He arrived in ___________ India yesterday.
(C) Fragile (A) a
(D) Spare (B) an
Ans - A (C) the
(D) No article

Question No.1 Ans - D

Fill in the blanks with suitable Preposition from the Question No.10
given alternatives. Replace the underlined phrase grammatically and
Portugal extended its unbeaten record to 10 conceptually with the help of the given
matches __________ losing to Uruguay at the options. If the given sentence is correct then select
last World Cup. the option 'The given sentence is

(A) for correct'.

(B) under The minister were ready to speaking with the


people affected by the disaster left by the
(C) about
hurricane.
(D) since
(A) The given sentence is correct
(B) was ready to spoke with the people affected by Question No.14
(C) was ready for speak with the people affected by Choose the word which best expresses the similar
meaning of the given word " DISDAIN ".
(D) was ready to speak with the people affected by
(A) Approval
Ans - D
(B) Admiration
Question No.11
(C) Sympathy
Fill in the blanks with suitable Preposition from the
given alternatives. (D) Hate
They walked ___________ the bridge. Ans - D
(A) across Question No.15
(B) for Find the word which is correctly spelt from the
given options.
(C) since
(A) Unwaranted
(D) about
(B) Vernakular
Ans - A
(C) Underscore
Question No.12
(D) Fragmanted
Fill in the blanks with suitable Article from the
given alternatives. Ans - C
He is one of _________ important batsmen in his Question No.16
team.
Fill in the blanks with suitable Preposition from the
(A) a given alternatives.
(B) an The pen is __________ the paper.
(C) the (A) for
(D) No article (B) through
Ans - C (C) besides
Question No.13 (D) beneath
Replace the underlined phrase grammatically and Ans - D
conceptually with the help of the given
Question No.17
options. If the given sentence is correct then select
Choose the best option from the given alternatives
the option 'The given sentence is
which can be substituted for the
correct'.
given word/sentence.
Science is a very difficult subject, but my teacher
That which lasts forever
does a great job easing us through it.
(A) Eternal
(A) but my teachers does a great job easing us
(B) Cannibal
(B) The given sentence is correct
(C) Fugitive
(C) but my teacher do a great job easing us
(D) Internal
(D) but my teachers does a greater job easing us
Ans - A
Ans - B
Question No.18 that which cannot be seen through
Find the word which is correctly spelt from the (A) Transparent
given options.
(B) Cyclic
(A) Incidantal
(C) Unique
(B) Saluetation
(D) Opaque
(C) Disputeble
Ans - D
(D) Orthodox
Question No.22
Ans - D
Replace the underlined phrase grammatically and
Question No.19 conceptually with the help of the given
Replace the underlined phrase grammatically and options. If the given sentence is correct then select
conceptually with the help of the given the option 'The given sentence is
options. If the given sentence is correct then select correct'.
the option 'The given sentence is
As the cost of living continues to surge, she will be
correct'. unable to paying the fees if the
Because she was so chaotic in the shopping mall, monthly wage does not.
Jane’s father refused to take her
(A) to pay the fees if the monthly wage do not
shopping during the holidays.
(B) to pay the fees if the monthly wage does not
(A) she was so chaotic in an shopping mall
(C) The given sentence is correct
(B) she so chaotic the shopping mall
(D) to paying the fees if the monthly wage do not
(C) she was so chaotic over the shopping mall
Ans - B
(D) The given sentence is correct
Question No.23
Ans - D
Choose the best option from the given alternatives
Question No.20 which can be substituted for the
Fill in the blanks with suitable Article from the given word/sentence.
given alternatives.
A person who looks at the darker side of things
Maria Sharapova is __________ Russian
(A) Optimist
professional tennis player.
(B) Amnesia
(A) a
(C) Pessimist
(B) an
(D) Somnambulist
(C) the
Ans - C
(D) No article
Question No.24
Ans - A
Find the word which is correctly spelt from the
Question No.21
given options.
Choose the best option from the given alternatives
(A) Excomunicate
which can be substituted for the
(B) Disolution
given word/sentence.
(C) Vigilant (B) Lock
(D) Constently (C) Free
Ans - C (D) Give
Question No.25 Ans - B
Choose the word which expresses nearly the Question No.4
opposite meaning of the given word " SKEPTICAL ".
Find the word which is correctly spelt from the
(A) Doubtful given options.
(B) Agnostic (A) Ethreal
(C) Certain (B) Exemplary
(D) Mistrustful (C) Feesible
Ans - C (D) Asimilate
Ans - B

Question No.1 Question No.5

Choose the word which expresses nearly the Find the word which is correctly spelt from the
opposite meaning of the given word " ENCROACH given options.
". (A) Gradiant
(A) Ignore (B) Eroneous
(B) Intrude (C) Fragmanted
(C) Crash (D) Exhaustive
(D) Interfere Ans - D
Ans - A Question No.6
Question No.2 Choose the word which best expresses the similar
Fill in the blanks with suitable Article from the meaning of the given word " REPLETE ".
given alternatives. (A) Charged
We are trying to give ______ incredible (B) Enlarge
performance in the farewell day function.
(C) Unfilled
(A) a
(D) Empty
(B) an
Ans - A
(C) the
Question No.7
(D) No article
Fill in the blanks with suitable Preposition from the
Ans - B given alternatives.
Question No.3 The rumour spread all ____________ the town.
Choose the word which best expresses the similar (A) since
meaning of the given word "
(B) for
EMBRACE ".
(C) across
(A) Distrust
(D) on brotherhood with its citizens.
Ans - C (A) was the feeling of love and devotion for one’s
Question No.13 (B) is feeling of love and devotion for one
Choose the best option from the given alternatives (C) The given sentence is correct
which can be substituted for the
(D) is the feeling of love and devotion for one’s
given word/sentence.
Ans - D
One who journeys from place to place
Question No.16
(A) Amphibian
Fill in the blanks with suitable Article from the
(B) Itinerant given alternatives.
(C) Polygamy The chief selector said that the former skipper
remains _____________ best wicket-keeper
(D) Teetotaller
and finisher in the game.
Ans - B
(A) a
Question No.14
(B) an
Replace the underlined phrase grammatically and
conceptually with the help of the given (C) the
options. If the given sentence is correct then select (D) No article
the option 'The given sentence is
Ans - C
correct'.
Question No.17
All three sectors have skilled
Fill in the blanks with suitable Article from the
labourers/professionals who have been working
given alternatives.
hard to
Our college has set up ____________ counselling
build the country’s economy.
centre and a student welfare group that
(A) sector have skilled labourers/professionals who
helps students to cope up with the stress.
has
(A) a
(B) sector have skilled labourers/professionals who
have (B) an
(C) The given sentence is correct (C) the
(D) sectors has skilled labourers/professionals who (D) No article
have
Ans - A
Ans - C
Question No.18
Question No.15
Choose the word which expresses nearly the
Replace the underlined phrase grammatically and opposite meaning of the given word " FOE ".
conceptually with the help of the given
(A) Rival
options. If the given sentence is correct then select
the option 'The given sentence is (B) Enemy

correct'. (C) Assistant

Patriotism is the feel of love and devotion for one’s (D) Adversary
country and a sense of alliance and
Ans - C Union ministers Nirmala Sitharaman and
Subrahmanyam Jaishankar will being
Question No.19
conferred the Distinguished Alumni Award by
Choose the best option from the given alternatives
Jawaharlal Nehru University.
which can be substituted for the
(A) The given sentence is correct
given word/sentence.
(B) will conferred the Distinguished
An event causing sudden damage or suffering
(C) will be confers the Distinguished
(A) Nostalgia
(D) will be conferred the Distinguished
(B) Chronology
Ans - D
(C) Ephemeral
Question No.23
(D) Catastrophe
Choose the best option from the given alternatives
Ans - D
which can be substituted for the
Question No.20
given word/sentence.
Fill in the blanks with suitable Preposition from the
The art of writing or solving codes
given alternatives.
(A) Cryptograph
They are living _______________ the temple.
(B) Potable
(A) behind
(C) Infallible
(B) with
(D) Pedantic
(C) over
Ans - A
(D) before
Question No.24
Ans - A
Find the word which is correctly spelt from the
Question No.21
given options.
Fill in the blanks with suitable Preposition from the
(A) Peculier
given alternatives.
(B) Reproache
The book is _____________ the table.
(C) Unanimity
(A) on
(D) Unmitigatad
(B) through
Ans - C
(C) across
Question No.25
(D) about
Replace the underlined phrase grammatically and
Ans - A
conceptually with the help of the given
Question No.22
options. If the given sentence is correct then select
Replace the underlined phrase grammatically and the option 'The given sentence is
conceptually with the help of the given
correct'.
options. If the given sentence is correct then select
The corals in the southern Israeli port Eilat are
the option 'The given sentence is
more like perishing due to the toxins and
correct'.
pollution from human activity and industry The act of intentionally killing oneself
entering the bay on a daily basis.
(A) Genocide
(A) are most likely perishing due to the
(B) Infanticide
(B) is most likely perishes due to the
(C) Sorocide
(C) is most likely perishing due to the
(D) Suicide
(D) The given sentence is correct
Ans - D
Ans - A
Question No.4
Fill in the blanks with suitable Article from the
Question No.1 given altematives.

Replace the underlined phrase grammatically and The January of _____ 2019 was hot and dry.
conceptually with the help of the given (A) a
options. If the given sentence is correct then select (B) an
the option The given sentence is
(C) the
Correct.
(D) No Article
The economic situation of farmers in the country is
a dismal one which has caused Ans - D
many of them to lose their livelihood. Question No.5
(A) in the country are a dismal one which has Choose the best option from the given alternatives
which can be substituted for the
(B) The given sentence is correct
given word/sentence.
(C) in the country are a dismal one which have
A person who travels on foot
(D) in the country is a dismal one which have
(A) Escapist
Ans - B
(B) Teetotaller
Question No.2
(C) Pedestrian
Fill in the blanks with suitable Article from the
given altematives. (D) Belligerent
The Supreme Court on Tuesday ordered immediate Ans - C
release of all ______ Journalists.
Question No.6
(A) a
Choose the word which expresses nearly the
(B) the opposite meaning of the given word
(C) an DEFAMATION".
(D) No Article (A) slam
Ans - B (B) Approval
Question No.3 (C) Lie
Choose the best option from the given alternatives (D) Detraction
which can be substituted for the
Ans - B
given word/sentence.
Question No.7
Fill in the blanks with suitable Preposition from the (C) for
given alternatives.
(D) across
The key chain is ______ the paper.
Ans - D
(A) about
Question No.11
(B) till
Choose the best option from the given alternatives
(C) beneath which can be substituted for the
(D) since given word/sentence.
Ans - C A home for children without parents
Question No.8 (A) Catastrophe
Choose the word which best expresses the similar (B) Sporadic
meaning of the given word
(C) Orphanage
STAGGER ".
(D) Widow
(A) Steady
Ans - C
(B) Peak
Question No.12
(C) Shake
Choose the word which expresses nearly the
(D) Continue opposite meaning of the given word
Ans - C BESET.
Question No.9 (A) Clarify
Replace the underlined phrase grammatically and (B) Invade
conceptually with the help of the given
(C) Harass
options. If the given sentence is correct then select
(D) Nudge
the option The given sentence is
Ans - A
correct.
Question No.13
Reputed colleges has more freedom because they
exist outside the city limit. Replace the underlined phrase grammatically and
conceptually with the help of the given
(A) The given sentence is corect
options. If the given sentence is correct then select
(B) have most freedom because they
the option The given sentence is
(C) has the most freedom because they
correct.
(D) have more freedom because they
The data show which the digital modes of payment
Ans - D recorded sharp fall in value
Question No.10 transacted during the month.
Fill in the blanks with suitable Preposition from the (A) Shows that the digital modes of
given aternatives.
(B) The given sentence is correct
I saw him walking _________ the street.
(C) Shows which the digital modes of
(A) with
(D) Showing that the digital modes of
(8) at
Ans - A The government has decided to allow
wedding.ceremonies in religious places aftera
Question No.14
request by leaders of different organisations.
Choose the word which best expresses the similar
meaning of the given word (A) have decided for allow wedding ceremonies in
SHUNNED (B) has decided for allow wedding ceremonies in
(A) Cherish (C) have decided to allow wedding ceremonies in
(B) Ignore (D) The given sentence is correct
(C) Accept Ans - D
(D) Lock Question Nao.18
Ans - B Find the word which is correctly spelt from the
given options.
Question No.15
(A) Uninteligible
Fill in the blanks with suitable Article from the
given alternatives (B) Introvart
Netaji Subhash Chandra Bose was ______ (C) Erudite
important leader of lIndian independence
(D) Cardiolgist
movement
Ans - C
(A) a
Question No.19
(B) the
Find the word which is correctly spelt from the
(C) No Article
given options.
(D) an
(A) Transiant
Ans - D
(B) Medeate
Question No.16
(C) Impanding
Fill in the blanks with suitable Preposition from the
(D) Virtuoso
given alternatives.
Ans - D
She has been writing examination ______ morning.
Question No.25
(A) since
Find the word which is correctly spelt from the
(B) by
given options.
(C) for
(A) Oscilate
(D) over
(B) Legitimate
Ans - A
(C) Emphathy
Question No.17
(D) Problamatic
Replace the underlined phrase grammatically and
Ans - B
conceptually with the help of the given
options. If the given sentence is correct then select
the option The given sentence is Traffic Police SI 2022
correct.
1.Fill in the blanks with suitable Article from the Mary opened the window.
given alternatives.
(A) The window was being opened by Mary.
Saudi Arabia is ______ world's top oil exporter and
(B) The window is opened by Mary.
has been the supplier of last
(C) The window was opened by Mary.
resort for decades.
(D) The window has been opened by Mary.
(A) No article
Ans - C
(B) a
5.Replace the underlined phrase grammatically and
(C) the
conceptually with the help of the given
(D) an
options. If the given sentence is correct then select
Ans - C the option 'The given sentence is
2.Replace the underlined phrase grammatically and correct'.
conceptually with the help of the given
Actor Chadwick Boseman " passed away in Friday
options. If the given sentence is correct then select after a long battle with" colon cancer.
the option 'The given sentence is
(A) passed away on Friday after a long battle with
correct'.
(B) passed away on Friday after a long battle on
The lady clerk " will works on the guidance course
(C) passed away in Friday after a long battle on
and update his " manager on her
(D) The given sentence is correct
progress every week.
Ans - A
(A) will works on the guidance course and update
her 6.Choose the best option from the given
alternatives which can be substituted for the
(B) will work in the guidance course and update his
given word/sentence.
(C) will work on the guidance course and update
her that which cannot be seen through
(D) The given sentence is correct (A) Transparent
Ans - C (B) Unique
3.From the given alternatives, choose the one that (C) Cyclic
best expresses the meaning of the
(D) Opaque
given idiom/phrase.
Ans - D
Rule out.
7.Convert the given direct/ indirect speech to
(A) To exclude out something indirect/ direct speech with the help of the
(B) To do something immediately given options.
(C) Someone who lacks intelligence "Please don't go away", Siraj said.
(D) No manners (A) Siraj begs them not to go away.
Ans - A (B) Siraj begged me do not go away.
4.Find the correct passive voice for the given (C) Siraj begs me not to go away.
sentence in active voice from the given
(D) Siraj begged me not to go away.
alternatives.
Ans - D Ans - C
8.Find the word which is correctly spelt from the 12.Convert the given direct/ indirect speech to
given options. indirect/ direct speech with the help of the
(A) Denomnation given options.
(B) Eloquant Sanjay said that he was sick and tired of working
for that client.
(C) Incorigible
(A) Sanjay said, "I am sick and tired of working for
(D) Notorious
this client."
Ans - D
(B) Sanjay said, "I have sick and tired of working for
9.Convert the given direct/ indirect speech to this client."
indirect/ direct speech with the help of the
(C) Sanjay said, "I was sick and tired of working for
given options. this client."

Jerome said to me, “I am going to Mumbai.” (D) Sanjay ordered, "I am sick and tired of working
for this client."
(A) Jerome told me that he is going to Mumbai.
Ans - A
(B) Jerome told me that he was going to Mumbai.
13.Replace the underlined phrase grammatically
(C) Jerome told me he is going to Mumbai. and conceptually with the help of the given
(D) Jerome told me he was going to Mumbai. options. If the given sentence is correct then select
Ans - B the option 'The given sentence is

10.Find the correct passive voice for the given correct'.


sentence in active voice from the given Small distractions "also become a issue when it
alternatives. comes to" children with special needs.

He will be writing a test tomorrow. (A) also become an issue when it comes to

(A) A test will be written by him tomorrow. (B) also becomes an issue when it comes to

(B) A test would be being written by him (C) also became a issue when it comes to
tomorrow. (D) The given sentence is correct
(C) A test will have been written by him tomorrow. Ans - A
(D) A test will be being written by him tomorrow. 14.Convert the given direct/ indirect speech to
Ans - D indirect/ direct speech with the help of the

11.From the given alternatives, choose the one given options.


that best expresses the meaning of the He said to her, "Why don't you go today?"
given idiom/phrase. (A) He asked her why she did not go that day.
Against the grain. (B) He told her why she does not go that day.
(A) When you complain about a loss from the past (C) He asked her why she does not go that day.
(B) To present a counter argument (D) He told her why she did not go that day.
(C) A difficult path to be followed Ans - A
(D) To succeed 15.Find the word which is correctly spelt from the
given options.
(A) Ambigous (A) besides
(B) Fastidiuous (B) for
(C) Exculpatory (C) beneath
(D) Hypotheticaly (D) through
Ans - C Ans - C
16.Find the word which is correctly spelt from the 20.Find the word which is correctly spelt from the
given options. given options.
(A) Heavanly (A) Satisfactarily
(B) Percentyle (B) Reappearence
(C) Spiritual (C) Philosophy
(D) Rhetaric (D) Circumferance
Ans - C Ans - C
17.Fill in the blanks with suitable Preposition from 21.Fill in the blank with suitable word from the
the given alternatives. given options.
They are living ___ the temple. A _____ in production leads to heavy losses.
(A) with (A) easy
(B) before (B) tail
(C) over (C) better
(D) behind (D) halt
Ans - D Ans - D
18.In the following question, one part of the 22.In the following question, one part of the
sentence may have an error. Find out which sentence may have an error. Find out which
part of the sentence has an error and select the part of the sentence has an error and select the
option corresponding to it. (Avoid option corresponding to it. (Avoid
punctuation errors) punctuation errors)
(A) Cerebral aneurysm is a serious health condition (A) For many Indians and Pakistanis, particularly /
/ (B) that can turn fatal if not / (C) (B) in rural areas, the intolerable heat
diagnose properly and in time. / (D) No Error will / (C) slash the valuable hours available to work
outdoors. / (D) No Error
(A) B
(A) D
(B) D
(B) B
(C) A
(C) A
(D) C
(D) C
Ans - D
Ans - A
19.Fill in the blanks with suitable Preposition from
the given alternatives. 23.Choose the best option from the given
alternatives which can be substituted for the
The pen is __ the paper.
given word/sentence. (C) wake
A person who looks at the darker side of things (D) woke
(A) Amnesia Ans - B
(B) Pessimist 28.Choose the best option from the given
alternatives which can be substituted for the
(C) Optimist
given word/sentence.
(D) Somnambulist
Accidental good fortune
Ans - B
(A) Shell
24.Fill in the blank with suitable tense.
(B) Fluke
Was she __ her marriage anniversary last week?
(C) Accomplice
(A) celebrates
(D) Brood
(B) will celebrating
Ans - B
(C) celebrate
29.Find the correct passive voice for the given
(D) celebrating
sentence in active voice from the given
Ans - D
alternatives.
25.Fill in the blank with suitable word from the
Raj collects money.
given options.
(A) Money has been collected by Raj.
Many people fought to ____ slavery during the
Civil War. (B) Money is being collected by Raj.
(A) read (C) Money is collected by Raj.
(B) hear (D) Money is been collected by Raj.
(C) dream Ans - C
(D) abolish 30.Find the correct passive voice for the given
sentence in active voice from the given
Ans - D
alternatives.
26.Fill in the blanks with suitable Preposition from
the given alternatives. He is doing his project work.
The rumors spread _____ the country. (A) His project work is being done by him.
(A) along (B) His project work was being done by him.
(B) between (C) His project work is been done by him.
(C) across (D) His project work is being did by him.
(D) below Ans - A
Ans - C 31.Convert the given direct/ indirect speech to
indirect/ direct speech with the help of the
27.Fill in the blank with suitable tense.
given options.
Jegan __ up early in the morning daily.
Rojer said to him, “You have completed your
(A) have wake
project".
(B) wakes
(A) Rojer told him that he completed his project. The special drive "are launched to identify the
dropouts and get" them back to the school
(B) Rojer told him that he has completed his
project. in the state.
(C) Rojer told he had completed his project. (A) is launch to identify the dropouts and get
(D) Rojer told him that he had completed his (B) are launched for identify the dropouts and get
project.
(C) is launched to identify the dropouts and get
Ans - D
(D) The given sentence is correct
32.Fill in the blank with suitable word from the
Ans - C
given options.
36.Convert the given direct/ indirect speech to
Scotland is a country which is _____ for its
indirect/ direct speech with the help of the
hospitality.
given options.
(A) dialysed
He said that his sister was getting married.
(B) conjured
(A) He said, "My sister was getting married."
(C) renowned
(B) He said to me, "My sister is getting married."
(D) confused
(C) He asked, "My sister is getting married."
Ans - C
(D) He said, "My sister is getting married.
33.Find the word which is correctly spelt from the
given options. Ans - D
(A) Observant 37.Choose the best option from the given
alternatives which can be substituted for the
(B) Asimilate
given word/sentence.
(C) Gradiant
A list of explanations of rare, technical or obsolete
(D) Mithical
words
Ans - A
(A) Antonym
34.Fill in the blanks with suitable Article from the
(B) Acronym
given alternatives.
(C) Glossary
He is one of _____ important batsmen in his team.
(D) Synonym
(A) the
Ans - C
(B) No article
38.Fill in the blank with suitable word from the
(C) a
given options.
(D) an
Reshma joined the _____ service after graduation.
Ans - A
(A) nervous
35.Replace the underlined phrase grammatically
(B) diplomatic
and conceptually with the help of the given
(C) lazy
options. If the given sentence is correct then select
the option 'The given sentence is (D) obedient
correct'. Ans - B
39.Fill in the blanks with suitable Article from the (C) Sudha told me that we would get our payment
given alternatives. from our institution tomorrow.
Zero-budget farming is ____ form of natural (D) Sudha told us that we would get our payment
farming which is neither chemical loaded nor from our institution the next day
organic.
Ans - D
(A) No article
43.Fill in the blanks with suitable Article from the
(B) the given alternatives.
(C) an The January of __ 2019 was hot and dry.
(D) a (A) the
Ans - D (B) an
40.From the given alternatives, choose the one (C) a
that best expresses the meaning of the
(D) No Article
given idiom/phrase.
Ans - D
A hot potato.
44.In the following question, one part of the
(A) Be happy when a person leaves sentence may have an error. Find out which
(B) A good idea or plan part of the sentence has an error and select the
option corresponding to it. (Avoid
(C) Instantly
punctuation errors)
(D) Something that is hard or terrible to handle
(A) The fear of public speaking is a / (B) common
Ans - D
phobia. It might range / (C) from mild
41.Choose the word which best expresses the
jitters to crippling fear and panic. / (D) No Error
similar meaning of the given word " REPLETE
(A) B
".
(B) C
(A) Empty
(C) D
(B) Unfilled
(D) A
(C) Charged
Ans - C
(D) Enlarge
45.Choose the best option from the given
Ans - C
alternatives which can be substituted for the
42.Convert the given direct/ indirect speech to
given word/sentence.
indirect/ direct speech with the help of the
A book written by an unknown
given options.
(A) Beneficiary
Sudha said to us, “You will get your payment from
your institution tomorrow.” (B) Anonymous
(A) Sudha told us that we will get our payment (C) Orator
from our institution the next day.
(D) Extempore
(B) Sudha told us that we would get our payment
Ans - B
from our institution tomorrow.
46.Fill in the blanks with suitable Preposition from (B) Small
the given alternatives.
(C) Avoid
Portugal extended its unbeaten record to 10
(D) Important
matches __ losing to Uruguay at the
Ans - B
last World Cup.
51.Choose the word which expresses nearly the
(A) under
opposite meaning of the given word "
(B) since
PLAUSIBLE ".
(C) for
(A) Probable
(D) about
(B) Logical
Ans - B
(C) Impossible
47.Fill in the blanks with suitable Preposition from
(D) Creditable
the given alternatives.
Ans - C
Susan and I agree ____ the book.
52.Fill in the blanks with suitable Preposition from
(A) near
the given alternatives.
(B) about
He was born ______ 1990.
(C) beneath
(A) at
(D) under
(B) for
Ans - B
(C) on
48.Choose the word which expresses nearly the
(D) in
opposite meaning of the given word "
Ans - D
SKEPTICAL ".
53.Choose the best option from the given
(A) Certain
alternatives which can be substituted for the
(B) Mistrustful
given word/sentence.
(C) Agnostic
One who journeys from place to place
(D) Doubtful
(A) Teetotaller
Ans - A
(B) Itinerant
49.Choose the word which best expresses the
(C) Amphibian
similar meaning of the given word " DISDAIN ".
(D) Polygamy
(A) Admiration
Ans - B
(B) Hate
54.Choose the word which expresses nearly the
(C) Approval
opposite meaning of the given word "
(D) Sympathy
TURMOIL ".
Ans - B
(A) Disorder
50.Choose the word which best expresses the
(B) Calm
similar meaning of the given word " TRIVIAL ".
(C) Bustle
(A) Large
(D) Disturbance Ans - C
Ans - B 59.Choose the word which expresses nearly the
opposite meaning of the given word " FOE ".
55.From the given alternatives, choose the one
that best expresses the meaning of the (A) Enemy
given idiom/phrase. (B) Adversary
Beat around the bush. (C) Rival
(A) Being in good health (D) Assistant
(B) To make a bad situation even worse Ans - D
(C) Not speaking directly or precisely 60.From the given alternatives, choose the one
that best expresses the meaning of the
(D) Not judging something by its initial appearance
given idiom/phrase.
Ans - C
Hang in there.
56.From the given alternatives, choose the one
that best expresses the meaning of the (A) Don't give up
given idiom/phrase. (B) Start over
Sticky fingers. (C) It's not complicated
(A) Something common (D) Go to sleep
(B) Don't be so critical Ans - A
(C) Keep objects that don't belong to you 61.Replace the underlined phrase grammatically
and conceptually with the help of the given
(D) Stop working on something
options. If the given sentence is correct then select
Ans - C
the option 'The given sentence is
57.Convert the given direct/ indirect speech to
correct'.
indirect/ direct speech with the help of the
Coconut oil exporters are facing difficulties in
given options.
importing copra following higher prices,
Where do they live?
container shortage, rise in shipment rates.
(A) You asked me where they live.
(A) is facing difficulties in importing copra following
(B) You ordered me where they lived.
(B) are facing difficulties in importing copra follow
(C) You asked me about where they lived.
(C) is facing difficulties in importing copra follow
(D) You asked me where they lived.
(D) The given sentence is correct
Ans - D
Ans - D
58.Find the word which is correctly spelt from the
62.Fill in the blank with suitable word from the
given options.
given options.
(A) Continous
They have to showcase an ______ of family
(B) Buraucratic togetherness.

(C) Pilgrimage (A) detection

(D) Sophistary (B) situation


(C) illusion Ans - A
(D) isolation 66.Fill in the blanks with suitable Article from the
given alternatives.
Ans - C
He arrived in ___ India yesterday.
63.Replace the underlined phrase grammatically
and conceptually with the help of the given (A) the
options. If the given sentence is correct then select (B) a
the option 'The given sentence is
(C) No article
correct'.
(D) an
Copra prices "have surged to a record ₹140 a kg"
Ans - C
following lower arrivals of raw coconuts
67.Convert the given direct/ indirect speech to
despite the harvest season having begun in Kerala.
indirect/ direct speech with the help of the
(A) has surged to a record ₹140 a kg
given options.
(B) have surge to a record ₹140 a kg
He said, "I can drive a bike".
(C) have surge to an record ₹140 a kg
(A) He said he could drive a bike.
(D) The given sentence is correct
(B) He said that he will drive a bike.
Ans - D
(C) He said that he could drive bike.
64.Fill in the blanks with suitable Article from the
(D) He said that he could drive a bike.
given alternatives.
Ans - D
Maria Sharapova is __ Russian professional tennis
player. 68.Find the correct passive voice for the given
sentence in active voice from the given
(A) the
alternatives.
(B) a
The cow ate a lot.
(C) an
(A) A lot was being eaten by the cow.
(D) No article
(B) A lot was ate by the cow.
Ans - B
(C) A lot was been eaten by the cow.
65.Replace the underlined phrase grammatically
and conceptually with the help of the given (D) A lot was eaten by the cow.
options. If the given sentence is correct then select Ans - D
the option 'The given sentence is
69.In the following question, one part of the
correct'. sentence may have an error. Find out which
The online platforms "requires a complex set of" part of the sentence has an error and select the
sequential steps. option corresponding to it. (Avoid
(A) require a complex set of punctuation errors)
(B) has required a complex set of (A) Positive body language convey / (B) confidence
and security / (C) in a group
(C) require an complex set of
discussion. / (D) No Error
(D) The given sentence is correct
(A) D
(B) A Ans - D
(C) C 74.Fill in the blank with suitable tense.
(D) B The professor ____ the use of the language to the
class.
Ans - A
(A) has explained
70.Fill in the blank with suitable tense.
(B) explain
He ______ to learn German for one year.
(C) had explain
(A) try
(D) will be explain
(B) trying
Ans - A
(C) has been trying
75.Choose the word which best expresses the
(D) has trying
similar meaning of the given word " ERUDITE ".
Ans - C
(A) Ignorant
71.Choose the word which best expresses the
(B) Common
similar meaning of the given word " DILATE ".
(C) Studious
(A) Diminish
(D) Support
(B) Curtail
Ans - C
(C) Condense
76.Find the word which is correctly spelt from the
(D) Enlarge
given options.
Ans - D
(A) Encaptulate
72.Find the correct active/ passive voice for the
(B) Beautifully
given sentence in passive/ active voice
(C) Detorioration
from the given alternatives.
(D) Contagiuous
Kirthi did not make a meal.
Ans - B
(A) A meal was not made by Kirthi
77.In the following question, one part of the
(B) A meal do not made by Kirthi
sentence may have an error. Find out which
(C) A meal does not made by Kirthi
part of the sentence has an error and select the
(D) A meal was not being made by Kirthi option corresponding to it. (Avoid

Ans - A punctuation errors)

73.Fill in the blank with suitable word from the (A) The Supreme Court over Friday / (B) refused to
given options. / (C) postpone NEET-PG 2022. / (D)

She was _____ about herself but said plenty about No Error
her husband.
(A) D
(A) miniature
(B) B
(B) fat
(C) A
(C) shallow
(D) C
(D) mysterious
Ans - C
78.Fill in the blanks with suitable Preposition from 82.Choose the word which expresses nearly the
the given alternatives. opposite meaning of the given word "
They walked ___ the bridge. TRANSCEND ".
(A) across (A) Surrender
(B) since (B) Excel
(C) for (C) Overstep
(D) about (D) Best
Ans - A Ans - A
79.Fill in the blank with suitable tense. 83.From the given alternatives, choose the one
that best expresses the meaning of the
My cousin's mother ____ a bike for him.
given idiom/phrase.
(A) buy
Hit the sack.
(B) will bought
(A) Feeling slightly ill
(C) will buying
(B) To present a counter argument
(D) will buy
(C) To go to bed
Ans - D
(D) All the advantages
80.Convert the given direct/ indirect speech to
indirect/ direct speech with the help of the Ans - C
given options. 84.Choose the best option from the given
alternatives which can be substituted for the
“Many happy returns of your birthday", they said
given word/sentence.
(A) They wished him many happy returns of your
birthday. That which lasts forever
(B) They wished him many happy returns of his (A) Eternal
birthday.
(B) Fugitive
(C) They wished him many happy returns of their
(C) Cannibal
birthday.
(D) Internal
(D) They wish him many happy returns of his
birthday. Ans - A
Ans - B 85.In the following question, one part of the
sentence may have an error. Find out which
81.Fill in the blank with suitable tense.
part of the sentence has an error and select the
Tomorrow we ___ for Spain.
option corresponding to it. (Avoid
(A) leaves
punctuation errors)
(B) will leaving
(A) Tomato flu is a unidentified fever / (B) that has
(C) have leave been detected / (C) in several parts of
(D) are leaving Kerala. / (D) No Error
Ans - D (A) A
(B) C
(C) B Ans - B
(D) D 4.Replace the underlined phrase grammatically and
Ans - A conceptually with the help of the given

options. If the given sentence is correct then select


the option 'The given sentence is
1.Find the correct active/ passive voice for the
given sentence in passive/ active voice correct'.

from the given alternatives. We "will playing" cricket next Monday with our
team.
My wallet had been lost by me
(A) will have playing
(A) I lost my wallet
(B) have played
(B) I have been lost my wallet
(C) will be playing
(C) I have lost my wallet
(D) The given sentence is correct
(D) I had lost my wallet
Ans - C
Ans - D
5.Convert the given direct/ indirect speech to
2.Fill in the blanks with suitable Article from the
indirect/ direct speech with the help of the
given alternatives.
given options.
We are trying to give __ incredible performance in
the farewell day function. The traffic incharge asked the man not to park his
car there.
(A) a
(A) The traffic incharge told the man, "Does not
(B) the
park your car here".
(C) an
(B) The traffic incharge told to the man, "Do not
(D) No article park your car here".

Ans - C (C) The traffic incharge told the man, "Do not park
your car here".
3.Convert the given direct/ indirect speech to
indirect/ direct speech with the help of the (D) The traffic incharge told the man, "Do not park
your car there".
given options.
Ans - C
Roja ordered her servant to bring her a cup of milk.
6.Replace the underlined phrase grammatically and
(A) Roja said her servant, "Bring me a cup of milk."
conceptually with the help of the given
(B) Roja said to her servant, "Bring me a cup of
options. If the given sentence is correct then select
milk."
the option 'The given sentence is
(C) Roja told to her servant, "Bring me a cup of
correct'.
milk."
The face value "of fake currency in Indian rupees
(D) Roja begged her servant, "Bring me a cup of
was" around Rs 87 crore, as per the
milk."
preliminary estimate.
(A) of fake currency in Indian rupees were 10.Fill in the blanks with suitable Article from the
given alternatives.
(B) of fake currency above Indian rupees was
The chief selector said that the former skipper
(C) of fake currency above Indian rupees were
remains _____ best wicket keeper and finisher in
(D) The given sentence is correct the game.

Ans - D (A) No article


7.Find the word which is correctly spelt from the (B) the
given options.
(C) an
(A) Paraphrese
(D) a
(B) Resollution
Ans - B
(C) Uniformmity
11.Replace the underlined phrase grammatically
(D) Receptive and conceptually with the help of the given

Ans - D options. If the given sentence is correct then select


the option 'The given sentence is
8.Choose the best option from the given
alternatives which can be substituted for the correct'.

given word/sentence. Residing primarily on Western Australia’s Rottnest


Island, Quokkas "loves for sleep and
A person who has changed his faith
are the happiest of" all animals.
(A) Chauvinist
(A) loves to sleep and is the happiest of
(B) Bohemian
(B) love to sleep and are the happiest of
(C) Apostate
(C) love to sleep and is the happiest of
(D) Connoisseur
(D) The given sentence is correct
Ans - C
Ans - B
9.Choose the best option from the given
alternatives which can be substituted for the 12.Fill in the blanks with suitable Article from the
given alternatives.
given word/sentence.
__ bread is the predominant food prepared from a
A person who regards the whole world as his
dough of flour and water.
country
(A) No article
(A) Invocation
(B) the
(B) Obsolete
(C) an
(C) Cosmopolitan
(D) a
(D) Veteran
Ans - A
Ans - C
13.Fill in the blank with suitable verb from the
given options.
My evening workout plan ___ of running for fifteen (A) Gudi Padwa or the Marathi New Year is / (B)
consecutive minutes celebrate on the first day / (C) of the

without stopping to rest. month of Chaitra. / (D) No Error

(A) produces (A) B

(B) consists (B) C

(C) controls (C) D

(D) contains (D) A

Ans - B Ans - A

14.Choose the word which expresses nearly the 17. Fill in the blank with suitable tense.
opposite meaning of the given word "
My team _____ volleyball for six years.
IMBECILE ".
(A) playing
(A) Smart
(B) have played
(B) Inane
(C) have been playing
(C) Dull
(D) has been playing
(D) Thick
Ans - D
Ans - A
18.Convert the given direct/ indirect speech to
15.Fill in the blanks with suitable Preposition from indirect/ direct speech with the help of the
the given alternatives.
given options.
India’s new data localisation provisions make it
The chief justice commanded them to call the
mandatory ___ foreign
accused in the court.
companies to store personal data of their users and
(A) The chief justice said to her, "Call the accused in
customers in India within the Indian
the court".
territory.
(B) The chief justice said to them, "Please Call the
(A) of accused in the court".

(B) about (C) The chief justice said them, "Call the accused in
the court".
(C) behind
(D) The chief justice said to them, "Call the accused
(D) for
in the court".
Ans - D
Ans - D
16.In the following question, one part of the
19.Replace the underlined phrase grammatically
sentence may have an error. Find out which
and conceptually with the help of the given
part of the sentence has an error and select the
options. If the given sentence is correct then select
option corresponding to it. (Avoid
the option 'The given sentence is
punctuation errors)
correct'.
The engaging show "were set in a village where" (C) The professor said that Gandhiji is born in India.
Jeetendra’s character has started his
(D) The professor said that Gandhiji has been born
first government job. in India.

(A) was sets in a village where Ans - B

(B) was set in a village whom 23.Fill in the blank with suitable word from the
given options.
(C) was set in a village where
The entertainment industry has _____ in the
(D) The given sentence is correct
recent decade.
Ans - C
(A) thrived
20.Choose the word which expresses nearly the
(B) vowelled
opposite meaning of the given word "
(C) retarded
BESET ".
(D) mauled
(A) Nudge
Ans - A
(B) Harass
24.From the given alternatives, choose the one
(C) Invade
that best expresses the meaning of the
(D) Clarify
given idiom/phrase.
Ans - D
Pipe dream.
21.Fill in the blank with suitable word from the
(A) Having a tendency to something
given options.
(B) An impossibility
The institute provides highly _____ computer
systems. (C) A motivational speech

(A) beloved (D) A complex, tricky, or odd person

(B) sophisticated Ans - B

(C) noticed 25.Replace the underlined phrase grammatically


and conceptually with the help of the given
(D) alleged
options. If the given sentence is correct then select
Ans - B
the option 'The given sentence is
22.Convert the given direct/ indirect speech to
correct'.
indirect/ direct speech with the help of the
The best indicators of a university’s performance
given options.
"is the learning outcomes and how" its
The professor said, “Gandhi ji was born in India."
education has impacted the students and society.
(A) The professor ordered that Gandhiji was born in
(A) is the learning outcomes and what
India.
(B) was the learning outcomes and how
(B) The professor said that Gandhiji was born in
India. (C) are the learning outcomes and how
(D) The given sentence is correct (C) Psephologist

Ans - C (D) Egotist

26.Fill in the blank with suitable tense. Ans - D

I ___ living in that city for seven years before I 30.Find the correct active/ passive voice for the
moved to Sydney. given sentence in passive/ active voice

(A) have been from the given alternatives.

(B) will have been How much do you like your sister?

(C) am (A) How much does your sister like by you?

(D) had been (B) How much does your sister liked by you?

Ans - D (C) How much is your sister liked by you?

27.Choose the word which best expresses the (D) How much is your sister like by you?
similar meaning of the given word "
Ans - C
STAGGER ".
31.Find the word which is correctly spelt from the
(A) Peak given options.

(B) Continue (A) Jubilent

(C) Shake (B) Ephimeral

(D) Steady (C) Unaffected

Ans - C (D) Garulous

28.Choose the word which expresses nearly the Ans - C


opposite meaning of the given word "
32.Choose the best option from the given
EFFEMINATE ". alternatives which can be substituted for the

(A) Epicene given word/sentence.

(B) Sissyish One who is not easily pleased by anything

(C) Womanlike (A) Anonymous

(D) Manly (B) Fastidious

Ans - D (C) Insolation

29.Choose the best option from the given (D) Renovation


alternatives which can be substituted for the
Ans - B
given word/sentence.
33.Convert the given direct/ indirect speech to
One who often talks of his achievements indirect/ direct speech with the help of the

(A) Reciprocate given options.

(B) Funambulist They said, "Let us come in".


(A) They requested that they might be allowed to 37.Choose the word which expresses nearly the
come in. opposite meaning of the given word "

(B) They requested they might be allowed to come SLENDER ".


in.
(A) Fragile
(C) They requested that they might being allowed
(B) Spare
to come in.
(C) Heavy
(D) They requested that they might been allowed
to come in. (D) Narrow

Ans - B Ans - C

34.Fill in the blank with suitable tense. 38.Replace the underlined phrase grammatically
and conceptually with the help of the given
The product manager _____ the target of
production for this month. options. If the given sentence is correct then select
the option 'The given sentence is
(A) has successfully achieved
correct'.
(B) had successfully achieves
The Academy "announced that the best picture
(C) have successfully achieved
category" will now be fixed at 10 films,
(D) have successfully achieves
rather than the fluctuating number of nominations.
Ans - A
(A) announce that the best picture category will
35.Find the word which is correctly spelt from the
(B) announce which the best picture category will
given options.
(C) announced that the best picture category need
(A) Temporal
to
(B) Trauema
(D) The given sentence is correct
(C) Underwritte
Ans - D
(D) Santuary
39.Find the correct active/ passive voice for the
Ans - A given sentence in passive/ active voice

36.Choose the word which best expresses the from the given alternatives.
similar meaning of the given word "
Which movie did you see last day?
PERTURB ".
(A) Which movie was saw by you last day?
(A) Soothe
(B) Which movie was seen by you last day?
(B) Confuse
(C) Which movie was being seen by you last day?
(C) Aid
(D) Which movie was been seen by you last day?
(D) Organize
Ans - B
Ans - B
40.From the given alternatives, choose the one
that best expresses the meaning of the
given idiom/phrase. (A) The festival are celebrated / (B) with great
enthusiasm and observation / (C) of full fledged
A blessing in disguise.
ritual. / (D) No Error
(A) To be under attack
(A) B
(B) Things are going well so far
(B) C
(C) Good outcome from evil situation
(C) A
(D) An ideal situation
(D) D
Ans - C
Ans - C
41.Convert the given direct/ indirect speech to
44.Fill in the blanks with suitable Preposition from
indirect/ direct speech with the help of the
the given alternatives.
given options.
A tablespoonful __ peanut butter contains almost
Janaki said to Mona, "Why are you sketching on 100 calories.
the paper?".
(A) to
(A) Janaki said Mona why she was sketching on the
(B) between
paper.
(C) since
(B) Janaki ordered Mona why she was sketching on
the paper. (D) of

(C) Janaki asked Mona why she is sketching on the Ans - D


paper.
45.Choose the best option from the given
(D) Janaki asked Mona why she was sketching on alternatives which can be substituted for the
the paper.
given word/sentence.
Ans - D
Person who eats too much
42.Choose the word which expresses nearly the
(A) Octogenarian
opposite meaning of the given word "
(B) Pessimist
BLISS ".
(C) Predator
(A) Rapture
(D) Glutton
(B) Paradise
Ans - D
(C) Sorrow
46.In the following question, one part of the
(D) Happiness
sentence may have an error. Find out which
Ans - C
part of the sentence has an error and select the
43.In the following question, one part of the option corresponding to it. (Avoid
sentence may have an error. Find out which
punctuation errors)
part of the sentence has an error and select the
(A) Mother’s Day is all about celebrating / (B) the
option corresponding to it. (Avoid
more important woman / (C) in the
punctuation errors)
world who raised you and shaped you. / (D) No 50.Fill in the blank with suitable tense.
Error
Our relatives _____ a simple life.
(A) D
(A) lead
(B) A
(B) have leads
(C) B
(C) have leading
(D) C
(D) leads
Ans - C
Ans - A
47.From the given alternatives, choose the one
51.In the following question, one part of the
that best expresses the meaning of the
sentence may have an error. Find out which
given idiom/phrase.
part of the sentence has an error and select the
A rough diamond. option corresponding to it. (Avoid

(A) A job, task or other activity that is easy or punctuation errors)


simple
(A) Mother's Day celebration is all about honoring
(B) Would never like to do something your mother, / (B) her motherhood and

(C) Happens very rarely the / (C) most special role of mothers in our lives. /
(D) No Error
(D) A person of exceptional character
(A) D
Ans - D
(B) C
48.Find the word which is correctly spelt from the
given options. (C) A

(A) Inadible (D) B

(B) Qualified Ans - A

(C) Reprimend 52.Find the word which is correctly spelt from the
given options.
(D) Infalible
(A) Etternal
Ans - B
(B) Mitigate
49.Fill in the blank with suitable word from the
given options. (C) Grievuous

The fifteen people arrested were still in police (D) Ordinence


____.
Ans - B
(A) affection
53.Convert the given direct/ indirect speech to
(B) reduction indirect/ direct speech with the help of the

(C) insulation given options.

(D) detention Everybody said, "How well he sings!"

Ans - D (A) Everybody exclaimed that he sung very well.


(B) Everybody exclaimed that he had sung very (A) That's true, I agree
well.
(B) To betray a partner
(C) Everybody exclaimed that he sings very well.
(C) Sick
(D) Everybody exclaimed that he sang very well.
(D) To get upset
Ans - D
Ans - B
54.Fill in the blank with suitable word from the
58.In the following question, one part of the
given options.
sentence may have an error. Find out which
Stress may act as a _____ for these illnesses.
part of the sentence has an error and select the
(A) exercise option corresponding to it. (Avoid

(B) failure punctuation errors)

(C) trigger (A) During Ramadan, Muslims across the globe / (B)
abstain from eating, drinking or
(D) attempt
smoking / (C) for the entire month from morning
Ans - C
and evening. / (D) No Error
55.Choose the word which best expresses the
(A) B
similar meaning of the given word "
(B) D
PLUNGE ".
(C) C
(A) Rough
(D) A
(B) Rise
Ans - C
(C) Ascent
59.Convert the given direct/ indirect speech to
(D) Fall
indirect/ direct speech with the help of the
Ans - D
given options.
56.Find the word which is correctly spelt from the
"Stand at ease", The Commander said to his men.
given options.
(A) The Commander asked his men to stand at
(A) Negligible
ease.
(B) Jugernaut
(B) The Commander said his men to stand at ease.
(C) Miscariage
(C) The Commander commanded his men to stand
(D) Immuenity at ease.

Ans - A (D) The Commander requested his men to stand at


ease.
57.From the given alternatives, choose the one
that best expresses the meaning of the Ans - C

given idiom/phrase. 60.Fill in the blanks with suitable Preposition from


the given alternatives.
Throw under the bus.
The rumour spread all ____ the town.
(A) for (A) win

(B) on (B) have won

(C) across (C) had won

(D) since (D) wins

Ans - C Ans - C

61.Choose the word which best expresses the 65.Fill in the blanks with suitable Preposition from
similar meaning of the given word " the given alternatives.

ENCUMBRANCE ". The key chain is _____ the paper.

(A) Burden (A) beneath

(B) Laziness (B) since

(C) Advantage (C) till

(D) Support (D) about

Ans - A Ans - A

62.Fill in the blank with suitable word from the 66.Find the correct active/ passive voice for the
given options. given sentence in passive/ active voice

Beyond all, the local scenery is ______. from the given alternatives.

(A) practicing The first gramophone was invented by Edison.

(B) offering (A) Edison has been invented the first gramophone

(C) enchanting (B) Edison has invented the first gramophone

(D) drafting (C) Edison will have been invented the first
gramophone
Ans - C
(D) Edison invented the first gramophone
63.From the given alternatives, choose the one
that best expresses the meaning of the Ans - D

given idiom/phrase. 67.Fill in the blanks with suitable Preposition from


the given alternatives.
Under the weather.
The book is _____ the table.
(A) An ideal situation
(A) across
(B) Make a problem worse
(B) about
(C) Sick
(C) through
(D) Calm down
(D) on
Ans - C
Ans - D
64.Fill in the blank with suitable tense.

She _____ six matches until that month.


68.From the given alternatives, choose the one (A) The legendary all-rounder recalled how / (B)
that best expresses the meaning of the Rohit had impress him / (C) with his

given idiom/phrase. leadership skills. / (D) No Error

Rabbit hole. (A) D

(A) Understand something complicated (B) B

(B) To joke with someone (C) C

(C) Enter a chaotic or problematic situation (D) A

(D) My patience has run out Ans - B

Ans - C 72.Replace the underlined phrase grammatically


and conceptually with the help of the given
69.Fill in the blank with suitable word from the
given options. options. If the given sentence is correct then select
the option 'The given sentence is
Shyamala ____ her summer income by painting
houses. correct'.

(A) augmented Hiromu Inada "from Japan already hold the title of
a" world’s oldest Ironman.
(B) allowed
(A) from Japan already holds the title of the
(C) aliened
(B) for Japan already holds the title of the
(D) chaired
(C) from Japan already hold the title of an
Ans - A
(D) The given sentence is correct
70.Fill in the blanks with suitable Article from the
given alternatives. Ans - A

Our college has set up ____ counselling centre and 73.Choose the word which best expresses the
a student welfare group similar meaning of the given word "

that helps students to cope up with the stress. SUBSTANTIAL ".

(A) the (A) Miniature

(B) an (B) Selfish

(C) No article (C) Insignificant

(D) a (D) Important

Ans - D Ans - D

71.In the following question, one part of the 74.Fill in the blanks with suitable Article from the
sentence may have an error. Find out which given alternatives.

part of the sentence has an error and select the My friend was _____ FBI agent in his early age.
option corresponding to it. (Avoid
(A) the
punctuation errors)
(B) a
(C) No article Ans - C

(D) an 78.Find the correct active/ passive voice for the


given sentence in passive/ active voice
Ans - D
from the given alternatives.
75.Convert the given direct/ indirect speech to
indirect/ direct speech with the help of the You should obey your teachers.

given options. (A) Your teachers should being obeyed by you

He said to his friend, "I know where everyone is" (B) Your teachers should be obeyed by you

(A) He told his friend that he knew where was (C) Your teachers be obeyed by you
everyone.
(D) Your teachers should been obeyed by you
(B) He told us friend that he knew where was
Ans - B
everyone.
Fill in the blanks with suitable Preposition from the
(C) He narrated his friend that he knew where was
given alternatives.
everyone.
I saw him walking ____ the street.
(D) He told him friend that he knew where is
everyone. (A) at

Ans - A (B) across

76.Convert the given direct/ indirect speech to (C) with


indirect/ direct speech with the help of the
(D) for
given options.
Ans - B
He said to me, "What can I do for you?”
79.Find the word which is correctly spelt from the
(A) He asked me what he can do for me. given options.

(B) He asked me what he could do for me. (A) Documant

(C) He asked me what he could done for me. (B) Imaterial

(D) He begged me what he could do for me. (C) Emanicipate

Ans - B (D) Fragile

77.Fill in the blanks with suitable Article from the Ans - D


given alternatives.
80.From the given alternatives, choose the one
I experienced _____ euphoric feeling after winning that best expresses the meaning of the
the game.
given idiom/phrase.
(A) the
Low hanging fruit.
(B) an
(A) Something that is easy to take advantage of
(C) a
(B) Something or someone that is very impressive
(D) No article
(C) To be defeated by someone
(D) Go insane or crazy 85.Choose the best option from the given
alternatives which can be substituted for the
Ans - A
given word/sentence.
82.Choose the best option from the given
alternatives which can be substituted for the Constant efforts to achieve something

given word/sentence. (A) Dormitory

An event causing sudden damage or suffering (B) Reservoir

(A) Ephemeral (C) Perseverance

(B) Catastrophe (D) Wardrobe

(C) Nostalgia Ans - C

(D) Chronology

Ans - B OPRB SI 2018

83.Fill in the blanks with suitable Preposition from 11 Select the synonym of elate
the given alternatives. A. faze
Centre approves the construction of buds ___ B. enervate
Kaleswaram project.
C. abase
(A) about D. exhilarate
(B) by Ans - D
(C) under 12 Select the antonym of to placate

(D) since A. to propitiate


B. to tranquilize
Ans - C
C. to mollify
84.Find the correct active/ passive voice for the
given sentence in passive/ active voice D. to provoke
Ans - D
from the given alternatives.
13 Select the antonym of obligation
I will present my concepts at the meeting tonight
A. commitment
(A) My concepts will be present at the meeting
B. bond
tonight
C. burden
(B) My concepts will being presented at the
D. irresponsibility
meeting tonight
Ans - D
(C) My concepts will been presented at the
meeting tonight 14 Select the antonym of ''to reveal''
A. to concede
(D) My concepts will be presented at the meeting
tonight B. to repudiate

Ans - D C. to divulge
D. to avow
Ans - B D. to apprehend
15 Select the synonym of batter Ans - A
A. insulate 21 Select the synonym of eccentric
B. assure A. chronic
C. bulwark B. bona fide
D. bash C. whimsical
Ans - D D. typic
16 Select the antonym of escort Ans - C
A. entourage 22 Select the antonym of steadfast
B. foe A. staunch
C. convoy B. rigid
D. cavalier C. ardent
Ans - B D. pliant
17 Select the synonym of stash Ans - D
A. cache 23 Select the antonym of ambushed
B. deficit A. Exposed
C. lack B. lurk
D. dearth C. Waylay
Ans - A D. Camouflage
18 Choose the synonym of to thrash Ans - A
A. to grasp 24 Select the synonym of ''bounder''
B. to pin A. mediocre
C. to pinch B. decorous
D. to beat up C. lout
Ans - D D. done
19 Select the antonym of to pilfer Ans - C
A. to filch 25 Select the antonym of horrify
B. to bestow A. appall
C. to purloin B. soothe
D. appropriate C. petrify
Ans - B D. affright
20 Select the antonym of to envisage Ans - B
A. oblivious 26 Select the synonym of rot
B. to confront A. stagnate
C. to visualize B. smell
C. mature A. scant
D. decay B. bereft
Ans - D C. adequate
27 Select the synonym of defuse D. destitute
A. alleviate Ans - C
B. agitate 33 Select the antonym of coalesce
C. incite A. adhere
D. irritate B. cleave
C. amalgamate
Ans - A D. separate
28 Select the antonym of astringe Ans - D
A. constringe 34 Select the synonym of discerning
B. liberate A. astute
C. tauten B. inept
D. circumscribe C. asinine
Ans - B D. naïve
29 Select the synonym of perusal Ans - A
A. delinquency 35 Select the synonym of famished
B. inconsideration A. jaded
C. inspection B. glut
D. laxity C. starved
Ans - C D. satiated
30 Select the antonym of conception Ans - C
A. clue 36 Choose the antonym of analogue
B. notion A. difference
C. cognition B. dialogue
D. fact C. correlate
Ans - D D. cognate
31 Select the synonym of astonish Ans - A
A. mundane 37 Select the synonym of bother
B. overwhelm A. obfuscate
C. calm B. beset
D. empress C. oblivious
Ans - B D. obscene
32 Select the antonym of void Ans - B
38 Select the synonym of thrashing D. pagan
A. prefabrication Ans - B
B. edifice 44 Select the antonym of cosset
C. bashing A. close
D. contour B. fondle
C. caress
Ans - C D. ignore
39 Select the antonym of violation Ans - D
A. felony 45 Select the synonym of deviance
B. obedience A. compliance
C. crime B. aberrance
D. abuse C. alliance
Ans - B D. accordance
40 Select the antonym of 'to actuate' Ans - B
A. to retard 46 Select the synonym of splinter
B. to impel A. chip
C. to spur B. stub
D. to propel C. share
Ans - A D. friction
41 Select the synonym of to recline Ans - A
A. to rewind 47 Select the antonym of veracity
B. to plump A. condor
C. to sheer B. probity
D. to lounge C. deceit
Ans - D D. rectitude
42 Select the synonym of incisive Ans - C
A. stolid 48 Select the synonym of improvident
B. daffy A. provident
C. concise B. thrifty
D. tedious C. miserly
Ans - C D. extravagant
43 Select the antonym of atheist Ans - D
A. skeptic 49 Select the antonym of to jostle
B. disciple A. to shove
C. agnostic B. to eschew
C. to hustle C. us
D. to scramble D. has
Ans - B Ans - C
50 Select the synonym of to gnaw 15 Sheeba was born ……………………. the year 1985.
A. to nibble A. in
B. to drool B. on
C. to slaver C. at
D. to spit D. of
Ans - A Ans - A
16 They are building a house next door to our
11 Suganya is typing. school.

A. simple past A. Next door to our school a house is being built by


them
B. simple present
B. Next door to our school is being built a house by
C. present continous them
D. past perfect C. A house next door to our school is being built by
Ans - C them.

12 Choose the option that is opposite in meaning D. A house is being built by them next door to our
to the word -Venal. school.

A. vicious Ans - D

B. corrupt 17 She run very fast ____ she might catch the train.

C. venial A. so

D. noble B. sooner

Ans - D C. because

13 Choose the most appropriate preposition from D. then


the given alternatives. There is still no cure ___ the Ans - A
common cold. 18 A ….....is used as the name of a person, thing,
A. through place, idea and a quality of a person.

B. for A. Noun

C. of B. Verb

D. to C. Adjective

Ans - B D. Pronoun

14 Fill in the Blank : Let…….. start now. Ans - A

A. We 19 Choose the option that is nearest in meaning to


the given word: Insurgent
B. our
A. loyal
B. submission C. (i) near/ outside (ii) in
C. Rebellious D. (i) on (ii) to
D. Patriotic Ans - A
Ans - C
20 Choose the most appropriate prepositions : I 25 Choose the option that is opposite in meaning
met my friend Yami __ the cold and snowy day in to the word- Lavish.
Kashmir. A. opulent
A. at B. grand
B. in C. ornate
C. with D. meagre
D. on Ans - D
Ans - B 26 Choose the most appropriate preposition from
the given alternatives. I have read the book ___.
21 What is the antonym of ABRUPT?
A. by
A. hurried
B. through
B. smooth
C. from
C. sudden
D. at
D. blunt
Ans - B
Ans - B
27 What is a person who does not believe in God
22 The crow _____ and stole the cheese.
called?
A. fly
A. Atheist
B. flew down
B. Ascetic
C. came
C. Lunatic
D. ran
D. Anarchist
Ans - B
Ans - A
23 You must look into this matter.
28 "Hit the sack" means …....................
A. This matter has been looked into by you.
A. Go to bed or go to sleep
B. This matter may be looked into by you.
B. Box the bag
C. This matter should be looked into by you.
C. Hit the potato sack with a stick
D. This matter into looked by you.
D. Throw the sack to the floor
Ans - C
Ans - A
24 (a) Meenal has achieved a lot in her life but she
29 Choose the correct order of the sentence.
cannot be given the credit (i) ……………………. all that
Mahatma Gandhi: withing seven days failing which
she boasts (ii) …………………… .
, (P)/to stop lawlessness and barbarianism (Q)/gave
A. (i) for (ii) about notice to the Government (R)/he would start civil
disobedience movement.(S)
B. (i) of (ii) in
A. RQPS C. tallest
B. SPQR D. none of these
C. RPQS Ans - B
D. SQPR 35 Come here ____ you have to see this!
Ans - A A. quickly
30 Take this medicine and you will get rid -------- B. well
the bad cold
C. carelessly
A. of
D. excitedly
B. over
Ans - A
C. at
36 Directions Each of the following questions
D. from containing main word followed by four words.
Select the word that is most similar in meaning.
Ans - A
Prosaic
31 ……………. sun sets in the west.
A. uninspiring
A. a
B. poetic
B. the
C. exotic
C. an
D. none of these
D. none of these
Ans - A
Ans - B
37 Directions Each of the following main words are
32 Look at the exquisite __ of pearls she is wearing
followed by four words. Select the word that is
round her neck.
furthest (opposite) in meaning to the main word.
A. set
Forbearance
B. ring
A. patience
C. chain
B. self-control
D. string
C. intolerance
Ans - D
D. preference
33 The scouts will cook dinner by __
Ans - C
A. them
38 Can _ lend me _ travel bag ?
B. theirs
A. your,you
C. ourselves
B. yours, you
D. themselves
C. you,your
Ans - D
D. you, yours
34 Rajesh is ____than Mukesh.
Ans - C
A. tall
39 She ………………….. (go) to her office with a friend.
B. taller
A. goes
B. gone A. a day of success
C. going B. a day of merry-making
D. has gone C. an eventful day
Ans - A D. a day of significance
40 Directions Out of the four alternatives, choose Ans - B
the one which can be substituted for the given
44 Directions Some of the following sentences are
words/sentences.The depository where state grammatically correct and some are incorrect. Find
records and documents are preserved. out
A. Museum which part of the sentence has an error and mark
that part. If there is no error mark that part as your
B. Library
answer. Another baffling change (A)/ that I notice
C. Emporium
in him now-a-days (B)/ is that he avoids to speak to
D. Archive me. (C)/No error(D)

Ans - D A. A
41 Directions Each of the following questions B. B
containing main word followed by four words.
C. C
Select the word that is most similar in meaning.
D. D
Solitude
Ans - C
A. musical composition
45 ……………….. boy stole the book. (demonstrative)
B. aloneness
A. It
C. true statement
B. This
D. single mindedness
C. The
Ans - B
D. That
42 Directions Each of the following main words are
followed by four words. Select the word that is Ans - D
farthest (opposite) in meaning to the main word.
46 Use appropriate adverbs of degree according to
Agony
the context of the sentences given below. I am
A. unpleasure
____ tired. But I will definitely try to come with
B. weep you.
C. None of these A. a little / a bit
D. Ecstasy B. very
Ans - D C. extremely
43 Directions In each of the following questions an D. None
idiomatic phrase is given followed by four
Ans - A
alternatives.
47 The cat …………………….. (drink) all the milk.
Choose the alternative that best expresses the
meaning of the expression. A. has drunk
A Gala day B. will drinking
C. drunked A. simple present
D. drinked B. present continous
Ans - A C. past perfect
48 Directions Out of the four alternatives, choose
D. present perfect
the one which can be substituted for the given
words/sentences. Medicine given to counteract Ans - D
poison. 12 Choose the best Active/Passive voice sentence
A. Antibiotic with same meaning as in- Mrs. Priya teaches us
grammar.
B. Antiseptic
C. Antidote A. Grammar is teach to us by Mrs. Priya.

D. Antifungal B. We are taught grammar by Mrs. Priya.

Ans - C C. Grammar is taught by Mrs. Priya.


49 Directions In each of the following questions an D. We are taught by Mrs. Priya's grammar.
idiomatic phrase is given followed by four
alternatives. Ans - C

Choose the alternative that best expresses the 13 Fill in the Blank : All felt that he ……a cheat.
meaning of the expression. Rest on one’s oars
A. was
A. take rest after working long
B. were
B. tired of working
C. had been
C. tired of boating
D. has been
D. tired of boating
Ans - A Ans - D

50 Directions Some of the following sentences are 14 Choose the correct Sentence.
grammatically correct and some are incorrect. Find
A. A dog barking up the tree
out
which part of the sentence has an error and mark B. A mistake made in something you are trying to
that part. If there is no error mark that part as your achieve

answer. Being a sunny day (A)/ I decided to skip C. Talking to the wrong person
(B)/ work and stay at home.(C) No error (D)
D. Yelling too much
A. A
Ans - B
B. B
15 Try not to compromise ……………………. your
C. C
principles.
D. D
A. on
Ans - A
B. in

C. the
11 Shakespeare has written dramas appealing to all
people of all ages . D. there
Ans - A A. Delight

16 I remember my sister taking me to the museum. B. Delighting

A. I remember taken to the museum by my sister. C. Delighted

B. I remember myself being taken to the museum D. Delights


by my sister.
Ans - C
C. I remember I was taken to the museum by my
21 Fill in the blank : The stranger …....in through
sister.
the window last night.
D. I remember being taken to the museum by my
A. creep
sister
B. creeping
Ans - D
C. creeps
17 Choose the option that is nearest in meaning to
the word- Asperse. D. crept

A. defend Ans - D

B. praise 22 Choose the option that is nearest in meaning to


the word-Desex.
C. caluminate
A. banish
D. eulogize
B. cripple
Ans - C
C. maim
18 Find the error in the sentence - "I shall go to
Dehradun tomorrow morning." D. alter

A. I shall go Ans - D

B. to Dehradun 23 Who teaches you English?

C. tomorrow morning A. By whom were you taught English?

D. no error B. By whom are you taught English?

Ans - D C. English is taught by whom?

19 Choose the option that is opposite in meaning D. By whom will you be taught English?
to the given word. : Priggish
Ans - B
A. sensible
24 There is a playground (i) ……………………. our
B. coxcombical house where children play (ii) ……………………. the
evening.
C. prim
A. (i) near/ outside (ii) in
D. unbending
B. (i) for (ii) about
Ans - A
C. (i) of (ii) in
20 Fill in the blank : The boys were …......to hear
that we were going to build a bridge. D. (i) on (ii) to
Ans - A 29 Change the underlined word to improve the
following sentence. They told me that they "had
25 Choose the best Active/Passive voice sentence
already sold"
with same meaning as in. "We have already done
the out all their belongings.

work." A. Has sold already

A. Already the work has been done. B. Have already been sold

B. The work is already done by us. C. Has already sold

C. The work has already been done by us. D. No improvement

D. The work had been done by us already. Ans - D

Ans - C 30 It is dangerous to enter -------- the enemy's


camp.
26 Arrange the jumbled words to form a
meaningful sentence from- A. in
shephered,sheep,lost,found,the,The.
B. on
A. The sheep found the shepherd lost.
C. by
B. Lost sheep found the shepherd.
D. into
C. The Shepherd found the lost sheep.
Ans - D
D. The Shepherd and the lost sheep were found.
31 Ram is ……………….. honest man.
Ans - C
A. an
27 Which spelling is correct?
B. a
A. Acesion
C. the
B. Accesion
D. none of these
C. Acession
Ans - A
D. Accession
32 All the kitchen __ are in that cupboard.
Ans - D
A. tools
28 What is a person who is present everywhere
B. devices
called?
C. utensils
A. Omnivore
D. equipment
B. Omnipotent
Ans - C
C. Omnipresent
33 The team members hugged __ after they had
D. Omniscient
won the competition.
Ans - C
A. all

B. both
C. each other D. restlessness

D. one another Ans - A

Ans - D 38 _ are going to the museum. Would you like to


join _ ?
34 Lisa is wearing a……………………. shirt today.
A. We, us
A. sleeve
B. He, him
B. sleeveless
C. You, you
C. sleepiest
D. They, their
D. none of these
Ans - A
Ans - B
39 My friend ………………….. (visit) his grandmother
35 He ____ put the vase on the table. It fell to the
every day.
floor.
A. visit
A. carelessly
B. visits
B. finally
C. visiting
C. well
D. visited
D. quickly
Ans - B
Ans - A
40 Directions Out of the four alternatives, choose
36 Directions Each of the following questions
the one which can be substituted for the given
containing main word followed by four words.
Select the word words/sentences.A person who lives by himself.

that is most similar in meaning. Itinerant A. Monk

A. frequent use of ‘it’ B. Recluse

B. anything involving repetition C. Extrovert

C. plan for proposed journey D. Prophet

D. travelling from place to place Ans - B

Ans - D 41 Directions Each of the following questions


containing main word followed by four words.
37 Directions Each of the following main words are
Select the word
followed by four words. Select the word that is
furthest that is most similar in meaning. Rectitude

(opposite) in meaning to the main word. A. duplication


Propensity
B. integrity
A. disinclination
C. rectification
B. forecast
D. recovery
C. stagnation
Ans - B C. farther

42 Directions Each of the following main words are D. none


followed by four words. Select the word that is
Ans - C
furthest
46.Use appropriate adverbs of degree according to
(opposite) in meaning to the main word. Timorous
the context of the sentences given below. The boy
A. trembling found

B. cowardly the girl ____ ugly and disagreed to marry her.

C. bright A. extremely

D. bold B. a little / a bit

Ans - D C. very

43 Directions In each of the following questions an D. none


idiomatic phrase is given followed by four
Ans - A
alternatives.
47.She …………………….. (not qualify) the written
Choose the alternative that best expresses the
test.
meaning of the expression. Gird up one’s Loins
A. not qualifying
A. get ready for a job
B. have not qualified
B. leave bag & baggage
C. has not qualified
C. hunt for lions
D. not qualified
D. a loin cloth
Ans - C
Ans - A
48 Directions Out of the four alternatives, choose
44.One of the state in which (A)/ Satyagraha was
the one which can be substituted for the given
offered was Rajkot,(B)/where he had spent his
youth. words/sentences. Violation of the sanctity of the
Church.
(C) /No error (D)
A. Infringement
A. A
B. Irreverence
B. B
C. Sacrilege
C. C
D. Ignorance
D. D
Ans - C
Ans - A
49.Directions In each of the following questions an
45.Amritsar is …………………… from the Tropic of
idiomatic phrase is given followed by four
Cancer than Delhi.
alternatives.
A. latter
Choose the alternative that best expresses the
B. further meaning of the expression. Wash dirty linen in
public
A. to criticise Ans - C

B. make personal quarrels public 14 Provide meaning for the given Idiom/Phrases:

C. to talk dirty things in public "Every Cloud Has A Silver Lining"


A. Be sarcastic
D. to wash clothers
B. Be pessimistic
Ans - B
C. Be optomistic
50.Many a student (A)/ has failed in the
D. None of these
Mathematics test (B)/but Dilip has scored 100 per
cent. (C) Ans - C
15 Don't forget _________ your homework!
/No error (D)
A. do
A. A
B. to do
B. B
C. doing
C. C D. did
D. D Ans - B
Ans - C 16 Identify the material noun : "Does wood sink in
water?"

11 Select the synonym of redemption A. sink

A. penalty B. does

B. retribution C. water

C. atonement D. wood

D. forfeit Ans - D

Ans - C 17 The economic recovery is far from convincing

12 Select the synonym of rip (1)/ and a lot of government action are required to

A. sew consolidate (2)/ the positive signals in different

B. terminate sectors. (3)/ No error. (4)

C. gash A. 1

D. cease B. 2

Ans - C C. 3

13 Provide Idiom/Phrase for the given meaning : D. 4

'To politely ask for someone's full attention' Ans - B

A. Let Sleeping Dogs Lie 18 The boss reminded them of the old saying (1)/

B. Mum's the word that honesty was the best policy (2)/ and told them

C. Lend Me Your Ear that

D. None of these they had better be honest in their work. (3)/ No


error (4)
A. 1 B. played
B. 2 C. plays
C. 3 D. plaing
D. 4 Ans - A
Ans - B 24 Identify the mistake in the sentence and correct
19 Honest men speak ………………. truth. it from the given options. It eeems that none of the
A. a option is true.
B. an A. seems
C. the B. sims
D. None of these C. sins
Ans - C D. seeks
20 Look at …………………….. second chapter of this Ans - A
book. 25 Find the non-finite verb from the given
A. a sentence. Giving the full presence was compulsory.
B. an A. giving
C. the B. presence
D. None of these C. was
Ans - C D. compulsory
21 Present Continuous - Where’s Renu? She Ans - A
is…………………. a shower. 26 Complete the sentence by filling appropriate
A. had non-finite verb. I watched the mild scolding with a
B. having _______
C. has frown.
D. None of these A. worry
Ans - B B. worries
22 Past Perfect Continuous Tense - …………………. he C. worried
been studying for examination for four days? D. to worry
A. had Ans - C
B. have 27 Find the finite verb from the given
C. has sentence.They stood up to salute the unfurling
D. None of these National flag.
Ans - C A. unfurling
23 Identify the mistake in the sentence and correct B. to salute
it from the given options. He was play games. C. stood
A. playing D. unfurling, to salute, stood
Ans - C A. List
28 Find the phrasal verb from the given sentence. B. Memorandum
The car broke down on the highway. C. Note
A. the car D. All of these
B. broke down Ans - B
C. on the highway 33 Give one word for the following : "Parts of your
D. None of these mind that tell you whether your actions are right or
Ans - B wrong."
29 Fill the gap with appropriate phrasal verb or A. Conscience
prepoition in the given sentence. He turned B. awakeness
________ to be a C. sharpness
gangster. D. All of these
A. out Ans - A
B. off 34 Tina is the most intelligent …………... all the
C. on students of the class.
D. All of these A. in
Ans - A B. than
30 Identify the pronoun form of the words quoted - C. of
All of "those" are expensive. D. None of these
A. Personal Ans - C
B. Indefinite 35 He is ………………….. than stupid.
C. Demonstrative A. more lazy
D. Both 1 & 2 B. lazy
Ans - C C. lazier
31 Use pronouns in place of nouns for the words D. many lazy
quoted - "Eric"cleaned the house. Ans - A
A. They 36 I get up …………….. in the morning.
B. We A. slow
C. He B. early
D. None of these C. fast
Ans - C D. None of these
32 Give one word for the following : "Brief written Ans - B
messages or reports from one person or 37 Adverb of …………. say where something
department in a happens.
company or organization to another" A. degree
B. place B. The child were looked after by him.
C. manner C. The child looked after to him.
D. None of these D. The child is looked after by him.
Ans - B Ans - D
38 Convert to indirect speech. I said to him, "Good 43 S1: All the land was covered by the ocean. P :
morning, how do you do". The leading god fought the monster, killed it and
A. I told him good morning and asked him how he chopped
did. its body in to two halves. Q : A terrible monster
B. I wished him good morning and asked him how prevented the gods from separating the land from
he did. the
C. I said to him that good morning, how he did. water. R : The god made the sky out of the upper
D. I said to him good morning and asked him how part of the body and ornamented it with stars. S :
he did. The
Ans - B god created the earth from the lower part, grew
39 Change the speech. He said to me, “Where is plants on it and populated it with animals
Hiya?” A. QPSR
A. He asked me where Hiya is. B. QPRS
B. He asked me where Hiya was. C. PQSR
C. He said where was Hiya. D. PQRS
D. He asked me where is Hiya. Ans - B
Ans - B 44 The key point of a formal letter is written in
40 The doctor had examined him. which part of a formal letter?
A. He had been examined by the doctor. A. Subject
B. He has been examined by the doctor. B. Body
C. None of these C. Intro
D. He was examined by the doctor. D. Heading
Ans - A Ans - B
41 She had cooked some aweets. 45 If you are writing a job application letter but you
A. Some sweets was been cooked by her. havent been given a named contact, how do you
B. Some sweets had been cooked by her. sign of
C. Some sweets has been cooked by her. your letter?
D. Some sweets have been cooked by her. A. Yours faithfully
Ans - B B. Yours sincerely
42 He looks after the child. C. Cheers
A. The child are looked after by him. D. None of these
Ans - A
46 He has a great admiration ……… my talent.
A. for OSSC Junior Executive Assistant 2022
B. of 1. Fill in the blank with suitable word from the
C. in given options.
D. None of these The jury returned a ______ of not guilty.
Ans - A
(A) verdict
47 I was able to know …….. sight.
(B) pursuit
A. on
B. at (C) germ

C. by (D) votary
D. None of these Ans - A
Ans - C
2. Fill in the blanks with suitable Article from the
48 His familiarity ……. the place saved him from a given alternatives.
lot of troubles.
Kathir wants to be ______ doctor.
A. for
(A) a
B. about
C. among (B) the

D. None of these (C) an


Ans - B (D) No article
49 …….. is awesome.
Ans - A
A. That a girl can play a guitar
3. Choose the word which expresses nearly the
B. Which could the girl play a guitar opposite meaning of the given word
C. That can the girl play a guitar
GLOOM".
D. Which the girl could play a guitar
(A) Delight
Ans - A
50 The man …..… we met at the mall yesterday is (B) Grief

the owner of the bookstore near your house. (C) Dullness


A. Whom
(D) Sorrow
B. Who
Ans - A
C. Which
D. Whose 4. Find the word which is correctly spelt from the
given options.
Ans - A
(A) Unfortunately

(B) Sceince
(C) Uncomfortabel Ans - B

(D) Negligant 8. B. I am a sales representative in a sports


company and my brother works as a trainer in
Ans - A
a Cricket coaching academy. I asked my brother to
5. Fill in the blanks with suitable Preposition from introduce (A) to his manager
the given alternatives.
so l can market some of my products in their
We have been living in this palace ___ 1956. academy. When I entered the manager's room, I
thought he was talking to (B) Later I realised he
(A) through was on the phone.
(B) for (A) I
(C) against (B) our
(D) since (C) himself
Ans - D (D) he
6. Choose the option with the correct punctuation Ans - C
for the given sentence.
9. Choose the word which best expresses the
why did the ground suddenly open up similar meaning of the given word
(A) why did the ground ,suddenly open up. DEADLOCK ".
(B) why did the ground suddenly open up! (A) Halt
(C) why did the Ground Suddenly open up! (B) Start
(D) Why did the ground suddenly open up? (C) Agreement
Ans - D (D) Solution
7. A. I am a sales representative in a sports Ans - A
company and my brother works as a trainer in
10. Choose the word which best expresses the
a Cricket coaching academy. I asked my brother to similar meaning ofthe given word
introduce (A) to his manager
CONFRONT
so I can market some of my products in their
academy. When I entered the manager's (A) Challenge

room, I thought he was talking to (B) Later I (B) Yield


realised he was on the phone.
(C) Dodge
(A) my
(D) Avoid
(B) me
Ans - A
(C) myself
11. Choose the word which expresses nearly the
(D) I opposite meaning of the given word
EMBRACE" Ans - D

(A) Free 15. Fill in the blank with suitable tense.

(B) Seize The manager and the Asst. manager ______ when
Ientered the room.
(C) Admit
(A) were talking
(D) Receive
(B) will talk
Ans - A
(C) are talking
12. Find the word which is correctly spelt from the
given options. (D) talked

(A) Belligerance Ans - A

(B) Compatition
Fill in the blank with suitable tense.
(C) Benevolent
1. _____ he go to school every day?
(D) Acommodation (A) Has

Ans - C (B) Does

13. Choose the option with the correct punctuation (C) Do


for the given sentence. (D) Was

he was driving through an indiana neighborhood at Ans - B


1230 am
2. Choose the word which best expresses the
(A) he was driving through an indiana similar meaning of the given word
neighborhood at 12:30 am. ALLIANCE"

(B) he was driving through an indiana (A) Divorce


neighborhood at 12:30 am?
(B) Hostility
(C) He was driving through an Indiana (C) Separation
neighborhood at 12:30 a.m.
(D) Bond
(D) He was driving, through an indiana
Ans - D
neighborhood at 12:30 am!
3. Fill in the blank with suitable tense.
Ans - C
The new bridge _______ two days ago.
14. Fill in the blank with suitable tense.
(A) have been constructed
Have you _______ to the minister? (B) was constructed

(A) speaking (C) constructed

(B) spoke (D) is being constructed


Ans - B
(C) speaks
4. Find the word which is correctly spelt from the
(D) spoken given options.
(A) Necesary (A) But hunters wanted them for their fur, meat,
and a fluid.
(B) Psychiatist
(B) but hunters wanted them for their fur, meat
(C) Recommend
and a fluid
(D) Mysteriuous
(C) But hunters wanted them for their fur meat and
Ans - C a fluid.

5. Choose the option with the correct punctuation (D) but hunters wanted them for their fur, meat
for the given sentence. and a fluid

larger animals include turtles fish crabs and sea Ans - A


cows
9. Choose the word which expresses nearly the
(A) larger animals include turtles fish crabs and sea opposite meaning of the given word
cows?
SWINDLED".
(B) Larger animals include turtles fish crabs and sea
(A) Deceived
cows!
(B) Beat
(C) larger animals include turtles fish, crabs, and
sea cows (C) Donated
(D) Larger animals include turtles, fish, crabs, and (D) Victimized
sea cows.
Ans - C
Ans - D
10. Choose the word which best expresses the
6. Choose the word which expresses nearly the similar meaning of the given word " GLEAM".
opposite meaning of the given word
(A) Shine
CONCISE ".
(B) Spank
(A) Compact
(C) Veil
(B) Lengthy
(D) Darkness
(C) Lean
Ans - A
(D) Summary
16. Fill in the blank with suitable word from the
Ans - B given options.
7. Find the word which is correctly spelt from the At last, he sold his cars to pay off his _____
given options.
(A) scuffles
(A) Bankcrupt
(B) demonstratorss
(B) Formiddable
(C) visitors
(C) Contentious
(D) creditors
(D) Endaevour
Ans - D
Ans - C
17. Fill in the blanks with suitable Article from the
8. Choose the option with the correct punctuation given alternatives.
for the given sentence.
Tina's mother is sensitive to ______ cigarette
but hunters wanted them for their fur meat and a smoke.
fluid
(A) a
(B) an (B) with
(C) the (C) around
(D) No article (D) Over
Ans - D Ans - B
18. Directions: In the passage given below there 1. Fill in the blanks with suitable Preposition from
are two blanks, each followed by four alternatives.
the given alternatives.
Select the appropriate pronoun which fills the
India’s new data localisation provisions make it
blank correct.
mandatory ___________ foreign
A. Once in a school, students began to scribble on
the class room walls. The janitor was companies to store personal data of their users and

annoyed by _____(A)_____ since he needed to customers in India within the Indian


clean that and it was hard for him to do so. territory.
Hence the Janitor requested young people to stop (A) about
doing(B) by leaving small
(B) for
notes with colorful pictures explaining his difficulty.
(C) of
(A) she
(D) behind
(B) they
Ans - B
(C) it
2. Fill in the blanks with suitable Article from the
(D) he
given alternatives.
Ans - C
I am ____________ oldest in my family.
19. B. Once in a school, students began to scribble
(A) the
on the class room walls. The janitor was
(B) a
Annoyed by _____(A)____ since he needed to
clean that and it was hard for him to do so. (C) an
Hence the Janitor requested young people to stop (D) No article
doing _____(B)_____ by leaving small Ans - A
notes with colorful pictures explaining his difficulty. 3. Choose the option with the correct punctuation
(A) that for the given sentence.
(B) them Is geetha your best friend
(C) which (A) Is geetha your Best Friend?
(D) what (B) Is geetha your Best friend?
Ans - A (C) Is geetha your best friend?
20. Fill in the blanks with suitable Preposition from (D) Is Geetha your best friend?
the given alternatives.
Ans - D
Study identifies autoimmune disease associated
4. Choose the word which expresses nearly the
_____ testicular cancer.
opposite meaning of the given word
(A) for
"PLAGUED ".
(A) Pursued 14. Find the word which is correctly spelt from the
(B) troubled given options.
(C) Comforted (A) Subtantive
(D) Afflicted (B) Retaliation
Ans - C (C) Viablity
5. Choose the word which best expresses the (D) Testimoni
similar meaning of the given word Ans - B
" BIZARRE ". 15. Choose the option with the correct punctuation
(A) Common for the given sentence.
(B) Abundant we went to london for a holiday
(C) Brief (A) we went to London for a holiday.
(D) Strange (B) We went to London for a holiday.
Ans - D (C) "we went to london for a holiday"
6. Fill in the blank with suitable tense. (D) "we went to london for a holiday."
He_________ the talk of the town. Ans - B
(A) has had 16. Choose the word which expresses nearly the
(B) have been opposite meaning of the given word
(C) had been "BANISHED ".
(D) have had (A) Accepted
Ans - C (B) Exiled
7. Fill in the blank with suitable word from the (C) Discarded
given options. (D) Evicted
Snow caused ____________ in the region. Ans - A
(A) bribery 17. A. Rajeev received a letter from his Uncle Suraj,
(B) chaos invited ____(A)____ to visit the radio
(C) substantial station where he works. I was pleased when Rajeev
(D) litigation asked me to go with him as his
Ans - B guest. It was an exciting visit. A guide met
8. Fill in the blank with suitable tense. ____(B)____ at the spot and we were taken
He has ________ his leg. to a comfortable place where engineers showed us
(A) hurted how programmes go on air.
(B) been hurt (A) he
(C) hurt (B) me
(D) been hurted (C) his
Ans - C (D) himself
Ans - B (D) Feesibility
18. B. Rajeev received a letter from his Uncle Suraj,
Ans - C
invited ____(A)____ to visit the radio
2. Choose the option with the correct punctuation
station where he works. I was pleased when Rajeev
asked me to go with him as his for the given sentence.
guest. It was an exciting visit. A guide met
she says you dont become a pianist until youre
____(B)____ at the spot and we were taken
past the age of 60
to a comfortable place where engineers showed us
how programmes go on air. (A) She says, “You don’t become a pianist until
(A) we
you’re past the age of 60.”
(B) our
(B) She says, You don’t become a pianist until
(C) us
(D) he you’re past the age of 60
Ans - C
(C) She says You don’t become a pianist until
19. Find the word which is correctly spelt from the
you’re past the age of 60.”
given options.
(A) Nusance (D) She says You don’t become a pianist until
(B) Improvement
you’re past the age of 60
(C) Iresponsible
Ans - A
(D) Slauhter
Ans - B 3. Fill in the blanks with suitable Preposition from
20. Choose the word which best expresses the
the given alternatives.
similar meaning of the given word " DETAIN ".
They are living _______________ the temple.
(A) Reserve
(B) Allow (A) behind
(C) Advance
(B) before
(D) Liberate
(C) with
Ans - A
1. Find the word which is correctly spelt from the (D) over

given options. Ans - A

(A) Coinside 4. Choose the option with the correct punctuation

(B) Analoguous for the given sentence.

(C) Eminent most people play and watch cricket in india


(A) Most people play and watch cricket in India. (C) Strong

(Correct Answer) (Chosen option) (D) Steady

(B) most people play and watch cricket in india. Ans - B

(C) Most people play and watch cricket in india 8. Find the word which is correctly spelt from the

(D) most people play and watch cricket in India given options.

Ans - A (A) Emphathic

5. Fill in the blank with suitable word from the (B) Extradition

given options. (C) Annarchy

I am looking for a/an _______________ answer. (D) Bizare

(A) aisle Ans - B

(B) affirmative 9. Choose the word which best expresses the

(C) pecuniary similar meaning of the given word "

(D) cushion IMPRUDENT ".

Ans - B (A) Careful

6. Fill in the blank with suitable tense. (B) Careless

He __________ met my mother yet. (C) Prudent

(A) has (D) Mindful

(B) haven't Ans - B

(C) did 10. Choose the word which best expresses the

(D) hasn't similar meaning of the given word "

Ans - D SHALLOW ".

7. Choose the word which expresses nearly the (A) Empty

opposite meaning of the given word " (B) Full

SUBSTANTIAL ". (C) Start

(A) Considerable (D) Major

(B) Broken Ans - A


11. A. Our neighbors and _____(A)_____ cleaned 13. Fill in the blank with suitable tense.

up a vacant lot on our street. One of the We __________ to the park every day.

ladies decided that we ladies should prepare lunch (A) will go

for everyone. Marie and I brought a (B) goes

salad and desserts. Frank and Jane brought ham (C) go

and chicken. The other ladies brought (D) are going

many different kinds of food. No people have ever Ans - C

worked as hard as ____(B)____ did. 14. Fill in the blanks with suitable Article from the

(A) us given alternatives.

(B) our The Unicorn is _______ fabulous animal.

(C) them (A) a

(D) we (B) No article

Ans - D (C) an

12. B. Our neighbors and _____(A)_____ cleaned (D) the

up a vacant lot on our street. One of the Ans - A

ladies decided that we ladies should prepare lunch 15. Choose the word which expresses nearly the

for everyone. Marie and I brought a opposite meaning of the given word "

salad and desserts. Frank and Jane brought ham MISCHIEVOUS ".

and chicken. The other ladies brought (A) Insidious

many different kinds of food. No people have ever (B) Obedient

worked as hard as ____(B)____did. (C) Destructive

(A) them (D) Risky

(B) our Ans - B

(C) we 1. Fill in the blank with suitable word from the


given options.
(D) us
He pleaded guilty to ______________ a police
officer.
Ans - C
(A) clinging
(B) assaulting 6. A. We couples should realize that disagreements
must occur between husband and
(C) opening
wife. I believe that a healthy quarrel is preferable
(D) joining
to bottling up irritations. During
Ans - B
arguments, Sathish and ____(A)____ will center
2. Choose the option with the correct punctuation our discussion on the problem, not on
for the given sentence.
other issues. ____(B)____ and I are going to make
penguins live in antartica a real effort to communicate better

(A) penguins, live in antartica with each other.

(B) penguins live in antartica. (A) me

(C) Penguins live in Antartica. (B) I

(D) "penguins live in antartica" (C) mine

Ans - C (D) my

3. Find the word which is correctly spelt from the Ans - B


given options.
7. B. We couples should realize that disagreements
(A) Inovative must occur between husband and

(B) Priscribe wife. I believe that a healthy quarrel is preferable


to bottling up irritations. During
(C) Sufficeint
arguments, Sathish and ____(A)____ will center
(D) Momentous our discussion on the problem, not on
Ans - D other issues. ____(B)____ and I are going to make
4. Fill in the blanks with suitable Preposition from a real effort to communicate better
the given alternatives. with each other.
I saw him walking ____________ the street. (A) Him
(A) at (B) His
(B) for (C) Himself
(C) across (D) He
(D) with Ans - D
Ans - C 8. Choose the word which expresses nearly the
5. Fill in the blank with suitable tense. opposite meaning of the given word "

The car ___________ here for the past few SWANK ".
months. (A) Indefinite
(A) has been parked (B) Complete
(B) is parked (C) Elegant
(C) was parking (D) Classy
(D) parked Ans - A
Ans - A
9. Choose the word which best expresses the (A) Nocturnel
similar meaning of the given word " DEVIOUS".
(B) Incumbant
(A) Fair
(C) Scorge
(B) Sincere
(D) Improbable
(C) Fishy
Ans - D
(D) Frank
14. Fill in the blanks with suitable Article from the
Ans - C given alternatives.
10. Fill in the blank with suitable tense. Immersion in literature and the arts during
childhood sows the seeds for ________
_________ they coming over for dinner?
evolved, sensitive and critical society.
(A) Is
(A) No article
(B) Will
(B) a
(C) Should
(C) an
(D) Are
(D) the
Ans - D
Ans - C
11. Choose the word which expresses nearly the
opposite meaning of the given word " 15. Choose the word which best expresses the
similar meaning of the given word "
PRISTINE ".
IMPOSTURE ".
(A) Refined
(A) Ethical
(B) Untouched
(B) Cheat
(C) Immaculate
(C) Honor
(D) Affected
(D) Probable
Ans - D
Ans - B
12. Choose the option with the correct punctuation
for the given sentence. Fill in the blanks with a suitable Article from the
given alternatives.
You’ll have enough space for a bed kitchen and
bathroom 1. A black hole is born when ___________ large
star collapses in on itself.
(A) You’ll have enough space for a bed, kitchen,
and bathroom. (A) the (Chosen option)
(B) you'll have enough space for a bed kitchen and (B) a (Correct Answer)
bathroom
(C) No article
(C) you’ll have enough space for a bed kitchen and
(D) an
bathroom.
Ans - B
(D) You'll have enough space for a bed kitchen and
bathroom. 2. Choose the option with the correct punctuation
for the given sentence.
Ans - A
my brothers bag is larger than mine
13. Find the word which is correctly spelt from the
given options. (A) my brothers bag is larger than mine.
(B) my brother's bag is larger than mine. 7. Fill in the blanks with suitable Preposition from
the given alternatives.
(C) My brother's bag is larger than mine.
We had a good time ______________ the trip.
(D) "my brothers bag is larger than mine"
(A) at
Ans - C
(B) into
3. Choose the word which expresses nearly the
opposite meaning of the given word " (C) during
RECEPTIVE ". (D) for
(A) Resistant Ans - C
(B) Flexible 8. Fill in the blank with suitable tense.
(C) Quick Why _______ he not listening to me nowadays?
(D) Bright (A) were
Ans - A (B) is
4. Find the word which is correctly spelt from the (C) was
given options.
(D) has
(A) Ladable
Ans - B
(B) Insentive
9. A. The Cold War was an ongoing political rivalry
(C) Accountablity between the United States and the
(D) Defuse Soviet Union and their respective allies. The United
States pursued ____(A)____ policy of
Ans - D
containment throughout the Cold War. Reagan and
5. Choose the option with the correct punctuation
Gorbachev complimented ____(B)____
for the given sentence.
by signing the arms control agreement and ending
do you like coffee
the cold war.
(A) Do you like coffee?
(A) its
(B) do you like coffee?
(B) whose
(C) do you like coffee!
(C) their
(D) Do you like, coffee?
(D) theirs
Ans - A
Ans - A
6. Find the word which is correctly spelt from the
10. B. The Cold War was an ongoing political rivalry
given options.
between the United States and the
(A) Susceptible
Soviet Union and their respective allies. The United
(B) Scrutinaize States pursued ____(A)____ policy of

(C) Hypothatical containment throughout the Cold War. Reagan and


Gorbachev complimented
(D) Relectant
____(B)____ by signing the arms control
Ans - A agreement and ending the cold war.
(A) each other
(B) one another Ans - D
(C) another 2. A. Snake is a poem by the renowned poet D.H.
Lawrence. In this poem, the poet
(D) other
describes the conflict in his mind after ____(A)____
Ans - A
sees a snake in his water-trough.
11. Choose the word which expresses nearly the
The snake is golden- brown in colour and as per the
opposite meaning of the given word "
social education, ____(B)____ is
LIBERATED ".
poisonous and needs to be killed. But, due to his
(A) Loose natural human instincts, the poet feels

(B) Free that it is a guest.

(C) Hold (A) he

(D) Detach (B) him

Ans - C (C) himself

12. Choose the word which best expresses the (D) his
similar meaning of the given word " GAZE ".
Ans - A
(A) Moderate
3. B. Snake is a poem by the renowned poet D.H.
(B) Ignorance Lawrence. In this poem, the poet

(C) Important describes the conflict in his mind after ____(A)____


sees a snake in his water-trough.
(D) Look
The snake is golden- brown in colour and as per the
Ans - D social education, ____(B)____ is
13. Fill in the blank with suitable word from the poisonous and needs to be killed. But, due to his
given options. natural human instincts, the poet feels
The _____________ of President Kennedy shocked that it is a guest.
the world.
(A) its
(A) perils
(B) it
(B) assassination
(C) itself
(C) stubble
(D) they
(D) depreciation
Ans - B
Ans - B
4. Choose the option with the correct punctuation
1. Choose the word which expresses nearly the for the given sentence.
opposite meaning of the given word "
every dog cat and pig in the kennel needs a good
IMBECILE ". place
(A) Dull (A) every dog cat, and pig in the kennel needs a
(B) Inane good place

(C) Thick (B) Every dog cat and pig in the kennel needs a
good place
(D) Smart
(C) Every dog, cat, and pig in the kennel needs a 14. Fill in the blanks with suitable Preposition from
good place. the given alternatives.
(D) every dog, cat and pig in the kennel needs a Be nice _________ your parents.
good place.
(A) in
Ans - C
(B) to
5. Fill in the blank with suitable tense.
(C) at
Radha and her sister _______ to swim.
(D) under
(A) likes
Ans - B
(B) is liking
15. Fill in the blank with suitable word from the
(C) like given options.
(D) had been liked They provided considerable __________ evidence
to support their argument.
Ans - C
(A) towering
6. Fill in the blanks with a suitable Article from the
given alternatives. (B) persuade
South India is _______ area including the five (C) empirical
southern Indian states.
(D) perturb
(A) No article
Ans - C
(B) an
16. Choose the word which best expresses the
(C) a similar meaning of the given word "
(D) the DISGUISE ".
Ans - D (A) Represent
7. Choose the word which best expresses the (B) Disclose
similar meaning of the given word "
(C) Conceal
INTIMIDATE ".
(D) Divulge
(A) Assure
Ans - C
(B) Soothe
17. Fill in the blank with suitable tense.
(C) Incite
After we___________ the egg cake, we began to
(D) Frighten feel sick.
Ans - D (A) had eaten
8. Find the word which is correctly spelt from the (B) is eaten
given options.
(C) have eaten
(A) Sucumb
(D) has eaten
(B) Recuncile
Ans - A
(C) Rigorous
18. Find the word which is correctly spelt from the
(D) Veritabel given options.
Ans - C (A) Ineficiency
(B) Noteworthy (C) My favorite colors are yellow green and red.
(C) Stringant (D) my favorite colors are yellow green and red!
(D) Tremenduous Ans - B
Ans - B
3. Fill in the blank with suitable word from the
19. Choose the option with the correct punctuation given options.
for the given sentence.
I was _______ by your question.
oh no I shouldnt have said that
(A) novice
(A) "oh no I shouldnt have said that'
(B) spur
(B) "oh no" I shouldnt have said that
(C) oh! no I shouldnt have said that (C) perennial

(D) Oh no! I shouldn't have said that. (D) intrigued

Ans - D Ans - D
20. Choose the word which expresses nearly the 4. Fill in the blank with suitable tense.
opposite meaning of the given word "
I have ______ the book.
ABANDONED ".
(A) loosen
(A) Forgotten
(B) lost
(B) Rejected
(C) Adopted (C) loose

(D) Dropped (D) looses

Ans - C Ans - B
1. Fill in the blanks witha suitable article from the 5. Fill in the blank with suitable tense.
given alternatives.
If you ________ me, I would have given you the
We are trying to give ______ incredible book.
performance in the farewell day function.
(A) has asked
(A) a
(B) had ask
(B) the
(C) had asked
(C) No article
(D) would ask
(D) an
Ans - C
Ans - D
6. Choose the word which expresses nearly the
2. Choose the option with the correct punctuation opposite meaning of the given word
for the given sentence.
INFIRM.
my favorite colors are yellow green and red
(A) Faint
(A) my favorite colors are yelow, green and red.
(B) Anaemic
(B) My favorite colors are yellow, green, and red.
(C) Fragile
(D) Strong 10. A. The theme of the short story "The Mark of
Vishnu" is superstition and ignorance.
Ans - D
Gunga Ram, is both superstitious and ignorant. He
7. Choose the option with the correct punctuation
believes that by leaving a saucer full
for the given sentence.
of milk for the snake to drink, _____(A)_____ is
dont forget will you
protecting the household from snake bites.
(A) Don't forget, will you?
But unfortunately, he dies when _____(B) ______
(B) don't forget will you? bites him accidentally.

(C) dont forget will you? (A) she

(D) dont forget will you! (B) himself

Ans - A (C) he
8. Fillin the blanks with suitable Preposition from (D) it
the given alternatives.
Ans - D
My sister lives in the building ______ the end of
11. Find the word which is correctly spelt from the
the crosscut road.
given options.
(A) over
(A) Discretion
(B) at
(B) Systemmic
(C) on
(C) Presumtion
(D) since
(D) Credibel
Ans - B
Ans - A
9. A. The theme of the short story "The Mark of
12. Choose the word which best expresses the
Vishnu" is superstition and ignorance.
similar meaning ofthe given word
Gunga Ram, is both superstitious and ignorant. He
INTERMITTENT
believes that by leaving a saucer full
(A) Healthy
of milk for the snake to drink, _____(A)_____ is
protecting the household from snake bites. (B) Aggressive

But unfortunately, he dies when _____(B) ______ (C) Irregular


bites him accidentally.
(D) Constant
(A) she
Ans - C
(B) he
13. Find the word which is correctly spelt from the
(C) himself given options.

(D) they (A) Aberation

Ans - B (B) Camouflage

(C) Dedlock
(D) Asertain 3. Choose the word which expresses nearly the
opposite meaning of the given word "
Ans - B
ALOOF ".
14. Choose the word which expresses nearly the
(A) Interested
opposite meaning of the given word
(B) Unfriendly
CONFRONT
(C) Reserved
(A) Brave
(D) Distant
(B) Avoid
Ans - A
(C) Resist
4. Fill in the blanks with suitable Article from the
(D) Challenge given alternatives.

Ans - B The Supreme Court on Tuesday ordered immediate


release of all _________ journalists.
15. Choose the word which best expresses the
(A) No Article
similar meaning ofthe given word REPLETE
(B) a
(A) Empty
(C) the
(B) Charged
(D) an
(C) Enlarge
Ans - C
(D) Unfilled 5. A. The Selfish Giant is a story of a young boy who
was a messenger of God and a
Ans - B
selfish giant. The giant would not allow the
1. Choose the word which best expresses the
children to play in his garden. Thus,
similar meaning of the given word "
____(A)____ was always winter in his garden.
STAGGER ".
Seeing this young boy, he realized that
(A) Steady
he was being selfish. Thus, he allowed the children
(B) Continue to play in his garden and also played

(C) Peak with ____(B)____.

(D) Shake (A) their

Ans - D (B) this

2. Fill in the blank with suitable tense. (C) it

The parcel ___________ until I had left. (D) that

(A) didn't arrived Ans - C

(B) arrived 6. B. The Selfish Giant is a story of a young boy who


was a messenger of God and a
(C) has arrived
selfish giant. The giant would not allow the
(D) didn't arrive children to play in his garden. Thus,
Ans - D ____(A)____ was always winter in his garden.
Seeing this young boy, he realized that he
was being selfish. Thus, he allowed the children to 10. Fill in the blanks with suitable Preposition from
play in his garden and also played the given alternatives.
with ____(B)____. The match was dedicated ______________ Indian
fast bowler Zaheer, who has
(A) they
announced his retirement from international
(B) their
cricket.
(C) them
(A) since
(D) it
(B) from
Ans - C
(C) against
7. Find the word which is correctly spelt from the
(D) to
given options.
Ans - D
(A) Entitlemant
11. Fill in the blank with suitable tense.
(B) Discrimnation
I like people who always ___________ the truth.
(C) Acknouledge
(A) spoke
(D) Subsistence
(B) will speak
Ans - D
(C) speaks
8. Choose the option with the correct punctuation
for the given sentence. (D) speak
i have to visit my grandmother buy vegetables and Ans - D
cook dinner
12. Choose the option with the correct punctuation
(A) "i have to visit my grandmother buy vegetables for the given sentence.
and cook dinner'
i have been working for the past 12 hours mohan
(B) I have to visit my grandmother, buy vegetables, complained
and cook dinner.
(A) "I have been working for the past 12 hours,''
(C) i have to visit my grandmother, buy vegetables Mohan complained.
and cook dinner.
(B) "'i have been working for the past 12 hours
(D) i have to visit my grandmother buy vegetables mohan complained"
and cook dinner?
(C) "i have been working for the past, 12 hours
Ans - B mohan complained"
9. Choose the word which best expresses the (D) i have been working for the past 12 hours
similar meaning of the given word " mohan complained.
INVINCIBLE ".
(A) Surrendering Ans - A
(B) Thin
(C) Powerful 13. Fill in the blank with suitable word from the
given options.
(D) Breakable
I said I would ___________ by their decision.
Ans - C
(A) metaphor
(B) abide 3. A. Drawing is my passion. I draw the land around
me, and lands I make up in my head. I
(C) envisage
draw people I see, and fictional creatures. I draw
(D) exorbitant
anything and ____(A)____ that gives me
Ans - B
inspiration. Lately, though, ____(B)____ strange is
14. Choose the word which expresses nearly the happening with my drawings. Some of
opposite meaning of the given word "
them are missing at nights.`
TRANSCEND ".
(A) everything
(A) Excel
(B) nobody
(B) Overstep
(C) nothing
(C) Surrender
(D) none
(D) Best
Ans - A
Ans - C
4. B. Drawing is my passion. I draw the land around
15. Find the word which is correctly spelt from the me, and lands I make up in my head. I
given options.
draw people I see, and fictional creatures. I draw
(A) Excarsion anything and ____(A)____ that gives me

(B) Fragiel inspiration. Lately, though, ____(B)____ strange is


happening with my drawings. Some of
(C) Disafection
them are missing at nights.
(D) Efficacious
(A) none
Ans - D
(B) something
1. Fill in the blank with suitable tense.
(C) nothing
My mother always__________ the most delicious
cakes. (D) someone
(A) makes Ans - B

(B) make 5. Find the word which is correctly spelt from the
given options.
(C) have made
(A) Breakthrough
(D) is made
(B) Bilataral
Ans - A
(C) Discreat
2. Choose the word which best expresses the
similar meaning of the given word " (D) Amentment

MINIATURE ". Ans - A

(A) Large 6. Fill in the blanks with suitable Preposition from


the given alternatives.
(B) Important
I got hit ____________ a vehicle.
(C) Small
(A) by
(D) Avoid
(B) since
Ans - C
(C) from 11. Choose the word which best expresses the
similar meaning of the given word "
(D) until
HARMONY ".
Ans - A
(A) Peace
7. Fill in the blank with suitable tense.
(B) Jangling
Ram ____________ our house next week.
(C) Dislike
(A) will be visiting
(D) Discord
(B) will be visited
Ans - A
(C) visited
12. Fill in the blank with suitable word from the
(D) would be visited
given options.
Ans - A
Tsunamis are associated with earthquakes since
8. Choose the word which expresses nearly the the underground tremors are what
opposite meaning of the given word "
_____________ the rising waves.
CONCISE ".
(A) innovate
(A) Lean
(B) cause
(B) Summary
(C) place
(C) Compact
(D) suspect
(D) Lengthy
Ans - B
Ans - D
13. Find the word which is correctly spelt from the
9. Choose the option with the correct punctuation given options.
for the given sentence.
(A) Zealosness
im tired arent I
(B) Fasionable
(A) im tired, arent I?
(C) Encomium
(B) I'm tired, aren't I?
(D) Concentratad
(C) im tired arent I?
Ans - C
(D) Im tired arent I?
14. Fill in the blank with the appropriate article a,
Ans - B an, or the, or select the option ‘No article’ if

10. Choose the option with the correct punctuation no article is needed.
for the given sentence.
Taj Mahal is one of _________ seven wonders of
is your brother a good cook the world.

(A) is your brother a good cook? (A) an

(B) is your brother, a good cook? (B) the

(C) is your brother a good cook! (C) a


(D) Is your brother a good cook? (D) No article

Ans - D Ans - B
15. Choose the word which expresses nearly the (B) activated
opposite meaning of the given word "
(C) contaminated
COMBATIVE ".
(D) escaped
(A) Antagonistic
Ans - A
(B) Aggressive
5. Fill in the blank with suitable tense.
(C) Peaceful
I _____________ TV right now.
(D) Quarrelsome
(A) am watching
Ans - C
(B) will be watching
1. Fill in the blank with suitable tense.
(C) was watching
_________ he still working on the puzzle at 8 pm
(D) watch
last night?
Ans - A
(A) Has
6. Fill in the blanks with suitable Article from the
(B) Is
given alternatives.
(C) Had
I know ___________ new bus station is somewhere
(D) Was near to the fruit shop.
Ans - D (A) an
2. Find the word which is correctly spelt from the (B) the
given options.
(C) a
(A) Condolance
(D) No Article
(B) Unprecedanted
Ans - B
(C) Abbreviation
12. Choose the word which best expresses the
(D) Suroundings similar meaning of the given word " AVENGE
Ans - C ".
3. Choose the word which best expresses the (A) Repay
similar meaning of the given word "
(B) Comfort
FORMIDABLE ".
(C) Cheer
(A) Poor
(D) Forgive
(B) Trivial
Ans - A
(C) Calm
13. Find the word which is correctly spelt from the
(D) Horrible given options.
Ans - D (A) Ordinence
4. Fill in the blank with suitable word from the (B) Grievuous
given options.
(C) Etternal
A new salon will be ___________ near the city.
(D) Mitigate
(A) established
Ans - D
14. Choose the option with the correct punctuation Ans - D
for the given sentence.
17. Choose the option with the correct punctuation
we will review our homework during tomorrows for the given sentence.
lesson
let's listen to music shall we
(A) we will review our homework during
(A) Let's listen to music, shall we?
tomorrows lesson.
(B) let's listen to music shall we?
(B) We will review our homework during
tomorrows lesson (C) let's listen to music, shall we?
(C) we will review our homework during (D) let's listen to music shall we!
tomorrows lesson?
Ans - A
(D) We will review our homework during
tomorrow’s lesson. 18. Choose the word which expresses nearly the
opposite meaning of the given word "
Ans - D
GLACE ".
15. A. Sejong swept his books off the shelf. Rain
pelted the world outside. The king opened (A) Dark

one of his many works, A guidebook to farming in (B) Dazzling


Korea’s geography. Words that could (C) Bright
keep his people from famine. He threw (D) Lambent
____(A)____ into the rain. His people were
Ans - A
illiterate and so _____(B)____ could read his
works. So he got angry and threw them. 19. Fill in the blanks with suitable Preposition from
the given alternatives.
(A) this
I am going to receive the first prize ____________
(B) them Annual day function.
(C) its (A) from
(D) it (B) to
Ans - D (C) in
16. B. Sejong swept his books off the shelf. Rain (D) on
pelted the world outside. The king opened
Ans - C
one of his many works, A guidebook to farming in
Korea’s geography. Words that could 20. Choose the word which expresses nearly the
opposite meaning of the given word "
keep his people from famine. He threw
____(A)____ into the rain. His people were AESTHETIC ".
illiterate (A) Calculated
and so _____(B)____ could read his works. So he (B) Unattractive
got angry and threw them.
(C) Deal
(A) no pronoun
(D) Coherence
(B) everyone
Ans - B
(C) someone
1. Find the word which is correctly spelt from the
(D) none given options.
(A) Emphatic 11. Fill in the blanks with suitable Preposition from
the given alternatives.
(B) Diligance
The economy grew ________ 5.8% in the last
(C) Florish
quarter of 2018-19.
(D) Establlish
(A) on
Ans - A
(B) by
7. Choose the option with the correct punctuation
(C) in
for the given sentence.
(D) through
shut up can you
Ans - B
(A) "shut up can you"
12. A. An uncounted number of men and women
(B) shut up can you!
have stepped up and served this country
(C) shut up can you?
so that ____(A)____ might keep our freedoms. We
(D) Shut up, can you? should all be thankful when we get a

Ans - D chance to repay ____(B)____. by being a good


citizen, paying all our taxes on time, we
8. Fill in the blank with suitable tense.
help the freedom fighters get their pension on
Where ________ you be going for check-up today? time.
(A) are (A) I
(B) will (B) you
(C) have (C) he
(D) were (D) we
Ans - B Ans - D
9. Choose the word which best expresses the 13. B. An uncounted number of men and women
similar meaning of the given word " have stepped up and served this country
RECEPTIVE ". so that ____(A)____ might keep our freedoms. We
(A) Unwilling should all be thankful when we get a

(B) Approachable chance to repay ____(B)____. by being a good


citizen, paying all our taxes on time, we
(C) Unfriendly
help the freedom fighters get their pension on
(D) unsusceptible time.
Ans - B (A) we
10. Find the word which is correctly spelt from the (B) them
given options.
(C) us
(A) Anticipated
(D) they
(B) Abdigate
Ans - B
(C) Apparantly
14. Choose the option with the correct punctuation
(D) Contemblate for the given sentence.
Ans - A you have such a nice dimple
(A) "You have such a nice dimple" (C) Argument
(B) you have such a nice dimple! (D) Agreement
(C) You have such a nice dimple! Ans - C
(D) you have such a nice dimple? 19. Fill in the blanks with suitable Article from the
given alternatives.
Ans - C
Hindustan Times is _________ Indian English-
15. Choose the word which expresses nearly the
language daily newspaper founded in
opposite meaning of the given word "
1924.
INEVITABLE ".
(A) the
(A) Imminent
(B) a
(B) Avoidable
(C) No Article
(C) Necessary
(D) an
(D) Inflexible
Ans - D
Ans - B
20. Fill in the blank with suitable tense.
16. Choose the word which expresses nearly the
opposite meaning of the given word " My mother_________ today, we must eat outside.
SHALLOW ". (A) was not cooking
(A) Unthinking (B) was cooking
(B) Superficial (C) cook
(C) Deep (D) is not cooking
(D) Sketchy Ans - D
Ans - C 1. Fill in the blanks with suitable Article from the
given alternatives.
17. Fill in the blank with suitable word from the
given options. Hereditary traits and factors may play __________
The company has been moved to new major role in tooth decay and gum
______________. disease.
(A) outrage (A) the
(B) premises (B) No Article
(C) ailment (C) an
(D) archieves (D) a
Ans - B Ans - D
18. Choose the word which best expresses the 2. Choose the word which expresses nearly the
similar meaning of the given word " opposite meaning of the given word "
WRANGLE ". CONCEIVED ".
(A) Harmony (A) Fancy
(B) Upgrade (B) Neglected
(C) Accepted (D) have sought
(D) Realized Ans - D
Ans - B 7. A. Mrs. Lindsey was _____(A)____ favorite
teacher at Lincoln Elementary School. She
3. Fill in the blank with suitable word from the
given options. was known for her entertaining teaching
techniques. One day she walked into the
The manager ___________ an extra worker to help
customers during the store’s busy classroom and announced to the class that they
would all be seeing stars in their
festival season.
futures. She then asked, “Does ____(B)____ know
(A) hired
what astronomy is?
(B) created
(A) anyone's
(C) glared
(B) someone's
(D) blurred
(C) no one's
Ans - A
(D) everyone's
4. Choose the word which expresses nearly the
Ans - D
opposite meaning of the given word "
8. B. Mrs. Lindsey was _____(A)____ favorite
ADORING "
teacher at Lincoln Elementary School. She
(A) Cherish
was known for her entertaining teaching
(B) Dislike techniques. One day she walked into the

(C) Esteem classroom and announced to the class that they


would all be seeing stars in their
(D) Venerate
futures. She then asked, “Does ____(B)____ know
what astronomy is?
Ans - B (A) no one's
5. Choose the word which best expresses the (B) everyone's
similar meaning of the given word "
(C) anyone
EMBRACE ".
(D) someone's
(A) Distrust
Ans - C
(B) Lock
9. Fill in the blanks with suitable Preposition from
(C) Free the given alternatives.
(D) Give Tourism is the practice ___________ travelling for
Ans - B happiness especially on holidays.

6. Fill in the blank with suitable tense. (A) of

He __________ out his father in the crowd. (B) from

(A) sought (C) for

(B) seeked (D) since

(C) seek Ans - A


10. Find the word which is correctly spelt from the (C) ram is a student! isnt he?
given options.
(D) Ram is a student isnt he?
(A) Brutel
Ans - A
(B) Coherant
15. Fill in the blank with suitable tense.
(C) Desperate
Raj and his friend ___________ when the food
(D) Dismandle arrived.
Ans - C (A) was swimming
11. Choose the word which best expresses the (B) were swimming
similar meaning of the given word " TRIVIAL
(C) is swimming
".
(D) had swum
(A) Important
Ans - B
(B) Large
1. Fill in the blanks with suitable Article from the
(C) Small
given alternatives.
(D) Avoid
I am willing to participate in _________ English
Ans - C elocution function conducted by our
12. Find the word which is correctly spelt from the institution.
given options.
(A) an
(A) Efficasy
(B) No article
(B) Mandaytory
(C) the
(C) Hazardous
(D) a
(D) Inadeqate
Ans - C
Ans - C
2. Choose the word which best expresses the
13. Choose the option with the correct punctuation
similar meaning of the given word "
for the given sentence.
TRANSCEND ".
we will see rahuls new house
(A) Surrender
(A) We will see Rahul's new house.
(B) Submission
(B) We will see rahuls new house.
(C) Transform
(C) we will see Rahul's new house
(D) Resignation
(D) We will see rahul's new house.
Ans - C
Ans - A
3. Find the word which is correctly spelt from the
14. Choose the option with the correct punctuation
for the given sentence. given options.

ram is a student isnt he (A) Spekulate

(A) Ram is a student, isn't he? (B) Transprent

(B) ram is a student isnt he? (C) Simultaneous


(D) Paramownt
Ans - C think I will take my father's camera, because
4. Find the word which is correctly spelt from the ____(A)____ is new and is working well. I
given options. definitely cannot take ____(B)____ because I broke
(A) Exemplery it last week when I was taking some
(B) Indifferance photos.
(C) Heirarchy (A) my
(D) Feudal (B) my's
Ans - D (C) mine
5. Fill in the correct prepositions. (D) mines
The team's late arrival was _____________ the Ans - C
rain. 8. Choose the option with the correct punctuation
(A) owing to for the given sentence.
(B) instead of what a tragedy
(C) within (A) what a tragedy.
(D) at (B) "what a tragedy"
Ans - A (C) what a tragedy?
6. A. This weekend I am planning to go to a (D) What a tragedy!
wedding party. I am not sure which camera I Ans - D
should take. I could take my brother's camera, but 9. Fried chicken is best when it is ____________, so
am not sure if it is working properly. I that there is plenty of crunch in each
think I will take my father's camera, because and every bite.
____(A)____ is new and is working well. I (A) drowsy
definitely cannot take ____(B)____ because I broke (B) bumby
it last week when I was taking some (C) crispy
photos. (D) snappy
(A) him's Ans - C
(B) his 10. Choose the word which best expresses the
(C) himselves similar meaning of the given word "
(D) his's PROFOUND ".
Ans - B (A) Intellectual
7. B. This weekend I am planning to go to a (B) Superficial
wedding party. I am not sure which camera I (C) Ignorant
should take. I could take my brother's camera, but (D) Moderate
am not sure if it is working properly. I Ans - A
11. Choose the word which expresses nearly the (B) Concord
opposite meaning of the given word " (C) Amity
INCEPTION ". (D) Peace
(A) Origin Ans - A
(B) Conclusion 6. Complete the sentence with appropriate
prepositions.
(C) Source
The railway track __________ the river was
(D) Outset
recently closed due to Waterlogging.
Ans - B
(A) for
12. Choose the option with the correct punctuation
(B) between
for the given sentence.
(C) by
please stop talking will you
(D) along
(A) please stop talking will you?
Ans - D
(B) Please stop talking, will you?
7. Choose the word which expresses nearly the
(C) please stop talking will you! opposite meaning of the given word "
(D) please stop talking will you. GAZE ".
Ans - B (A) Survey
13. Fill in the blank with suitable tense.
(B) Ignorance
The court _________ the person responsible for
(C) Goggle
the mischief.
(D) Stare
(A) was held
Ans - B
(B) held
8. Fill in the blanks with suitable Article from the
(C) is held given alternatives.
(D) hold He is good at ________ English.
Ans - B (A) No article
14. Fill in the blank with suitable tense. (B) an
My daughter was sleeping when I ________ home. (C) the
(A) is coming (D) a
(B) have come
Ans - A
(C) came
9. A. On his deathbed, Aldous Huxley reflected on
(D) come his entire life's learning and summoned
Ans - C it up in simple words "Let us be kinder to one
____(A)____." Often we believe we must
20. Choose the word which expresses nearly the
achieve a great feat to lead a fulfilling life which is
opposite meaning of the given word "
not true. Kindness shown towards
HARMONY ".
your people makes up ____(B)____ you are.
(A) Clash
(A) another 13. Find the word which is correctly spelt from the
given options.
(B) each
(A) Legitimate
(C) other
(B) Persistant
(D) no pronoun
(C) Multilatral
Ans - A
(D) Irrepaerable
10. B. On his deathbed, Aldous Huxley reflected on
his entire life's learning and summoned Ans - A
it up in simple words "Let us be kinder to one 14. Choose the word which best expresses the
____(A)____." Often we believe we must similar meaning of the given word "
achieve a great feat to lead a fulfilling life which is FLAWED "
not true. Kindness shown towards
(A) Adjusted
your people makes up ____(B)____ you are.
(B) Damaged
(A) who
(C) Unbroken
(B) no pronoun
(D) Repaired
(C) whom
Ans - B
(D) whose
15. Fill in the blank with suitable tense.
Ans - A
The new regulation also _______ not allow the use
11. Choose the option with the correct punctuation of mobile phones in cafeteria.
for the given sentence.
(A) has
ouch that really hurts
(B) do
(A) ouch! that really hurts?
(C) have
(B) ouch that really hurts?
(D) does
(C) ouch that really hurts!
Ans - D
(D) Ouch! That really hurts.
16. Find the word which is correctly spelt from the
Ans - D given options.
12. Choose the option with the correct punctuation (A) Reitrate
for the given sentence.
(B) Letargy
it became the joke of the family says ms. miller
(C) Outraige
(A) “It became the joke of the family,” says Ms.
(D) Milestone
Miller.
Ans - D
(B) "it became the joke of the family says ms.
Miller" 17. Fill in the blank with suitable tense.
(C) it became the joke of the family says ms. Miller! I am sorry I __________ any attention.
(D) it became the joke of the family says ms. (A) paid
Miller?
(B) wasn't paying
Ans - A
(C) was paying
(D) am paying (C) like
Ans - B (D) have like
18. Choose the word which best expresses the Ans - C
similar meaning of the given word "
Question No.2
POTENCY ".
(A) Laziness Fill in the blank with suitable tense.

(B) Inability The Indian government has __________ the UN for


the current climate crisis.
(C) Capacity
(A) been sought
(D) Apathy
Ans - C (B) sought

19. Choose the word which expresses nearly the (C) seeked
opposite meaning of the given word "
(D) seek
ADVENTITIOUS "
Ans - B
(A) Sporadic
Question No.3
(B) Accidental
Convert the given direct/ indirect speech to
(C) Planned
indirect/ direct speech with the help of the
(D) Fortuitous
given options.
Ans - C
Jubin said, "Let us decide the matter together."
20. Fill in the blank with suitable word from the
given options. (A) Jubin suggested that they might decided the
Although their graduation dates _____________, matter together.
the two classmates never knew each (B) Jubin suggested that they should decided the
other during college. matter together.
(A) encouraged (C) Jubin suggested that they should decide the
(B) invested matter together.

(C) contaminated (D) Jubin suggested that they should have decided
the matter together.
(D) coincided
Ans - C
Ans - D
OSSC CGL Exam 2021 Question No.4

Question No.1 India always _______ neutral during the conflicts


between nations.
Fill in the blank with appropriate verb in agreement
with its subject. (A) had remained

John and Lee __________ to go to the films. (B) remained

(A) likes (C) remain

(B) are liking (D) remains


Ans - D Question No.8

Question No.5 Fill in the blank with suitable tense.

Find the correct active voice for the given sentence The manager has _______ in a case registered
in passive voice from the given under IPC.

alternatives. (A) been holding

The complaints are categorized by them. (B) holding

(A) They categorizes the complaints. (C) held

(B) They are categorized the complaints. (D) been held

(C) They categorized the complaints. Ans - D

(D) They categorize the complaints. Question No.9

Ans - D Fill in the blanks with suitable Preposition from the


given alternatives.
Question No.6
She went to the wrong address ____________
Education is not an end, but a means to an end. In
mistake.
other words, we do not educate
(A) by
children only for the purpose of educating them;
__________ purpose is to fit them for life. (B) with

(A) our (C) in

(B) that (D) through

(C) this Ans - A

(D) what Question No.10

Ans - A Fill in the blanks with suitable Preposition from the


given alternatives.
Question No.7
I stayed in bed ____________ half past seven.
Find the correct active/ passive voice for the given
sentence in passive/ active voice (A) by

from the given alternatives. (B) until

They visit this orphanage every year. (C) beside

(A) This orphanage is being visited by them every (D) Because


year.
Ans - B
(B) This orphanage is visited by them every year.
Question No.11
(C) This orphanage was visited by them every year.
Fill in the blanks with suitable Article from the
(D) Every year, this orphanage has been visited by given alternatives.
them.
I am going to write ___________ letter for my
Ans - B friend.
(A) a Question No.15

(B) the In these days, very ______ theses are the result of
a proper research discipline.
(C) an
(A) few
(D) No article
(B) every
Ans - A
(C) little
Question No.12
(D) a few
Fill in the blanks with suitable Article from the
given alternatives. Ans - A

In April last year, ________ Reserve Bank of India


issued a directive on Storage of Question No.1
Payment System Data. Fill in the blank with suitable tense.
(A) the The bill also _______ not allow the use of crypto
currencies in India.
(B) a
(A) have
(C) an
(B) does
(D) No Article
(C) has
Ans - A
(D) do
Question No.13 Ans - B
You _______ take leave now. Your application form Question No.2
is submitted.
Fill in the blank with appropriate verb in agreement
(A) will with its subject.

(B) may Either Julie or her sisters __________ the house


every day.
(C) would
(A) cleans
(D) could
(B) clean
Ans - B (C) is cleaning
Question No.14 (D) cleaned
_______ veena, she also knows how to play violin Ans - B
and flutes. Question No.3
(A) However The exam was very tough; _____, I wrote well.
(B) Besides (A) though
(C) Otherwise (B) by chance

(D) Unless (C) and

Ans - B (D) however


Ans - D (A) a
Question No.4 (B) the
Choose the correct article to complete the (C) No article
following sentence.
(D) an
Australia’s jobless rate unexpectedly increased in
Ans - D
________ January despite a surge in
Question No.13
full-time employment.
Find the correct active/ passive voice for the given
(A) a
sentence in passive/ active voice
(B) an
from the given alternatives.
(C) No article
They announced the awards for this year.
(D) the
(A) The awards for this year are announced by
Ans - C them.
Question No.10 (B) The awards for this year announced by them.
Choose the best preposition from the given (C) The awards for this year had announced by
alternatives for the blank in the given them.
question (D) The awards for this year were announced by
them.
Mother placed several cups _____________ the
kitchen table. Ans - D
(A) besides Question No.14
(B) for She _______ speak three languages fluently.
(C) over (A) May
(D) on (B) Can
Ans - D (C) Should
Question No.11 (D) Need
The children _____ (play) when the ice cream van Ans - B
arrived.
Question No.15
(A) are playing
Find the correct active/ passive voice for the given
(B) played sentence in passive/ active voice
(C) has been playing from the given alternatives.
(D) were playing You haven't submitted your assignment yet.
Ans - D (A) your assignment has not submitted yet.
Question No.12 (B) your assignment has not been submitted yet.
Choose the correct article to complete the (C) your assignment had not been submitted yet.
following sentence.
(D) your assignment has been submitted yet.
Rakshan bought __________ orange colour water
Ans - B
bottle.
Question No.10 (C) The awards for this year had announced by
them.
Choose the best preposition from the given
alternatives for the blank in the given question (D) The awards for this year were announced by
them.
Mother placed several cups _____________ the
kitchen table. Ans - D
(A) besides Question No.14
(B) for She _______ speak three languages fluently.
(C) over (A) May
(D) on (B) Can
Ans - D (C) Should
Question No.11 (D) Need
The children _____ (play) when the ice cream van Ans - B
arrived.
Question No.15
(A) are playing
Find the correct active/ passive voice for the given
(B) played sentence in passive/ active voice
(C) has been playing from the given alternatives.
(D) were playing You haven't submitted your assignment yet.
Ans - D (A) your assignment has not submitted yet.
Question No.12 (B) your assignment has not been submitted yet.
Choose the correct article to complete the (C) your assignment had not been submitted yet.
following sentence.
(D) your assignment has been submitted yet.
Rakshan bought __________ orange colour water
Ans - B
bottle.
(A) a
Question No.1
(B) the
Find the correct active/ passive voice for the given
(C) No article
sentence in passive/ active voice
(D) an
from the given alternatives.
Ans - D
They are using students like machines.
Question No.13
(A) Students have used by them like machines.
Find the correct active/ passive voice for the given
(B) Students are used by them like machines.
sentence in passive/ active voice
(C) Students are being used by them like machines.
from the given alternatives.
(D) Students are using machines.
They announced the awards for this year.
Ans - C
(A) The awards for this year are announced by
them. Question No.2
(B) The awards for this year announced by them.
She skipped her breakfast ______, she fainted in question
the classroom.
We are living _________ Chennai.
(A) besides
(A) in
(B) because
(B) for
(C) meanwhile
(C) from
(D) and so
(D) beside
Ans - D
Ans - A
Question No.3
Question No.7
______ student is so unique with different skills in
Fill in the blank with suitable tense.
this class.
According to the news reports, two dozen apples
(A) a few
__________ one hundred and fifty
(B) All
rupees.
(C) Some
(A) has cost
(D) Each
(B) costed
Ans - D
(C) is costing
Question No.4
(D) cost
Fill in the blank with appropriate verb in agreement
Ans - D
with its subject.
Question No.8
The players as well as their coacher ___________
good at their training. _____ you stand by me, even if the situation gets
worse?
(A) are
(A) Should
(B) have
(B) will
(C) has
(C) would
(D) is
(D) could
Ans - A
Ans - B
Question No.5
Question No.9
Last night, she ______ on the floor with her eyes
closed in despair. Find the correct active voice for the given sentence
in passive voice from the given
(A) laid
alternatives.
(B) is lying
This document has been proof-read by him.
(C) had laid
(A) He had been proof-read this document.
(D) lay
(B) He proof-read this document.
Ans - D
(C) He had proof-read the document.
Question No.6
(D) He has proof-read this document.
Choose the best preposition from the given
alternatives for the blank in the given Ans - D
Question No.10 (A) The manager ordered the employee to file
those papers now.
Fill in the blank with suitable tense.
(B) The manager ordered the employee to file
The committee _________ taken a firm decision.
those papers immediately.
(A) have
(C) The manager ask the employee to file those
(B) was papers immediately.

(C) is (D) The manager ordered the employee to file


those papers then.
(D) has
Ans - B
Ans - D
Question No.19
Question No.11
Advertisements have become a part of our lives.
Choose the correct article to complete the ___________ cannot live without them,
following sentence.
they attack us from all sides.
Our Principal placed _______ portrait of Queen
Victoria in the school. (A) Them
(A) an (B) We

(B) the (C) Our

(C) a (D) Their

(D) No article Ans - B

Ans - C Question No.20

Question No.12 Fill in the blanks with suitable Preposition from the
given alternatives.
Choose the correct article to complete the
following sentence. The officials were very enthusiastic ____________
the targets achieved by the employees.
The aim of __________ launched policy is to
prevent two-wheelers and four-wheelers from (A) about

straying inside. (B) in

(A) the (C) by

(B) a (D) at

(C) an Ans - A

(D) No article
Ans - A Question No.1

Question No.18 Convert the given direct/ indirect speech to


indirect/ direct speech with the help of the
Convert the given direct/ indirect speech to
indirect/ direct speech with the help of the given options.
given options. Karan said, “We are playing a game today."

The manager said to the employee, “File these (A) Karan said to that they were playing a game
papers immediately” that day.
(B) Karan said that they were playing a game today.
(C) Karan said that they are playing a game that Question No.5
day.
Find the correct active/ passive voice for the given
(D) Karan said that they were playing a game that sentence in passive/ active voice
day.
from the given alternatives.
Ans - D
They will declare holiday, in case of floods.
Question No.2
(A) Holiday will declared by them, in case of floods.
She missed her flight _____ heavy traffic.
(B) Holiday will be declared by them, in case of
(A) moreover floods.
(B) since (C) Holiday would be declared by them, in case of
floods.
(C) due to
(D) Holiday will declare by them, in case of floods.
(D) because
Ans - B
Ans - C
Question No.6
Question No.3
Add ____ bowl of sugar to the water to make
Find the correct active voice for the given sentence
syrup.
in passive voice from the given
(A) the
alternatives.
(B) this
The elephant herds were being led by their female
heads. (C) an
(A) Their female heads are leading the elephant (D) a
herds.
Ans - D
(B) Their female heads leading the elephant herds.
Question No.7
(C) Their female heads were leading the elephant
Fill in the blank with suitable tense.
herds.
I_____________ for this book since Monday.
(D) Their female heads were being led the elephant
herds. (A) has been searching
Ans - C (B) has searched
Question No.4 (C) have been searching
Choose the best preposition from the given (D) had searched
alternatives for the blank in the given
Ans - C
question
Question No.13
He is ready to move ________ his pets.
The whole family _______ come together during
(A) within festivals, when we lived in the village.
(B) along (A) used to
(C) with (B) ought to
(D) through (C) have to
Ans - C (D) need to
Ans - A Question No.18
Question No.14 Choose the correct article to complete the
following sentence.
Advertisements are a help because they tell
__________ about a new product. What does she usually have for ______ dinner?
(A) we (A) an
(B) us (B) a
(C) whom (C) No article
(D) this (D) the
Ans - B Ans - C
Question No.15 Question No.19
Fill in the blank with the appropriate article a, an, Fill in the blanks with suitable Preposition from the
or the, or select the option ‘No article’ if given alternatives.
no article is needed. She agreed ________ the idea of our chief cook
yesterday.
Steyn was out of action for _________ past six
weeks due to a fracture on his right hand. (A) for
(A) the (B) to
(B) an (C) since
(C) a (D) in
(D) No article Ans - B
Ans - A Question No.20
Question No.16 Fill in the blank with appropriate verb in agreement
with its subject.
The cattle ________ on the grasslands.
The kid as well as their parents ____________
(A) grazes
missing.
(B) will graze
(A) is
(C) graze
(B) have
(D) would graze
(C) are
Ans - C
(D) has
Question No.17
Ans - A
Fill in the blank with suitable tense.
He _________ physics for a year now.
Question No.1
(A) have been studying
Remind me a day earlier ______ I may forget to
(B) have studied book tickets.
(C) studied (A) whereas

(D) has been studying (B) otherwise

Ans - D (C) however


(D) but (B) Yash told me that my friends are waiting for
me.
Ans - B
(C) Yash told me that my friends were waiting for
Question No.2
me.
Find the correct active voice for the given sentence
(D) Yash told me my friends were waiting for me.
in passive voice from the given
Ans - C
alternatives.
Question No.6
She will be given a chance by them.
Fill in the blank with the appropriate article a, an,
(A) They would give her a chance.
or the, or select the option ‘No article’ if
(B) They gave her a chance.
no article is needed.
(C) They will give her a chance.
India needs a new consensus on its energy and
(D) They give her a chance. economic transition for _____________

Ans - C sustainable future.

Question No.3 (A) No article

______ you like to have some more coffee? (B) a

(A) Shall (C) an

(B) Would (D) the

(C) Do Ans - B

(D) Will Question No.7

Ans - B I _____ like to read the book.

Question No.4 (A) would

Fill in the blank with suitable tense. (B) could

Every boy and every girl __________ the new (C) might
regulations now.
(D) should
(A) is supporting
Ans - A
(B) has been supported
Question No.8
(C) was supported
Fill in the blanks with suitable Preposition from the
(D) has supported given alternatives.

Ans - A The river Ganges flows ________ Uttar Pradesh.

Question No.5 (A) through

Convert the given direct/ indirect speech to (B) during


indirect/ direct speech with the help of the
(C) above
given options.
(D) against
Yash said to me, “Your friends are waiting for you.”
Ans - A
(A) Yash told me my friends are waiting for me.
Question No.9
A poor farmer died leaving only three things for Question No.18
__________ two sons - a mango tree, a
Fill in the blank with appropriate verb in agreement
blanket and a cow. with its subject.
(A) this Each of the six boys in the class ____________ his
task.
(B) each
(A) was completed
(C) there
(B) were completed
(D) his
(C) have completed
Ans - D
(D) has completed
Question No.10
Ans - D
Fill in the blanks with suitable Preposition from the
given alternatives. Question No.19
We are going to deal ___________ technical Find the correct active/ passive voice for the given
subject. sentence in passive/ active voice
(A) until from the given alternatives.
(B) over She will have taken them to her home.
(C) under (A) They will have taken by her to her home.
(D) with (B) They will been taken by her to her home.
Ans - D (C) They would have taken by her to her home.
Question No.11 (D) They will have been taken by her to her home.
Fill in the blank with suitable tense. Ans - D
By this week, we _________ launched the new Question No.20
app.
Fill in the blanks with suitable Article from the
(A) had given alternatives.
(B) will have English has become _______ common language
among people.
(C) have
(A) No article
(D) would
(B) a
Ans - B
(C) the
Question No.17
(D) an
It was ____ interesting poem that reminded me of
my childhood. Ans - B
(A) an
(B) the Question No.1
(C) a Convert the given direct/ indirect speech to
indirect/ direct speech with the help of the
(D) his
given options.
Ans - A
He said, “I must go next month.”
(A) He said that he would have to gone the next The population _______ the turtle in Assam has
month. gone down by a great extent.
(B) He said that he had to go the following month. (A) of
(C) He said that he will have to go the following (B) at
month.
(C) in
(D) He said that he would have to go the next
(D) for
month.
Ans - A
Ans - B
Question No.11
Question No.7
Fill in the blanks with suitable Article from the
She was uploading the document but ______
given alternatives.
she was also checking for errors before finalizing
Tina's mother is sensitive to _____ cigarette
them.
smoke.
(A) At the same time
(A) the
(B) however
(B) No article
(C) By chance
(C) an
(D) besides
(D) a
Ans - A
Ans - B
Question No.8
Question No.12
Fill in the blank with suitable tense.
Bread and butter with scrambled eggs ______ my
The General manager of the firm didn't_______ to favourite breakfast, when I was little.
attend the meeting.
(A) were
(A) want
(B) was
(B) wants
(C) are
(C) wanting
(D) be
(D) wanted
Ans - B
Ans - A
Question No.13
Question No.9
Cocoa trees grow only in warm areas that get a lot
If you can't, who ____? of rain. The trees grow long fruit
(A) would called ‘pods’ ___________ range in colour from
bright yellow to deep purple.
(B) should
(A) that
(C) may
(B) those
(D) will
(C) when
Ans - D
(D) what
Question No.10
Ans - A
Fill in the blanks with suitable Preposition from the
given alternatives. Question No.14
Find the correct active/ passive voice for the given Ans - D
sentence in passive/ active voice
Question No.18
from the given alternatives.
Fill in the blank with the appropriate article a, an,
She might create problems. or the, or select the option ‘No article’ if
(A) Problems will be created by her. no article is needed.
(B) Problems may be created by her. ________ companies you work with are worth over
300 million dollars.
(C) Problems might created by her.
(A) The
(D) Problems might be created by her.
(B) No article
Ans - D
(C) A
Question No.15
(D) An
____ dog is very dangerous.
Ans - A
(A) This
Question No.19
(B) These
Find the correct active voice for the given sentence
(C) A
in passive voice from the given
(D) Those
alternatives.
Ans - A
This cannot be set right by them so easily.
Question No.16
(A) They can set this right so easily.
Fill in the blanks with suitable Preposition from the
(B) They cannot set this right so easily.
given alternatives.
(C) They cannot be set this right so easily.
Hamilton was born in the US mid-West
___________ 1936, just three years before World (D) They can be set this right so easily.
War II began. Ans - B
(A) in Question No.20
(B) on Fill in the blank with appropriate verb in agreement
with its subject.
(C) from
Ruby is one of the girls who _____________ always
(D) at
on time.
Ans - A
(A) have
Question No.17
(B) were
Fill in the blank with suitable tense.
(C) are
He ________ in the United Kingdom for a year and
(D) is
half now.
Ans - C
(A) have been living
(B) have lived
Question No.1
(C) is living
You ought to _______ to our commands.
(D) has been living
(A) obey Fill in the blank with appropriate verb in agreement
with its subject.
(B) should obey
Ruby and her sisters ________________ at school.
(C) not obey
(A) have
(D) obeyed
(B) are
Ans - A
(C) is
Question No.2
(D) was
Fill in the blanks with suitable Preposition from the
given alternatives. Ans - B
Australia consolidated their lead ________ the Question No.11
ongoing Ashes cricket series.
Fill in the blanks with suitable Article from the
(A) against given alternatives.
(B) on India looks all set for _________ remarkable
transition.
(C) in
(A) No article
(D) to
(B) the
Ans - C
(C) an
Question No.8
(D) a
Find the correct active/ passive voice for the given
sentence in passive/ active voice Ans - D
from the given alternatives. Question No.12
They would have called him. Convert the given direct/ indirect speech to
indirect/ direct speech with the help of the
(A) He will have called by them.
given options.
(B) He would have been call by them.
“Are you alone, my son?” asked a soft voice close
(C) He would have been called by them.
behind me.
(D) He will have been called by them.
(A) A soft voice close behind me asked if I am living
Ans - C alone.

Question No.9 (B) A soft voice close behind me asked if I am


alone.
Fill in the blank with suitable tense.
(C) A soft voice close behind me asked if I was
My friends and I__________ stuck there. alone.
(A) had been (D) A soft voice close behind me asks if I was alone.
(B) have Ans - C
(C) has been Question No.13
(D) is Fill in the blanks with suitable Article from the
Ans - B given alternatives.

Question No.10 Most of ______________ chimneys come with a


lifetime warranty offer so that you can
enjoy cooking for many years. (B) their
(A) an (C) it's
(B) the (D) it
(C) a Ans - D
(D) No Article Question No.18
Ans - B Fill in the blanks with suitable Preposition from the
given alternatives.
Question No.14
She hit the nail ____________ the wood.
Bharathi is calm and flexible, ____ her sister is just
the opposite of her. (A) through
(A) yet (B) for
(B) despite (C) about
(C) although (D) since
(D) whereas Ans - A
Ans - D Question No.19
Question No.15 ___ you please help me with this?
___ kids don't like chocolates. (A) Would
(A) No (B) Might
(B) Few (C) Should
(C) Much (D) Can
(D) Any Ans - D
Ans - B Question No.20
Question No.16 Fill in the blank with suitable tense.
Find the correct active voice for the given sentence Neither she nor her sister ___________ at the
in passive voice from the given party.
alternatives. (A) are
Ask your questions. (B) was
(A) Let the questions be asked. (C) were
(B) Let your question be asked. (D) has
(C) Let your questions be asked. Ans - B
(D) Let your questions asked.
Question No.1
Ans - C
During my school days, I ____ walk any distance
Question No.17
easily.
I finished my poem, and ____________ was
(A) should
beautiful!
(B) could
(A) this
(C) will (A) as a result
(D) must (B) despite
Ans - B (C) yet
Question No.2 (D) though
Fill in the blank with appropriate verb in agreement Ans - C
with its subject.
Question No.6
The coachers, as well as the management,
A gentleman was repeatedly picking up starfish
____________ to win.
along a beach in Mexico. ___________
(A) wants
was throwing them into the sea.
(B) want
(A) They
(C) has wanted
(B) She
(D) have wanted
(C) He
Ans - B
(D) Himself
Question No.3
Ans - C
Fill in the blanks with suitable Preposition from the
Question No.7
given alternatives.
You might not _______ to see the dog run over by
The World Bank Group has downgraded global real
a van.
GDP growth __________ 2.6% in 2019.
(A) want
(A) since
(B) wants
(B) through
(C) had wanted
(C) to
(D) wanted
(D) for
Ans - A
Ans - C
Question No.8
Question No.4
Fill in the blanks with suitable Preposition from the
Find the correct active voice for the given sentence
given alternatives.
in passive voice from the given
She has been absent __________ Monday.
alternatives.
(A) since
Let the windows be closed.
(B) for
(A) Close windows.
(C) over
(B) Close the window.
(D) above
(C) Closed the windows.
Ans - A
(D) Close the windows.
Question No.9
Ans - D
Bring ___ snacks to your uncle.
Question No.5
(A) each
Ram was always careful while proofreading texts
__ he couldn't provide the texts errorfree. (B) some
(C) every (A) had become
(D) any (B) have become
Ans - B (C) has become
Question No.15 (D) becomes
Fill in the blanks with suitable Article from the Ans - A
given alternatives.
Question No.19
English has become one of _______ common
Fill in the blanks with suitable Article from the
languages among people.
given alternatives.
(A) the
She does not like ______ pets, but she likes her
(B) an cousin's cat.
(C) No article (A) a
(D) a (B) the
Ans - A (C) No article
Question No.16 (D) an
Fill in the blank with suitable tense. Ans - C
Adam __________ a healthy regime every day. Question No.20
(A) is following Convert the given direct/ indirect speech to
indirect/ direct speech with the help of the
(B) has followed
given options.
(C) had been followed
She said, “Where shall I be this time next month!”
(D) follows
(A) She wondered where she would be that time
Ans - D
the following month.
Question No.17
(B) She wonder where she would be that time the
Find the correct active/ passive voice for the given following month.
sentence in passive/ active voice
(C) She wondered where she will be that time the
from the given alternatives. following month.

They condemned the movie. (D) She wondered where she would be that time
next month.
(A) The movie was condemned them.
Ans - A
(B) The movie was condemned by them. Fill in the blank with appropriate verb in agreement
(C) The movie condemned by them. with its subject.

(D) The movie condemned them. Nobody ____________ the trouble she has seen.

Ans - B (A) know

Question No.18 (B) have known

Fill in the blank with suitable tense. (C) knows

By the time he was twelve, he__________ a great (D) known


orator. Ans - C
Question No.2 (D) across
I am sure, he ___ make you proud. Ans - D
(A) will Question No.11
(B) may Find the correct active/ passive voice for the given
sentence in passive/ active voice
(C) can
from the given alternatives.
(D) should
History teaches us lessons.
Ans - A
(A) Lessons are taught by history.
Question No.3
(B) Lessons are being taught by history.
Kausani is situated at a height of 6,075 feet in the
Central Himalayas. ____________ is (C) The lessons are taught by history.
an unusually attractive little town. It covers just (D) Lessons are taught by the history.
about 5.2 sq km.
Ans - A
(A) Their
Question No.12
(B) It
Fill in the blanks with suitable Preposition from the
(C) It's given alternatives.
(D) Them Scientists ___________ Thursday unveiled the
most detailed simulation of a black
Ans - B
holes.
Question No.4
(A) in
Convert the given direct/ indirect speech to
indirect/ direct speech with the help of the (B) on
given options. (C) for
Rubita said to me, “I will do it now or never.” (D) at
(A) Rubita told she would do it then or never. Ans - B
(B) Rubita told that she would do it then or never. Question No.13
(C) Rubita told me that she will do it then or never. Fill in the blank with the appropriate article a, an,
or the, or select the option ‘No article’ if
(D) Rubita told me that she would do it then or
never. no article is needed.
Ans - D She has ______________ good friend who lives
near her town.
Question No.10
(A) an
Fill in the blanks with suitable Preposition from the
given alternatives. (B) No article
The government is constructing a bridge ______ (C) a
the river.
(D) the
(A) for
Ans - C
(B) besides
Question No.14
(C) about
Fill in the blank with suitable tense. Fill in the blanks with suitable Article from the
given alternatives.
By this time next week, I __________ for the Indian
Cricket team. The company is looking for ______ MA graduate.
(A) will be playing (A) a
(B) will have playing (B) an
(C) will have been played (C) No article
(D) has played (D) the
Ans - A Ans - B
Question No.15 Question No.19
Find the correct active voice for the given sentence Fill in the blank with suitable tense.
in passive voice from the given
When he called me, I _________ a bath.
alternatives.
(A) has been taking
Parts of Thirukural was translated by Kindersely.
(B) took
(A) Kindersely was translated parts of Thirukural.
(C) was taking
(B) Kindersely translated parts of Thirukural.
(D) am taking
(C) Kindersely has translated parts of Thirukural.
Ans - C
(D) Kindersely had translated parts of Thirukural.
Question No.20
Ans - B
The actress expressed a ___ of thanks to her fans.
Question No.16
(A) few
The medicine didn't work for her. ____, it made her
(B) many
sick even more.
(C) lot
(A) By chance
(D) several
(B) In spite of
Ans - C
(C) Eventhough
(D) Instead
Question No.6
Ans - D
Find the correct active/ passive voice for the given
Question No.17 sentence in passive/ active voice
She ______ her children with great difficulty but from the given alternatives.
she never gave up her patience.
He noted down the questions.
(A) raises
(A) The questions are noted down by them.
(B) arose
(B) The questions noted down them.
(C) raised
(C) The questions were noted down by them.
(D) rose
(D) The questions noted down by them.
Ans - C
Ans - C
Question No.18
Question No.7
Fill in the blank with suitable tense. Question No.11
The German course ____________ yet. Fill in the blanks with suitable Article from the
given alternatives.
(A) has started
Jasprit Bumrah has taken _____________
(B) has not been started
cricketing world by storm.
(C) have started
(A) a
(D) started
(B) an
Ans - B
(C) No Article
Question No.8
(D) the
We are going to Lal park today, ___ you join us?
Ans - D
(A) shall
Question No.12
(B) will
Fill in the blanks with suitable Preposition from the
(C) could given alternatives.
(D) may What is the time ________ your watch?

Ans - B (A) through

Question No.9 (B) at

Find the correct active voice for the given sentence (C) by
in passive voice from the given
(D) on
alternatives.
Ans - C
The chief guest is welcomed by them.
Question No.13
(A) They welcome the chief guest.
Fill in the blanks with suitable Preposition from the
(B) They welcomed the chief guest. given alternatives.

(C) They are welcoming the chief guest. It has been raining _______________ morning.

(D) They had welcome the chief guest. (A) at

Ans - A (B) since

Question No.10 (C) by

Convert the given direct/ indirect speech to (D) for


indirect/ direct speech with the help of the
Ans - B
given options.
Question No.14
She said,” I killed a mosquito”
Last month we got ____ new customers and
(A) She said that she has killed a mosquito. increased our profit.

(B) She said that she killed a mosquito. (A) many


(C) She said that she had killed a mosquito. (B) little

(D) She explained that she had killed a mosquito. (C) every

Ans - C (D) any


Ans - A Question No.19
Question No.15 Fill in the blank with suitable tense.
Fill in the blank with appropriate verb in agreement If he had cancelled the ticket, I___________ so
with its subject. much.
Science ______________ Jack's favorite subject, (A) would worry
while social is Jenny's favorite subject.
(B) will have worried
(A) have
(C) have worried
(B) is
(D) would have worried
(C) are
Ans - D
(D) does
Question No.20
Ans - B
Fill in the blanks with suitable Article from the
Question No.16 given alternatives.
The baker made his musical entry on the scene I have ____________ umbrella in my bag.
with the ‘Jhang jhung’ sound of _________
(A) a
specially made bamboo staff.
(B) an
(A) them
(C) the
(B) us
(D) No article
(C) they
Ans - B
(D) his
Question No.1
Ans - D
Find the correct active voice for the given sentence
Question No.17 in passive voice from the given
Water _______ at 100 degree celsius. alternatives.
(A) boiled These jewels were designed by her.
(B) will be boiled (A) She designs these jewels.
(C) boils (B) She designed these jewels.
(D) should boil (C) She has designed these jewels.
Ans - C (D) She had designed these jewels.
Question No.18 Ans - B
He didn't write his homework ____ he was Question No.2
fractured.
Fill in the blanks with suitable Preposition from the
(A) whereas given alternatives.
(B) because My father cried __________ my graduation.
(C) due to (A) with
(D) while (B) beside
Ans - B (C) since
(D) during Ans - A
Ans - D Question No.12
Question No.3 Fill in the blanks with suitable Article from the
given alternatives.
Where is ___ bike?
Goa is ____________ beautiful State in India.
(A) their
(A) the
(B) all
(B) No article
(C) this
(C) a
(D) your
(D) an
Ans - D
Ans - C
Question No.9
Question No.13
You ___ prove him wrong, as you have no
evidences. If you _____ on time, you will see the president.
(A) should (A) were
(B) can't (B) have
(C) ought to (C) will be
(D) will (D) are
Ans - B Ans - D
Question No.10 Question No.14
Fill in the blanks with suitable Article from the In the following question, one part of the sentence
given alternatives. may have an error. Find out which
Lionel Messi wins best FIFA player of part of the sentence has an error and select the
____________ year. option corresponding to it. (Avoid
(A) an punctuation errors)
(B) a (A) Pradhu alongwith her six relatives / (B) are
coming to pay the amount / (C) for the
(C) No Article
gift. / (D) NO ERROR
(D) the
(A) C
Ans - D
(B) A
Question No.11
(C) B
Fill in the blank with suitable tense.
(D) D
Major cities in India _________ significantly over
the last few years. Ans - C
(A) have been polluted Question No.15
(B) has polluted Fill in the blank with suitable tense.
(C) are polluted Raj usually __________ his dog for a walk every
evening.
(D) has been polluted
(A) take (D) over
(B) takes Ans - B
(C) is taking Question No.19
(D) will take Find the correct active/ passive voice for the given
sentence in passive/ active voice
Ans - B
from the given alternatives.
Question No.16
They will pack and send the goods.
Convert the given direct/ indirect speech to
indirect/ direct speech with the help of the (A) The goods will be packed and sent by them.
given options. (B) The goods would be packed and sent by them.
She said to her niece, "I know where is everyone". (C) The goods will packed and sent by them.
(A) She told her niece that she know where was (D) The goods are packed and sent by them.
everyone.
Ans - A
(B) She told her niece that she knew where is
Question No.20
everyone.
____ her physical illness, Ananya achieved in sports
(C) She told her niece that she know where is
with the support of her parent.
everyone.
(A) In spite
(D) She told her niece that she knew where was
everyone. (B) Despite
Ans - D (C) Though
Question No.17 (D) In addition
That day had come about through the Ans - B
unimaginable sacrifices of thousands of
____________ people.
Question No.1
(A) my
Sharp equipments ____ be kept away from kids.
(B) Myself
(A) may
(C) mine
(B) shall
(D) I
(C) must
Ans - A
(D) would
Question No.18
Ans - C
Fill in the blanks with suitable Preposition from the
given alternatives. Question No.2

She feels a lot more confident ___________ Convert the given direct/ indirect speech to
herself. indirect/ direct speech with the help of the

(A) above given options.

(B) to He said to the girl, “Where did you learn western


dance?”
(C) since
(A) He asked the girl where she had learnt western Fill in the blanks with suitable Article from the
dance. given alternatives.
(B) He asks the girl where she had learnt western Regina gave birth to twins in _______ December.
dance.
(A) an
(C) He asked the girl where she has learnt western
(B) the
dance.
(C) a
(D) He ask the girl where she has learnt western
dance. (D) No article
Ans - A Ans - D
Question No.3 Question No.7
Find the correct active/ passive voice for the given Fathima sings very well; ____, she has a sweet
sentence in passive/ active voice voice.
from the given alternatives. (A) eventhough
Crows are chasing away cuckoos. (B) indeed
(A) Cuckoos were being chased away by crows. (C) instead
(B) Cuckoos are being chase away by crows. (D) although
(C) Cuckoos are being chased away by crows. Ans - B
(D) Cuckoos are chased away by crows. Question No.8
Ans - C Fill in the blank with suitable tense.
Question No.4 People will consider everything before they
__________.
Fill in the blanks with suitable Preposition from the
given alternatives. (A) voted
The chief decided _________ meet all the officials. (B) voting
(A) till (C) vote
(B) for (D) will be voted
(C) during Ans - C
(D) to Question No.9
Ans - D If she was there, you would _______ the better
results.
Question No.5
(A) saw
She was waiting for ___ cab.
(B) see
(A) few
(C) seeing
(B) a
(D) have seen
(C) any
Ans - B
(D) the
Question No.10
Ans - A
Question No.6
Fill in the blanks with suitable Article from the (B) have needed
given alternatives.
(C) will be needing
He came direct from ________ Brazil.
(D) needed
(A) an
Ans - A
(B) a
Question No.19
(C) the
Find the correct active voice for the given sentence
(D) No article in passive voice from the given
Ans - D alternatives.
Question No.11 This ring will be treasured as a keepsake by her.
In the following question, one part of the sentence (A) She would treasure this ring as a keepsake.
may have an error. Find out which
(B) She will treasure this ring as a keepsake.
part of the sentence has an error and select the
(C) She will treasure these ring as a keepsake.
option corresponding to it. (Avoid
(D) She treasures this ring as a keepsake.
punctuation errors)
Ans - B
(A) Prema alongwith her parents / (B) is coming to
pay the amount / (C) for the course. / Question No.20
(D) NO ERROR Fill in the blanks with suitable Preposition from the
given alternatives.
(A) B
The tables were lined up ______________ the wall.
(B) C
(A) since
(C) A
(B) about
(D) D
(C) of
Ans - D
(D) along
Question No.12
Ans - D
There was a man ________ had four sons. He
wanted his sons to learn not to judge things
too quickly. Question No.1
(A) those Fill in the blanks with suitable Article from the
given alternatives.
(B) that
____________ elephant is roaming near the
(C) who
agricultural fields.
(D) whom
(A) No article
Ans - C
(B) The
Question No.13
(C) An
Fill in the blank with suitable tense.
(D) A
I have been studying for the past 2 hours. I
Ans - C
________ rest.
Question No.2
(A) need
_______ cannot judge a person in a single Ans - B
encounter.
Question No.6
(A) My
Find the correct active/ passive voice for the given
(B) We sentence in passive/ active voice
(C) Me from the given alternatives.
(D) Us The farmers were receiving government's
subsidies.
Ans - B
(A) Government's subsidies were being received by
Question No.3
the farmers.
Fill in the blank with suitable tense.
(B) Government's subsidies are being received by
He __________ for Indian cricket team last year. the farmers.

(A) to play (C) Government's subsidies were received by the


farmers.
(B) have played
(D) Government's subsidies were not being
(C) play received by the farmers.
(D) played Ans - A
Ans - D Question No.7
Question No.4 Fill in the blank with suitable tense.
Choose a color, ____ you want. While I was watching TV, I _________ a call.
(A) more (A) got
(B) each (B) had got
(C) this (C) gotten
(D) whichever (D) was getting
Ans - D Ans - A
Question No.5 Question No.8
Convert the given direct/ indirect speech to I _____ go back to the shop immediately, I think I
indirect/ direct speech with the help of the left my mobile there.
given options. (A) might
The poor wage worker said, "O God, have mercy on (B) would
my soul".
(C) have to
(A) The poor wage worker has prayed to God to
have mercy on his soul. (D) may

(B) The poor wage worker prayed to God to have Ans - C


mercy on his soul.
Question No.9
(C) The poor wage worker prays to God to have
I _________ to an audio book, when you came
mercy on his soul.
home.
(D) The poor wage worker prayed to God for have
(A) was listening
mercy on his soul.
(B) am listening Question No.13
(C) should be listening Find the correct active/ passive voice for the given
sentence in passive/ active voice
(D) will be listening
from the given alternatives.
Ans - A
The desserts have been prepared by her.
Question No.10
(A) She has not prepared the desserts.
Ganesh respects his wife, _____ he often disagrees
with her. (B) She has prepared the desserts.
(A) eventhough (C) She have prepared the desserts.
(B) in addition (D) She had prepared the desserts.
(C) meanwhile Ans - B
(D) because Question No.14
Ans - A Fill in the blanks with suitable Article from the
given alternatives.
Question No.11
Kavi is ready to hire __________ shortlisted
In the following question, one part of the sentence
candidates.
may have an error. Find out which
(A) a
part of the sentence has an error and select the
option corresponding to it. (Avoid (B) No article
punctuation errors) (C) an
(A) Every man and woman / (B) was ready to (D) the
attend / (C) the event. / (D) NO ERROR
Ans - D
(A) A
Question No.15
(B) D
Fill in the blanks with suitable Preposition from the
(C) B given alternatives.
(D) C I am living ____________ my parents.
Ans - B (A) with
Question No.12 (B) since
Fill in the blanks with suitable Preposition from the (C) through
given alternatives.
(D) at
England skipper Joe Root is excited to bat
Ans - A
__________ number three in the Ashes cricket
series.
Question No.1
(A) in
____ should wear uniform.
(B) over
(A) Anyone
(C) at
(B) Somebody
(D) on
(C) Everyone
Ans - C
(D) Anybody (D) the
Ans - C Ans - C
Question No.2 Question No.6
Fill in the blanks with suitable Preposition from the Find the correct active/ passive voice for the given
given alternatives. sentence in passive/ active voice
Our Headmaster asked the chief guest __________ from the given alternatives.
distribute the sweets.
They will have released the movie next year.
(A) for
(A) The movie will have been released by them
(B) until next year.
(C) since (B) The movie would have been released by them
next year.
(D) to
(C) The movie will have released by them next
Ans - D
year.
Question No.3
(D) The movie will be released by them next year.
Fill in the blank with suitable tense.
Ans - A
Uday __________ here until July 25 last year.
Question No.7
(A) will work
Fill in the blank with suitable tense.
(B) would work
He ___________ hard to complete the assignment
(C) has been working since January 2000.

(D) worked (A) has worked

Ans - D (B) have been working

Question No.4 (C) had worked

It was a boring movie. ______, I had fun with my (D) has been working
friends.
Ans - D
(A) although
Question No.8
(B) neverthless
Idleness is a great vice and a great danger.
(C) eventhough __________ is the father of all evils.

(D) instead (A) What

Ans - B (B) When

Question No.5 (C) It

Fill in the blanks with suitable Article from the (D) There
given alternatives.
Ans - C
India achieved independence in _________ 1947.
Question No.9
(A) a
You _______ this task for the well-being of the
(B) an poor children. Won't you?

(C) No article (A) accomplish


(B) accomplished Question No.13
(C) will accomplish Convert the given direct/ indirect speech to
indirect/ direct speech with the help of the
(D) will have accomplished
given options.
Ans - C
Rajan remarked “What a wonderful day!”
Question No.10
(A) Rajan exclaimed that it is a wonderful day
They were hopeful. She ______ attend the
wedding. (B) Rajan exclaimed that it was a wonderful day
(A) ought (C) Rajan requested that it was a wonderful day
(B) may (D) Rajan exclaimed to that it is a wonderful day
(C) must to Ans - B
(D) need to Question No.19
Ans - B Fill in the blanks with suitable Article from the
given alternatives.
Question No.11
I am going to see _________ Statue of Liberty this
In the following question, one part of the sentence
year.
may have an error. Find out which
(A) No article
part of the sentence has an error and select the
option corresponding to it. (Avoid (B) the
punctuation errors) (C) an
(A) The Director, with / (B) all his employees, were (D) a
/ (C) fired from the organization. / (D)
Ans - B
NO ERROR
Question No.20
(A) B
Find the correct active voice for the given sentence
(B) D in passive voice from the given
(C) A alternatives.
(D) C She had just closed the data file.
Ans - A (A) The data file had just been closed by her.
Question No.12 (B) The data file has just been closed by her.
Fill in the blanks with suitable Preposition from the (C) The data file had not just been closed by her.
given alternatives.
(D) The data file have just been closed by her.
We went for a walk ________________ the beach.
Ans - A
(A) between
Question No.1
(B) above
_____ prove them wrong, She works hard.
(C) along
(A) In contrast
(D) about
(B) In addition
Ans - C
(C) In order to
(D) Despite (D) sets
Ans - C Ans - D
Question No.2 Question No.6
Find the correct active/ passive voice for the given Fill in the blanks with suitable Article from the
sentence in passive/ active voice given alternatives.
from the given alternatives. The Hindu is _________ ninth-most-circulated
newspaper in India.
Call the police.
(A) the
(A) Let the police be called.
(B) an
(B) Let the police been called.
(C) a
(C) Let the police be call.
(D) No Article
(D) Let the police called.
Ans - A
Ans - A
Question No.7
Question No.3
Now-a days, Mother’s Day celebrations are held
Fill in the blanks with suitable Article from the
throughout the world though __________
given alternatives.
may not fall at the same time.
All the people are ready to watch _________
cricket match between India and Australia. (A) they
(A) No article (B) whichever
(B) the (C) those
(C) an (D) them
(D) a Ans - A
Ans - B Question No.8
Question No.4 Fill in the blanks with suitable Preposition from the
given alternatives.
Fill in the blank with suitable tense.
There are approximately 80 different types
His uncle does not _________ the impact.
____________ autoimmune diseases and they
(A) understands
affect more than 23 million Americans.
(B) understand
(A) of
(C) understanding
(B) with
(D) understood
(C) from
Ans - B
(D) for
Question No.5
Ans - A
The sun ______ in the west.
Question No.9
(A) will set
She asked for ____ topic to write a poem.
(B) setting
(A) another
(C) set
(B) both Question No.18
(C) no Fill in the blanks with suitable Preposition from the
given alternatives.
(D) each
My sister lives ________ the building at the end of
Ans - A
the river view road.
Question No.10
(A) over
Fill in the blank with suitable tense.
(B) in
My brother wouldn't __________ anything like
(C) across
that.
(D) since
(A) doing
Ans - B
(B) does
Question No.19
(C) do
In the following question, one part of the sentence
(D) did
may have an error. Find out which
Ans - C
part of the sentence has an error and select the
Question No.16 option corresponding to it. (Avoid

Find the correct active voice for the given sentence punctuation errors)
in passive voice from the given
(A) One must finish one's / (B) commitments in
alternatives. time to / (C) avoid the future issues. / (D)

The course will have been completed by them next NO ERROR


year.
(A) A
(A) They will have complete the course next year.
(B) B
(B) They will completed the course next year.
(C) D
(C) They will have completed the course next year.
(D) C
(D) They would have completed the course next
Ans - C
year.
Question No.20
Ans - C
She _____ report at the head office today before 5
Question No.17
p.m.
Convert the given direct/ indirect speech to
(A) have to
indirect/ direct speech with the help of the
(B) had do
given options.
(C) needs to
Mona said, "I am very busy now."
(D) need to
(A) Mona said that he was very busy then
Ans - C
(B) Mona said to that he was very busy then
Question No.6
(C) Mona said that he was very busy now
Choose the correct article to complete the
(D) Mona said that he is very busy then
following sentence.
Ans - A
They are ready to receive ___________ maths (A) May
question paper for the annual exam.
(B) Shouldn't
(A) an
(C) Could
(B) the
(D) Should
(C) a
Ans - C
(D) No article
Question No.11
Ans - B
Fill in the blank with suitable tense.
Question No.7
I have never ___________ such a mess.
Latha and her sister resemble ___.
(A) seeing
(A) each other
(B) seen
(B) each one
(C) been seeing
(C) every other
(D) saw
(D) anyone
Ans - B
Ans - A
Question No.12
Question No.8
Nothing __________ the same forever.
The teacher asked ___ the class is ready for the
(A) remains
test.
(B) remain
(A) whether
(C) will be remaining
(B) also
(D) remained
(C) that
Ans - A
(D) unless
Question No.13
Ans - A
Soybeans belong to the legume family. The beans
Question No.9
are the seeds of the leguminous
Choose the best preposition from the given
soybeans plants. ____________ can be grown on a
alternatives for the blank in the given
variety of soils and in a wide range of
question.
climates.
Mother placed several cups _____________ the
(A) They
kitchen table.
(B) What
(A) for
(C) There
(B) over
(D) Them
(C) besides
Ans - A
(D) on
Question No.14
Ans - D
Fill in the blank with suitable tense.
Question No.10
The editor will ____________ editing this
____ you please tell me the address?
newspaper for 8 years by next January.
(A) had (D) They would not have been insisted by you.
(B) have been Ans - D
(C) has been Question No.18
(D) have Fill in the blanks with suitable Article from the
given alternatives.
Ans - B
Janu does not like ______ pet animals, but she likes
Question No.15
her cousin's parrot.
Fill in the blanks with suitable Preposition from the
(A) a
given alternatives.
(B) an
India will qualify for the semi-finals with a victory
_________ Bangladesh. (C) the
(A) through (D) No article
(B) over Ans - D
(C) along Question No.19
(D) for In the following question, one part of the sentence
may have an error. Find out which
Ans - B
part of the sentence has an error and select the
Question No.16
option corresponding to it. (Avoid
Convert the given direct/ indirect speech to
punctuation errors)
indirect/ direct speech with the help of the
(A) One must finish one's / (B) commitments in
given options.
time / (C) for avoid the future issues. /
Siva said, “I bought a truck yesterday.”
(D) NO ERROR
(A) Siva said that he had bought a truck yesterday.
(A) A
(B) Siva said that he has bought a truck the
(B) D
previous day.
(C) C
(C) Siva said that he has buy a truck the previous
day. (D) B
(D) Siva said that he had bought a truck the Ans - C
previous day.
Question No.20
Ans - D
Find the correct active voice for the given sentence
Question No.17 in passive voice from the given
Find the correct active/ passive voice for the given alternatives.
sentence in passive/ active voice
They could identify the man.
from the given alternatives.
(A) The man cannot identified by them.
You would not have insisted them.
(B) The man could be identified by them.
(A) They would have been insisted by you.
(C) The man can be identified by them.
(B) They would not have insist by you.
(D) The man cannot be identified by them.
(C) They would not have insisted by you.
Ans - B
Question No.1 Ans - D
Fill in the blanks with suitable Article from the Question No.10
given alternatives.
Fill in the blank with suitable tense.
I am going to commute with ___________ help of
Steel ____________ strong when nickel is added.
my friend.
(A) is becoming
(A) an
(B) will became
(B) No article
(C) become
(C) a
(D) becomes
(D) the
Ans - D
Ans - D
Question No.11
Question No.2
Rani ___return home before it gets dark.
Kisa Gotami became weary and hopeless, and sat
down at the wayside watching the (A) would
lights of the city, as __________ flickered up and (B) has to
were extinguished again.
(C) might
(A) they
(D) shall
(B) whose
Ans - B
(C) those
Question No.12
(D) them
Find the correct active/ passive voice for the given
Ans - A sentence in passive/ active voice
Question No.3 from the given alternatives.
Choose the correct article to complete the Let the food be served.
following sentence.
(A) Serves the food.
Russian policies are thought by many to be among
_________ best in the world. (B) Serve the food.

(A) the (C) Let Serve the food.

(B) No article (D) Served the food.

(C) a Ans - B

(D) an Question No.13

Ans - A She found him ____ in the village.

Question No.9 (A) everywhere

They ______ just ______ the hall. (B) anywhere

(A) have been; leaving (C) nowhere

(B) had; left (D) none

(C) has; left Ans - C

(D) have; left Question No.14


Fill in the blank with suitable tense. Question No.18
She __________ a popular actor. In the following question, one part of the sentence
may have an error. Find out which
(A) resemble
part of the sentence has an error and select the
(B) resembles
option corresponding to it. (Avoid
(C) is resembling
punctuation errors)
(D) will resemble
(A) You, Mary and / (B) me have / (C) completed
Ans - B our work. / (D) NO ERROR

Question No.15 (A) B

Fill in the blanks with suitable Preposition from the (B) D


given alternatives.
(C) A
I fell asleep ____________ the drama.
(D) C
(A) since
Ans - A
(B) for
Question No.19
(C) on
Convert the given direct/ indirect speech to
(D) during indirect/ direct speech with the help of the

Ans - D given options.

Question No.16 They said, "The girl is hiding in the place where we
left her."
Don't help your friend copy your paper in the
exams. ___ help them when you prepare for (A) They said that the girl was hiding in the place
where they had left her.
exams.
(B) They said that the girl was being hiding in the
(A) In the same way place where they had left her.
(B) Meanwhile (C) They said that the girl has been hiding in the
(C) Instead place where they had left her.

(D) In addition (D) They said that the girl was hiding in the place
where they have left her.
Ans - C
Ans - A
Question No.17
Question No.20
Fill in the blanks with a suitable Preposition from
the given alternatives. Find the correct active voice for the given sentence
in passive voice from the given
The girl who sat ____________ him was her
daughter. alternatives.

(A) since Do not leave the room.

(B) between (A) Let the room not be left.

(C) in (B) Let the room be left.

(D) beside (C) Let the rooms not been leave.

Ans - D (D) Let the room not be leave.


Ans - A (C) below
Question No.6 (D) above
Find the correct active/ passive voice for the given Ans - B
sentence in passive/ active voice
Question No.10
from the given alternatives.
I met ____ of the two friends.
Healthy food nourishes children.
(A) nobody
(A) Children nourished by healthy food.
(B) neither
(B) Children are not being nourished by healthy
(C) few
food.
(D) no one
(C) Children are nourished by healthy food.
Ans - B
(D) Children are being nourished by healthy food.
Question No.11
Ans - C
In the following question, one part of the sentence
Question No.7
may have an error. Find out which
Fill in the blanks with suitable Preposition from the
part of the sentence has an error and select the
given alternatives.
option corresponding to it. (Avoid
We need to walk ____________ this forest.
punctuation errors)
(A) on
(A) Bijesh wastes not / (B) only money / (C) and
(B) to also his life / (D) NO ERROR
(C) since (A) D
(D) through (B) C
Ans - D (C) B
Question No.8 (D) A
Fill in the blank with suitable tense. Ans - B
I ____________ in buying the product. Question No.12
(A) have interested Fill in the blank with suitable tense.
(B) will interest He ________ 5 different languages.
(C) had been interested (A) is speaking
(D) am interested (B) will have being speaking
Ans - D (C) speaks
Question No.9 (D) has speak
Fill in the blanks with a suitable Preposition from Ans - C
the given alternatives.
Question No.13
I wear a ring __________ my finger.
You ____ reject me without a proper reason.
(A) at
(A) can't
(B) on
(B) must to
(C) should Ans - C
(D) ought Question No.17
Ans - A If we look at life closely, we will realize how
beautiful _________ is.
Question No.14
(A) itself
Fill in the blank with the appropriate article a, an,
or the, or select the option ‘No article’ if (B) it
no article is needed. (C) my
The Taj Mahal is one of __________ World’s (D) who
wonder.
Ans - B
(A) No article
Question No.18
(B) a
Fill in the blanks with suitable Article from the
(C) the given alternatives.
(D) an He is going for __________ cup of coffee.
Ans - C (A) the
Question No.15 (B) No article
Everything was discussed in the meeting. ___ the (C) an
hike in the salary, there will be an
(D) a
official announcement tomorrow.
Ans - D
(A) Meanwhile
Question No.19
(B) Also
_______ that we have already registered our
(C) Regarding names, the security didn't ask for more
(D) Since details.
Ans - C (A) To Notice
Question No.16 (B) Having been noticed
Convert the given direct/ indirect speech to (C) Noticed
indirect/ direct speech with the help of the
(D) Noticing
given options.
Ans - D
The officer said, "I shall finish this task in time."
Question No.20
(A) The officer said that he should have finished
Find the correct active voice for the given sentence
that task in time.
in passive voice from the given
(B) The officer said that he will finish that task in
alternatives.
time.
This app will fetch the data.
(C) The officer said that he should finish that task in
time. (A) The data will fetched by this app.
(D) The officer said that he should finished that (B) The data would fetched by this app.
task in time.
(C) The data will be fetched by this app.
(D) The data would fetch this app. (D) about
Ans - C Ans - D
Question No.4
OSSC SCEW Exam 2022 Find the correct active/ passive voice for the given
sentence in passive/ active voice
Question No.1
from the given alternatives.
In the following question, one part of the sentence
may have an error. Find out which She will be given a chance by them.
part of the sentence has an error and select the (A) They would give her a chance.
option corresponding to it. If the
(B) They gave her a chance.
sentence does not have any error then select the
(C) They will give her a chance.
option 'NO ERROR'.(Avoid punctuation
(D) They give her a chance.
errors)
Ans - C
(A) Ram and myself / (B) are going to visit / (C) the
college on Sunday. / (D) NO ERROR. Question No.5
(A) A Replace the underlined phrase grammatically and
conceptually with the help of the given
(B) C
options. If the given sentence is correct then select
(C) D
the option 'The given sentence is
(D) B
correct'.
Ans - A
The gender wage gap are a measure of what male
Question No.2 employees are paid in comparison to
Find the correct 'indirect speech' for the given female employees.
sentence in 'direct speech' from the given
(A) is a measuring of what male employees is
alternatives.
(B) is a measure of what male employees is
He says, “I eat an apple a day.”
(C) is a measure of what male employees are
(A) He says that he eats an apple a day.
(D) The given sentence is correct
(B) He explained that he eats an apple a day.
Ans - C
(C) He demanded that he eats an apple a day.
Question No.6
(D) He told that he eat an apple a day.
In the following question, one part of the sentence
Ans - A may have an error. Find out which
Question No.3 part of the sentence has an error and select the
option corresponding to it. If the
Fill in the blanks with suitable Preposition from the
given alternatives. sentence does not have any error then select the
option 'NO ERROR'.(Avoid punctuation
Susan and I agree ____________ the book.
errors)
(A) under
(A) How long has / (B) you been working / (C) in
(B) beneath
this office?/ (D) NO ERROR.
(C) near
(A) B R: the spotlight in 2017, when
(B) D S: his vector work of Lord Hanuman
(C) A (A) RSPQ
(D) C (B) PSRQ
Ans - C (C) QSRP
Question No.7 (D) PRSQ
Fill in the blanks with suitable Preposition from the Ans - D
given alternatives.
Question No.10
The chief guest asked the teachers to tie a ribbon
The car_____ in my family for 12 years.
____________ the gift box.
(A) had been
(A) beyond
(B) was
(B) around
(C) has been
(C) since
(D) being
(D) between
Ans - C
Ans - B
Question No.11
Question No.8
Replace the underlined phrase grammatically and
Rearrange the following to form a meaningful
conceptually with the help of the given
sentence and find the most logical order
options. If the given sentence is correct then select
from the given options.
the option 'The given sentence is
P: if you are in the mood for some indulgence
correct'.
Q: this weekend, look no further for we bring you
The irresponsible and careless aspirant was living a
the
aimless life by living at home with
R: sandwich and pasta in one dish
his guardian without trying to get a job.
S: perfect blend of creamy and cheesy
(A) were living an aimless life by living at
(A) RSPQ
(B) was living an aimless life by living about
(B) SRPQ
(C) was living an aimless life by living at
(C) PQSR
(D) The given sentence is correct
(D) PSQR
Ans - C
Ans - C
Question No.12
Question No.9
Fill in the blanks with suitable Article from the
Rearrange the following to form a meaningful given alternatives.
sentence and find the most logical order
Rajesh is __________ lionhearted person.
from the given options.
(A) a
P: the graphic artist came into
(B) an
Q: became a sensational hit
(C) the
(D) No article from the given alternatives.
Ans - A She will have taken them to her home.
Question No.13 (A) They will have taken by her to her home.
Have you _____ your work? (B) They would have taken by her to her home.
(A) had completed (C) They will have been taken by her to her home.
(B) completing (D) They will been taken by her to her home.
(C) has completed Ans - C
(D) completed Question No.17
Ans - D Guru _______ English everyday.
Question No.14 (A) studied
Find the correct 'direct speech' for the given (B) was studying
sentence in 'indirect speech' from the given
(C) had studied
alternatives.
(D) studies
Ramya asked Shwetha, why she was painting on
Ans - D
the wall.
Case Study - 18 to 19
(A) Ramya said to Shwetha, ‘Why are you painting
on the wall?’ Directions: Read the passage and fill the blanks
with suitable pronoun.
(B) Ramya said that to Shwetha, ‘Why are you
painting on the wall?’ Question No.18
(C) Ramya said that Shwetha, ‘Why are you A. The poem 'Twinkle Twinkle little star' was first
painting on the wall?’ published with the title ‘The Star’ in
(D) Ramya said to Shwetha, that ‘Why are you 1806 in Rhymes for the Nursery, a collection of
painting on the wall?’ poems by Jane Taylor and her sister Ann
Ans - A Taylor. ____(A)____ lines have been written from
the perspective of a young child who
Question No.15
wonders and addresses the star directly to ask
Fill in the blanks with suitable Article from the
what ____(B)____ is.
given alternatives.
(A) That
Athletics is ________ Olympic Sport.
(B) It
(A) a
(C) Those
(B) an
(D) This
(C) the
Ans - C
(D) No Error
Question No.19
Ans - B
B. The poem 'Twinkle Twinkle little star' was first
Question No.16
published with the title ‘The Star’ in
Find the correct active/ passive voice for the given
1806 in Rhymes for the Nursery, a collection of
sentence in passive/ active voice
poems by Jane Taylor and her sister Ann
Taylor. ____(A)____ lines have been written from Question No.3
the perspective of a young child who
Find the correct active/ passive voice for the given
wonders and addresses the star directly to ask sentence in passive/ active voice
what ____(B)____ is.
from the given alternatives.
(A) this
She might create problems.
(B) those
(A) Problems may be created by her. (Chosen
(C) that option)
(D) it (B) Problems might created by her.
Ans - D (C) Problems might be created by her. (Correct
Answer)
(D) Problems will be created by her.
Question No.1
Ans - C
Find the correct 'indirect speech' for the given
sentence in 'direct speech' from the given Question No.4
alternatives. The boss ______ my salary.
He said to me, "Please, shut the door." (A) raised (Correct Answer)
(A) He told to shut the door. (B) aroused
(B) He requested me to shut the door. (Correct (C) rose (Chosen option)
Answer) (Chosen option)
(D) arose
(C) He requested to me to shut the door.
Ans - A
(D) He questioned me to shut the door.
Question No.5
Ans - B
Replace the underlined phrase grammatically and
Question No.2 conceptually with the help of the given
In the following question, one part of the sentence options. If the given sentence is correct then select
may have an error. Find out which the option 'The given sentence is
part of the sentence has an error and select the correct'.
option corresponding to it. If the
The boy was puzzled by his recent weight gain
sentence does not have any error then select the since he exercised regularly and ate
option 'NO ERROR'.(Avoid punctuation
healthy meals.
errors)
(A) were puzzled by his recent weight gain since he
(A) Every boy and girl / (B) were ready to attend /
(B) was puzzle by his recent weight gain since he
(C) the function / (D) NO ERROR.
(C) were puzzling by his recent weight gain since he
(A) B (Correct Answer)
(D) The given sentence is correct (Correct Answer)
(B) C
(Chosen option)
(C) A (Chosen option)
Ans - D
(D) D
Question No.6
Ans - A
Fill in the blanks with suitable Preposition from the (B) an
given alternatives.
(C) the (Correct Answer) (Chosen option)
The rumors spread _____________ the country.
(D) No Error
(A) between
Ans - C
(B) along
Question No.10
(C) across (Correct Answer) (Chosen option)
Rearrange the following to form a meaningful
(D) below sentence and find the most logical order
Ans - C from the given options.
Question No.7 P: the funds will be given to
Find the correct 'direct speech' for the given Q: the start-ups selected
sentence in 'indirect speech' from the given
R: knowledge partners
alternatives.
S: by different
You requested your mother to grant her leave for
(A) PQSR (Correct Answer) (Chosen option)
some time.
(B) SRPQ
(A) You said your mother, “Please grant her leave
for some time.” (Chosen option) (C) PSQR
(B) You said to your mother, “Please grant her (D) SQRP
leave for some time.” (Correct
Ans - A
Answer)
Question No.11
(C) You said to your mother, “grant her leave for
some time.” In the following question, one part of the sentence
may have an error. Find out which
(D) You told that your mother, “Please grant her
leave for some time.” part of the sentence has an error and select the
option corresponding to it. If the
Ans - B
sentence does not have any error then select the
Question No.8 option 'NO ERROR'.(Avoid punctuation
I _____ rent yet. errors)
(A) paid (A) Ram and I / (B) is going to visit / (C) the college
on Sunday. / (D) NO ERROR.
(B) is paying
(A) A
(C) haven't paid (Correct Answer) (Chosen option)
(B) B (Correct Answer)
(D) was paid
(C) C
Ans - C
(D) D (Chosen option)
Question No.9
Ans - B
Fill in the blanks with suitable Article from the
given alternatives. Question No.12
I am going to see _________ Statue of Liberty this Fill in the blanks with suitable Article from the
year. given alternatives.
(A) a
The crash of _______ Ethiopian Airlines passenger This cannot be set right by them so easily.
jet bound for Nairobi on Sunday raised
(A) They cannot be set this right so easily. (Chosen
questions about the safety of the Boeing aircrafts. option)
(A) an (Correct Answer) (Chosen option) (B) They can set this right so easily.
(B) No article (C) They can be set this right so easily.
(C) the (D) They cannot set this right so easily. (Correct
Answer)
(D) a
Ans - D
Ans - A
Question No.16
Question No.13
Rearrange the following to form a meaningful
Replace the underlined phrase grammatically and
sentence and find the most logical order
conceptually with the help of the given
from the given options.
options. If the given sentence is correct then select
the option 'The given sentence is P: human body is known
correct'. Q: biological clock that
Once a new business has been started, the owner R: to have an in-built
will need to retail those products on
S: is genetically driven
their own website.
(A) RQPS
(A) Once an new business has been started
(B) PSQR
(Chosen option)
(C) RSQP
(B) Once a new business has been start
(D) PRQS (Correct Answer) (Chosen option)
(C) Once an new business has been start
Ans - D
(D) The given sentence is correct (Correct Answer)
Question No.17
Ans - D
When I first met him, he_____costly gold chain.
Question No.14
(A) is wearing
Fill in the blanks with suitable Preposition from the
given alternatives. (B) wore (Chosen option)
The squad found a bomb ___________ the truck. (C) was wearing (Correct Answer)
(A) underneath (Correct Answer) (Chosen option) (D) wears
(B) against Ans - C
(C) for Case Study - 18 to 19 2.00
(D) to Directions: Read the passage and fill the blanks
with suitable pronoun.
Ans - A
Question No.18
Question No.15
B. It’s dark. It is always dark these days. Lights in
Find the correct active/ passive voice for the given
the sky burn your eyes, so it is
sentence in passive/ active voice
advisable to keep ____(A)____ away from the light
from the given alternatives.
or keep your face to ground in the
hopes that they’ll go away. But they don’t. The air (C) exclusive
is heavy and poisonous to breathe. so
(D) elusive
it is better to wear a mask at all times. ____(B)____
Ans - C
are tough rules to follow.
63) My Uncle Samuel _____ (be) a source of
(A) Their (Chosen option)
inspiration for me since I was a young boy.
(B) That
(A) was being
(C) This
(B) will be
(D) Those (Correct Answer)
(C) is being
Ans - D
(D) has been
Question No.19
Ans - D
A. It’s dark. It is always dark these days. Lights in
64) The dog looks ferocious. You _____ go near it.
the sky burn your eyes, so it is
(Choose the correct modal).
advisable to keep ____(A)____ away from the light
(A) wouldn’t
or keep your face to ground in the
(B) needn’t
hopes that they’ll go away. But they don’t. The air
is heavy and poisonous to breathe. so (C) won’t
it is better to wear a mask at all times. ____(B)____ (D) mustn’t
are tough rules to follow.
Ans - D
(A) your
65) The antonym of ‘Acumen’ is _________.
(B) these (Chosen option)
(A) Craziness
(C) that
(B) Stupidity
(D) yours (Correct Answer)
(C) Brilliance
Ans - D
(D) Lethargy
Ans - B
OPRB SI Exam 2019
66) Choose the option that is opposite in meaning
61) Wars always bring ______ consequences for to the word ‘Adherent.’
the nation. (Choose the best option).
(A) Advocate
(A) prosperous
(B) Adversary
(B) extensive
(C) Disciple
(C) poor
(D) Follower
(D) disastrous
Ans - B
Ans - D
67) The antonym of ‘Conspicuous’ is _________.
62) This is an ________ product for the fashionable
youngsters. (Choose the best option). (A) Unpopular

(A) extensive (B) Unclear

(B) allusive (C) Indistinct


(D) Lost
Ans - C (D) Make people feel more comfortable
68) Fill in the blanks with the appropriate article to Ans - C
be used. Mark 'No article required' if there is no
73) A displayed sheet giving news or information is
need to
called?
use the article in the blank. Compliance of
(A) An office order
_________ seat belts in the rear seat is poor in
India. (B) A note sheet
(A) A (C) A notice
(B) An (D) A report
(C) The Ans - C
(D) No article required 74) No woman had ever had the honour of a
university chair at the Sorbonne ________ Marie,
Ans - D
who was the
69) London is____ European city. (Choose the best
only one that could hold that position.
option).
(A) after
(A) A
(B) until
(B) An
(C) by the time
(C) The
(D) when
(D) No article
Ans - A
Ans - A
75) A plan for carrying out a process or procedure,
70) ‘See eye to eye’ means _______.
giving lists of intended events and times.
(A) To look for someone.
(A) Compile
(B) To look lovingly at someone.
(B) Catalogue
(C) To agree with someone
(C) Schedule
(D) To keep an eye on someone.
(D) Inscribe
Ans - C
Ans - C
71) The meaning of the expression ‘To let someone
76) A short stay at a place
off the hook’ is _________.
(A) Voyage
(A) To allow someone not to be punished.
(B) Lodge
(B) To allow someone to catch fish.
(C) Sojourn
(C) To allow someone to release the fish.
(D) Trip
(D) To take the fish off the hook.
Ans - C
Ans - A
77) A person who helps another commit a crime is
72) ‘Break the ice’ means _______.
an ______.
(A) Break the ice cubes
(A) accomplish
(B) Make friends in the first meeting
(B) accomplice
(C) Remove obstacles from the path
(C) associate Ans - B
(D) assistant 83) I ran _____ the street to greet my friend.
(Choose the best option).
Ans - B
(A) along
78) One who talk in sleep (Answer in one word)
(B) across
(A) Celibate
(C) over
(B) Intestate
(D) on
(C) Somniloquy
Ans - A
(D) Gullible
84) Which of the following sentence is
Ans - C
grammatically correct?
79) A solution or remedy for all difficult diseases is
(A) I have gone to Delhi yesterday
_____.
(B) I has gone to Delhi yesterday
(A) Panacea
(C) I went to Delhi yesterday
(B) Elixir
(D) I have went to Delhi yesterday
(C) Ambrosia
Ans - C
(D) Vitamins
85) "Find correct sequence of sentences:
Ans - A
P: advocated complete equality
80) The manager was ____________ an
explanation of his conduct. Q: in 1854, Anthony devoted herself
(A) called for R: between men and women.
(B) called off S: to the rights of women and"
(C) called to (A) QSRP
(D) called up (B) QSPR
Ans - A (C) SPRQ
81) She had been ill ___________ Thursday (D) SRPQ
morning.
Ans - B
(A) since
86) The teacher said, “The Sun rises in the east.”
(B) for (Change into Indirect speech).
(C) on (A)The teacher said that the Sun rises in the east.
(D) none of these (B) The teacher exclaimed that Sun rises in the
east.
Ans - A
(C) The teacher said that Sun rose in the east.
82) Why don't you sit ____________ me?
(D) The teacher said to everybody that Sun will rise
(A) close
in the east.
(B) beside
Ans - A
(C) along
87) I/ have seen/ him/ a moment ago. (Point out
(D) next the part with error).
(A) I 92) Admonish means _________.
(B) have seen (A) Urge
(C) him (B) Force
(D) a moment ago (C) Reprimand
Ans - B (D) Counsel
88) The captain along with his team /are Ans - C
practicing/ very hard for the/ forthcoming
93) Encumbrance means _________.
matches. (Point out the
(A) Difficulty
part with error).
(B) Burden
(A) The captain along with his team
(C) Block
(B) are practicing
(D) Binding
(C) very hard for the
Ans - B
(D) forthcoming matches
94) I find _____ horror movies disgusting. (Choose
Ans - B
the correct option).
89) Choose the correct spelling of the word.
(A) watched
(A) Prejorative
(B) to have watched
(B) Pejorative
(C) to watch
(C) Pejoretive
(D) watching
(D) Pedgorative
Ans - D
Ans - B
95) Select the most appropriate option that can
90) Choose the correct spelling of the word. substitute the underlined segment in the given
sentence. If
(A) Beelligerence
there is no need to substitute it, select ' No
(B) Beligerence
substitution required'. Which movie have you
(C) Belligerence watched on

(D) Belligirence television when I called you?

Ans - C (A) are you watching

91) "Directions: The following sentence has been (B) no substitution required
split into four segments. Identity the segment that
(C) were you watching
contains a grammatical error. Had you / not
reached in time, / we will have / lost our lives." (D) did you watched
(A) Had you Ans - C
(B) lost our lives 96) When I opened my room, I found that
somebody _____ into my room during the night.
(C) not reached in time
(Choose the correct option).
(D) we will have
(A) break
Ans - D
(B) broke
(C) has broken 'one who follows a socially unconventional ways of
life'.
(D) had broken
A. bohemian
Ans - D
B. Socialite
97) The show had already started, when I _____ on
the TV. (Choose the correct option). C. aesthete
(A) switched D. pedagogue
(B) could switch Ans - A
(C) had switched 27. Choose the correct one-word substitute for the
given phrase/expression.
(D) have switched
'a small group of people having control of a
Ans - A
country'
98) She has 'to begin gaining experience' if she
A. democracy
wishes to join the media industry. (Choose the
correct option) B. autocracy
(A) Cost an arm and a leg C. oligarchy
(B) Get one’s feet wet D. bureaucracy
(C) Cross one’s T’s and dots Ans - C
(D) To sit on the fence 28. Choose the correct one-word substitute for the
given phrase/expression.
Ans - B
'someone who can be easily persuaded to believe
99) The umpire declared the batsman out. (Change
something.
into passive voice).
A. credible
(A) The batsman was out declared by the umpire.
B. gullible
(B) The batsman was declared out by the umpire.
C. malicious
(C) The umpire was declared out by the batsman.
D. comprehensible
(D) The batsman was declared to be out by the
umpire. Ans - B
Ans - B 29. Choose the correct one-word substitute for the
given
100) Keep to the left. (Change into passive voice).
'an inactive state resembling deep sleep.
(A) You are 'requested to keep it to the left.
A. lazy
(B) Let the left be kept.
B. hibernation
(C) Let us keep to the left.
C. indolent
(D) You should all keep to the left.
D. loafing
Ans - A
Ans - B
30. Choose the correct one-word substitute for the
OPRB SI Exam 2021
given phrase/expression
26. Choose the correct one-word substitute for the
‘handwriting that is not clear enough to be read':
given phrase/expression.
A. eligible 35. Choose the appropriate meaning of the
following idiom- 'out of the woods’.
B. legible
A. out of work place
C. illegible
B. out of forest area
D. decipherable
C. out of one's comfort zone
Ans - C
D. out of danger area
31. Choose the appropriate meaning of the idiom-
'to die in harness’. Ans - D
A. to die in vain Fill in the blanks by choosing the correct
preposition from among the options given below.
B. to die in honour
36. This book comprises ______ five chapters.
C. to die while in service
A. about
D. to die for the motherland
B. of
Ans - C
C. with
32. Choose the appropriate meaning of the idiom-
'do not hold water'. D. in
A. cannot be believed Ans - A
B. cannot be held safely 37. He was not ashamed ______ asking for even
more money.
C. cannot be carried
A. at
D. a dry river bed
B. from
Ans - A
C. of
33. Choose the appropriate meaning of the idiom-
'beat about the bush’. D. to
A. clear the bush by beating Ans - C
B. hide in the bush in order to escape getting 38. My sister was not required to conform
beaten up ________ the traditional ways of her husband's
C. approach a subject without coming to the point family.
D. beat a bush randomly A. to
Ans - C B. for
34. Choose the appropriate meaning of the C. of
following idiom- "break the ice'.
D. in
A. do or say something to relieve tension when
Ans - A
strangers meet
39. Mr. Nambiar is thoroughly disillusioned
B. to put ice cubes in a glass to prepare sherbet
_______ his job.
C. make alliance with a former enemy
A. from
D. to have a truce with the enemy
B. of
Ans - A
C. with
D. at D. a little
Ans - C Ans - B
40. During economic recession one should avoid 45. We decided to abandon our trip as we had
spending money _______ luxury items. ______ money left.
A. on A. few
B. at B. a few
C. for C. little
D. with D. a little
Ans - A Ans - C
Fill in the blanks by choosing the correct option 46. Abhimanyu was ______ engrossed in his book
from among the ones given below. even to look up.
41. Priyanka Chopra has played the role of A. very
_________ FBI agent in "Quantico."
B. much
A. a
C. too
B. an
D. So
C. the
Ans - C
D. none of the above
47. ______ my brother, who does not like thrillers,
Ans - B enjoyed the film.
42. Each new word has _____ difference meaning. A. Alone
A. a B. Even
B. an C. Only
C. the D. Though
D. none of the above Ans - B
Ans - A Choose the word that is opposite in meaning to
the given word.
43. He is ________ one-eyed man.
48. 'Eulogise'
A. a
A. celebrate
B. an
B. extol
C. the
C. question
D. zero article
D. criticize
Ans - A
Ans - D
44. I have got _________ close friends that I meet
regularly. 49. 'Relish'
A. few A. dislike
B. a few B. gusto
C. little C. savour
D. attack B. pacify
Ans - A C. pamper
50. 'Brittle'. D. promise
A. weak Ans - B
B. week 56. 'insolent'.
C. fragile A. impromptu
D. formidable B. indifferent
Ans - D C. indiscreet
Choose the correctly spelled word from the given D. impertinent
options.
Ans - D
51.A. dilemma
57. 'emulate'
B. dillemma
A. encompass
C. dilema
B. resist
D. dilemma
C. scold
Ans - A & D
D. imitate
52.A. occassionally
Ans - D
B. occasionally
58. 'alleviate'
C. ocassionally
A. aggravate
D. Ocassionally
B. mitigate
Ans - B
C. avoid
53. A. miliu
D. modify
B. milieu
Ans - B
C. milaeu
59. fastidious'
D. Milieau
A. meticulous
Ans - B
B. vehement
54. A. accomodate
C. ludicrous
B. acommodate
D. generous
C. accommodatte
Ans - A
D. accommodate
Which of the following sentences is correct?
Ans - D
60. A. Mr. Das is a friend of my father's.
Choose the word that is similar in meaning to the
B. Mr. Das is a friend of my father.
given word.
C. Mr. Das is a friend of mine father.
55. ’placate'
D. Mr. Das is the friend of my father's.
A. instigate
Ans - B
61. A. The only people interested in the book Fireman Exam 2023
seems to be lawyers.
1. I am going home ______ five days. (choose the
B. The only people interested in this book are correct option)
seeming to be lawyers.
A. on
C. The only people interested in the book are
B. at
seemingly to be lawyers.
C. in
D. The only people interested in the books seem to
be lawyers. D. no preposition
Ans - D Ans - C
62. A. The only India in the class is you. 2. If Raman had more time, he _____ more. (choose the
correct option)
B. The only Indian in the class are you.
C. The only Indian in the class to be you. A. travelled

D. The only Indian in the class have to be you. B. would travel

Ans - A C. would have travelled

Fill in the blanks by choosing the correct option D. would have been travelling
from among the ones given below. Ans - C
63. They ______ offended if I did not go to see 3. Please help me ______ the camp. (choose the correct
them. option)
A. will be A. setting
B. Would have been
B. set
C. were
C. for setting
D. would be
D. none of the above
Ans - D
Ans - C
64. If | had known you were in town I ______ you.
4. ______ you mind lending me your pen for a while?
A. would visit (choose the correct option)
B. would have visited A. could

C. could visit B. should

D. would have been visited C. may

Ans - B D. would

65. We met a lot of old friends ______ we were on Ans - D


holiday.
5. The most important of all my goals _______ to learn
A. while English.

B. for (choose the correct option)

C. during A. is

D. at B. are

Ans - A C. be
D. have been Ans - B
Ans - A 11.Inever liked sea food, but changed my opinion after
trying lobster. (Choose the correct
6. Which of the following options has the similar
meaning to the word ‘forbid'? phrasal verb from the options given below to replace
the underlined word)
A. forbear
A. came around
B. foretell
B. came up
C. prohibit
C. came down
D. propose
D. came out
Ans - C
Ans - A
7. Which of the following options has the opposite
meaning of the word 'extravagant? 12.To 'see eye to eye° means:

A. spendthrift A. to look tenderly at someone.

B. lavish B. to agree with someone

C. simple C. to argue with someone

D. thrifty D.to have an eye contact with someone

Ans - D Ans - B
8. A period often years is called a: 13. To 'call it a day' means:

A. century A. to stop working on something


B. decade B. to work better

C. millennium C. to start over


D. none of the above D. to be late for something

Ans - B Ans - A
9. A person who studies human societies and cultures is 14. I asked her what her problem was, but she ____
called: quiet.

A. a philologist (Choose the correct option.)

B. a pathologist A. Stayed
C. a humanist B. put

D. an anthropologist C. kept

Ans - D D. felt

10. The child resembles his grandfather. (Choose the Ans - C


correct phrasal verb from the options given below to
replace the underlined word) 15. She is a _____ athlete

A. looks after (Choose the correct option.)

D. takes after A. famous

C. falls after B. primary


D. looks for C. best
D. winner
Ans - A

You might also like